You are on page 1of 135

Versin 1.

0
Walter Mora F.
Textos Universitarios
md p
md p
md p
md p
md p
md p
md p
md p
md p
md p
md p
md p
md p
md p
md p
md p
md p
md p
md p
md p
md p
md p
md p
md p
md p
md p
md p
md p
md p
md p
md p
md p
Teora de Nmeros
Revista Digital Matemtica Educacin e Internet (www.cidse.itcr.ac.cr/revistamate)
5
md p
md p
md p
md p
md p
md p
md p
md p
md p
md p
md p
md p
md p
md p
md p
md p
md p
md p
md p
md p
md p
md p
md p
md p
md p
md p
md p
md p
md p
md p
md p
md p
INTRODUCCI

ONALATEOR

IA
DE N

UMEROS.
Versi on 1.0
Walter Mora F.
Revista digital Matem atica, Educaci on e Internet
(www.cidse.itcr.ac.cr/revistamate)
Escuela de Matem atica
Instituto Tecnol ogico de Costa Rica.
Textos Universitarios
Revista Digital Matemtica Educacin e Internet (www.cidse.itcr.ac.cr/revistamate)
Contenido
Prefacio 4
1 Fundamentos 1
1.1 Principios 1
1.2 Valor absoluto y la funci on sgn(x) 3
1.3 N umeros Poligonales 4
Ejercicios 8
2 Divisibilidad 9
2.1 Algoritmo de la divisi on 10
Ejercicios 11
2.2 Criba de Erat ostenes: C omo colar n umeros primos. 14
2.2.1 Algoritmo e implementaci on. 15
2.3 M aximo com un divisor 17
2.4 Algoritmo de Euclides. 19
2.4.1 Algoritmo e implementaci on. 21
2.5 Algoritmo Extendido de Euclides. 21
2.5.1 Algoritmo e implementaci on. 23
2.6 Ecuaciones Diof anticas lineales. 24
2.7 Teorema fundamental de la aritm etica 26
Ejercicios 30
3 Congruencias 37
3.1 Congruencias m odulo m 37
3.2 Calendarios: Qu e dia naci o Ud?. 40
3.3 Trucos de divisibilidad. 41
3.4 Cuadrados M agicos 41
3.5 Clases residuales m odulo m 43
3.6 Congruencias lineales 48
3.7 Teorema Chino del resto 50
3.8 Congruencias de Orden Superior 53
Ejercicios 54
4 Potencias mod m 57
4.1 Orden de un elemento m odulo m. 57
4.2 El Teorema peque no de Fermat. 58
4.3 Teorema de Euler 61
4.3.1 El recproco del Teorema peque no de Fermat 67
4.4 Teorema de Wilson 68
4.5 Teorema de Carmichael 70
Ejercicios 72
5 Raices primitivas y logaritmo discreto 76
5.1 Introducci on 76
5.2 Raices primitivas 76
5.3 Logaritmo discreto o Indicador 80
Ejercicios 84
6 Residuos Cuadr aticos 86
6.1 Congruencias cuadr aticas m odulo m 86
6.2 Criterio de Euler 88
6.3 Simbolos de Legendre y Jacobi 90
6.3.1 Lema de Gauss 94
6.3.2 Ley de Reciprocidad Cuadr atica. 97
6.4 Smbolo de Jacobi. 103
Ejercicios 105
7 Estimaciones, Estadsticas y Promedios 107
7.1 Funciones Aritm eticas 107
7.2 A los n umeros primos les gustan los juegos de azar. 111
7.3 Orden de Magnitud 113
7.4 Teorema de los n umeros primos 114
7.4.1 La funci on Zeta de Riemann 118
7.4.2 Teorema de Mertens. 122
7.5 N umeros Arm onicos 125
7.6 Acerca de los factores de un n umero grande 127
Ejercicios 129
Bibliografa 130
Bibliografa 130
3
Prefacio
Este texto corresponde a los apuntes de clase del curso Introducci on a la Teora
de N umeros. Los estudiantes del curso fueron: Beatriz Nu nez, Catalina Camacho,
Julian Arias, Hern an Vquez, Jery Barrantes, Andrea Mora, Bianca Chacon y Luis
Fernando Mora, de la sede de Paraso de la Universidad de Costa Rica.
Esta es la base te orica mnima para un texto m as extenso sobre algoritmos en teora
de n umeros y polinomios (en preparacion).
W. MORA.
Cartago, Costa Rica
Julio, 2009
1 FUNDAMENTOS
1.1 Principios
Los n umeros enteros son el ingrediente principal en teora de n umeros. En esta
introdu-ccion, establecemos brevemente la notacion y el signicado de algunos
smbolos que se relacionan con los enteros y que seran de amplio uso en el texto.
En lo que sigue, usaremos la siguiente notacion
1. N ={0, 1, 2, ...} y N
+
={1, 2, ...}.
2. Z
+
={1, 2, ...} =N
+
3. R
+
={x R : x > 0}.
Principio del Buen Orden: Todo conjunto no vaco de n umeros naturales
contiene un elemento mnimo.
En particular, si S Z tiene al menos un elemento positivo, entonces S tiene un
entero positivo mnimo.
EJEMPLO 1.1 Sea S ={2x +3y : x, y Z}. S Z y S tiene elementos positivos, por
ejemplo 2 1+3 0 =2. Por el principio del buen orden, S tiene un elemento positivo
mnimo. En este caso, el elemento positivo mnimo es 1. Para vericar esto solo
necesitamos probar que 1 S : Por ejemplo 1 = 2 2+3 1.
Principio del palomar: Si k es un entero positivo y k +1 o m as objetos son
asignados a k cajas, entonces hay al menos alguna caja a la que se le asignaron dos
o m as objetos.
EJEMPLO 1.2 En un grupo de 367 personas, debe haber al menos dos que cumplen
a nos el mismo da, porque hay solo 366 posibles das para cumplir a nos.
Introducci on a la Teora de N umeros.. Walter Mora F.
Derechos Reservados 2009 Revista digital Matem atica, Educacin e Internet (www.cidse.itcr.ac.cr/revistamate/)
1
2 FUNDAMENTOS
Principio del Inclusi on-Exclusi on: Sean A y B dos conjuntos nitos. En-
tonces |A B| =|A| +|B| |A B|.
EJEMPLO 1.3 Sea A = {2, 3, 4, 1} y B = {2, 3, 7, 8, 1}. |A| = 4, |B| = 5, |A B| =
|{1, 2, 3, 4, 7, 8}| = 6 y |A B| =|{2, 3, 1}| = 3. Luego, |A B| = 6 =|A| +|B| |A
B| = 4+53.
Principio de Inducci on: Para probar que una proposici on P(n) es verdadera
para todo entero positivo n, se deben ejecutar los dos pasos siguientes:
(1) Vericar que P(n) se cumple para n = 1,
(2) Probar que si se cumple P(k) (hip otesis de induccion), entonces se cumple
P(k +1)
Figura 1.1 Idea de inducci on matem atica usando un juego de domino.
Se puede probar que el principio de induccion es un metodo valido de prueba si
asumimos el principio del buen orden como un axioma.
EJEMPLO 1.4 Historicamente, el primer ejemplo que se conoce, en el que se us o in-
ducci on matematica, aparece en el libro Arithmeticorum Libri Duo de Francesco
Maurolico (1494-1575). En este libro, entre otras cosas, Maurolico presenta gran
variedad de propiedades de los enteros y las pruebas de estas propiedades. Para las
demostraciones, el ideo el metodo de induccion matematica. La primera vez que se
usa el metodo, es para probar que la suma de los primeros n enteros impares es n
2
.
El nombre inducci on matematica, lo us o por primera vez el matematico ingles
John Wallis.
Probar que 1+3+5+... +n = n
2
Solucion: En este caso, n indica el n umero de sumandos.
3
(1) La proposici on es correcta para n = 1 pues 1 = 1
2
(2) Hip otesis de induccion: suponemos que la proposici on es cierta para n = k, es
decir, 1+3+5+... +k = k
2
. Ahora, sumamos k +1 a ambos lados,
1+3+5+... +k +k +1 = k
2
+k +1 = (k +1)
2
.
Por lo tanto, hemos demostrado que si la proposici on es correcta para n = k,
es correcta para n = k +1. Entonces, la formula es valida para todo n N,
por el principio de induccion.
Principio de Inducci on Completa: Para probar que una proposici on P(n)
es verdadera para todo entero positivo n, se deben ejecutar los dos pasos siguientes:
(1) Vericar que P(n) se cumple para n = 1,
(2) Probar que si se cumple P(1) P(2) ... P(k) (hip otesis de induccion),
entonces se cumple P(k +1)
Se puede probar que el principio de induccion completa es equivalente al principio
de induccion. Es decir, cada principio puede ser demostrado asumiendo el otro. La
ganancia es que el principio de induccion completa es m as exible. A el principio
de induccion completa tambien se le llama principio de induccion fuerte o segundo
principio de induccion.
EJEMPLO 1.5 Si n es un entero mayor que uno, n se puede escribir como un producto
de primos. La demostracion de este hecho se hace con induccion fuerte. Puede ver
el teorema 2.13, que esta m as adelante.
1.2 Valor absoluto y la funcion sgn(x)
Muchas veces es conveniente separar el n umero y su signo. Para esto usamos la
funci on signo. En las aplicaciones es necesario que esta funci on solo tome dos
valores 1 y 1.
(1) Denimos sgn(x) = 1 si x 0 y sgn(x) =1 si x < 0.
(2) |a| = a sgn(a) = a/sgn(a).
Introducci on a la Teora de N umeros.. Walter Mora F.
Derechos Reservados 2009 Revista digital Matem atica, Educacin e Internet (www.cidse.itcr.ac.cr/revistamate/)
4 FUNDAMENTOS
EJEMPLO 1.6 | 5| =5 sgn(5) =5 1 = 5.
Las funciones [x_ [x] y x[
Si x R y n Z tal que n x < (n+1) entonces [x_ = n.
Si x R y n Z tal que n1 < x n entonces [x] = n.
x[ =
_
_
_
[x_ si x 0
[x] si x < 0
EJEMPLO 1.7 (a) 1.4 = 1 (b) 1.4 = 2 (c) 1.4 =2 (d) 1.4 =1
EJEMPLO 1.8 n = 2k o n = 2k +1 para alg un entero k, ya sea que n es par o impar.
(a) Si n = 2k +1 3, entonces
_
n3
2
_
=
_
2k 2
2
_
= k 1
(b) Si n = 2k 3, entonces
_
n3
2
_
=
_
2k 3
2
_
=
_
k 1
1
2
_
= k 2
(c) Si p = 8k 1 > 3, entonces
p1
2

_
p
4
_
= 4k 1(2k 1) = 2k
1.3 N umeros Poligonales
Los Pitag oricos usaban la palabra gnomon (= ) para referirse a los enteros im-
pares 1, 3, 5, 7, ... . Ellos observaron que n
2
es la suma de los n primeros impares,
1 = 1
2
,
1+3 = 2
2
,
1+3+5 = 3
2
,
1+3+5+7 = 4
2
,

En el ejemplo (1.4) ya habamos indicado que Francesco Maurolico (1494-1575),
prob o este hecho usando por primera vez, induccion matematica. Una prueba
geometrica se puede observar en la gura (1.2).
5
1+3+5 = 3
2
1+3 = 2
2
Figura 1.2 Cada cuadrado es construido agregando un n umero impar (los crculos azules)
Los n umeros cuadrados s
n
, corresponden a la cantidad de puntos en un arreglo
cuadrangular de nn. En este caso s
n
= n
2
, como acabamos de ver.
Los n umeros triangulares t
n
corresponden a la cantidad de crculos (o puntos u otra
cosa) en un arreglo triangular con n columnas, como se ve en la gura (1.3).
1
1
2
3
1
2
3
4
5
6
1
2
3
4
5
6
7
8
9
10
1
2
3
4
5
6
7
8
9
10
11
12
13
14
15
Figura 1.3 N umeros triangulares t
1
= 1, t
2
= 3, t
3
= 6, t
4
= 10, ...
Como cada columna tiene un elemento m as que la columna anterior, tenemos que
t
n
= 1+2+ +n1+n
Podemos tomar dos copias de t
n
y hacerlas encajar, de tal manera que obtengamos
un rectangulo,como se ve en la gura (1.4),
2 1
. 3 2
.
4 3
.
Figura 1.4 2t
1
= 2 1, 2t
2
= 3 2, 2t
3
= 4 3, ...
esto nos lleva de inmediato a una formula cerrada para t
n
,
t
n
=
n(n+1)
2
.
La gura (1.5) tambien constituye una prueba geometrica de la relacion entre
n umeros triangulares y cuadrados,
t
n
+t
n1
= s
n
6 FUNDAMENTOS
Figura 1.5 t
n
+t
n1
= s
n
Es facil responder la pregunta ?Cuando un n umero triangular es cuadrado?. Esto
sucede si t
n
= s
m
, ahora usamos nuestras formulas,
t
n
= s
m

n(n+1)
2
= m
2
(2n+1)
2
8m
2
= 1
Los n umeros tetraedricos T
n
son los analogos de los triangulares en 3D. Estos
n umeros son la cantidad de puntos en una pir amide tetraedica, como se observa en
la gura (1.6),
Figura 1.6 T
1
= 1, T
2
= 4, T
3
= 10, ...
Como la nesima capa es un arreglo triangular de t
n
puntos, entonces
T
n
=t
1
+t
2
+... +t
n
La prueba geometrica es un poco m as complicada. Se requiere usar cubos, en vez
de puntos, de tal manera que varias copias encajen perfectamente para formar un
cuboide. Por ejemplo, consideremos T
3
= 10, en la gura (1.7) se puede observar la
nueva conguracion de T
3
usando cubos. Las dos copias de T
3
ajustan bien, pero
no constituyen un cuboide.
Figura 1.7 Dos copias de T
3
= 10
Para lograr un cuboide necesitamos seis copias de T
3
, como se ve en la en la gura
(1.8)
7
Figura 1.8 Seis copias de T
3
= 10
Entonces, con seis copias de T
3
obtenemos un cuboide de orden 3(3+1)(3+2),
por tanto T
3
= 3(3 +1)(3 +2)/6. Generalizando, si sumamos seis copias de T
n
,
obtenemos es un cuboide de orden n(n+1) (n+2), es decir,
T
n
=
n(n+1)(n+2)
6
Los n umeros piramidales P
n
(de base cuadrada) corresponden a la cantidad de ob-
jetos en una pir amide de base cuadrada y altura n. En la gura (1.9) se muestra
una conguracion para P
4
= 30.
+ +
+
=
Figura 1.9 P
4
= 30
La kesima capa en la pir amide es un cuadrado con s
k
= k
2
objetos, entonces
P
n
= 1
2
+2
2
+... +n
2
Para obtener una formula para P
n
, usamos la relacion entre n umeros cuadrados y
n umeros triangulares, esto nos lleva a una expresi on en terminos de T
n
.
P
n
=
n

k=1
k
2
=
n

k=1
(t
k
+t
k1
)
=
n

k=1
t
k
+
n

k=1
t
k1
= T
n
+T
n1
=
n(n+1)(2n+1)
6
.
8 FUNDAMENTOS
En general, un n umero poligonal es un tipo de n umero gurado, que cuenta la
cantidad de objetos en un arreglo en forma de cuadrado, tri angulo, etc. La gura
(1.10) muestra algunos de estos arreglos,
1
1
2
3
1
2
3
4
5
6
1
2
3
4
5
6
7
8
9
10
1 1
2
3
4 1
2
3
4
5
6
7
8
9 1
2
3
4
5
6
7
8
9
10
11
12
13
14
15
16
1
1
2
3
4
5
1
2
3
4
5
6
7
8
9
10
11
12
1
2
3
4
5
6
7
8
9
10
11
12
13
14
15
16
17
18
19
20
21
22
1 1
2
3
4
5
6 1
2
3
4
5
6
7
8
9
10
11
12
13
14
15 1
2
3
4
5
6
7
8
9
10
11
12
13
14
15
16
17
18
19
20
21
22
23
24
25
26
27
28
1
1
2
3
4
5
6
7
1
2
3
4
5
6
7
8
9
10
11
12
13
14
15
16
17
18
1
2
3
4
5
6
7
8
9
10
11
12
13
14
15
16
17
18
19
20
21
22
23
24
25
26
27
28
29
30
31
32
33
34
Figura 1.10 N umeros poligonales
EJERCICIOS
1.1 Probar, usando el principio de induccion, las formulas para s
n
, t
n
, y T
n
1.2 Muestre que 8t
n
+1 = s
2n+1
1.3 Probar, usando induccion, que si a y n son enteros positivos, existe otro
entero positivo m tal que am > n
1.4 Use induccion para probar que 1+2
3
+3
3
+... +n
3
= n
2
(n+1)
2
/4
1.5 Probar la formula para la suma de los primeros n terminos en una progresion
aritmetica,
a+(a+d) +(a+2d) +... +[a+(nl)d] = na+
n(n1)
2
d
1.6 Probar que si x = 1 es un n umero real jo, entonces
1+x +x
2
+x
3
+ +x
k
=
1x
k+1
1x
, k N
2 DIVISIBILIDAD
Denici on 2.1 Sean a, b enteros con b = 0. Decimos que b divide a a si existe un
entero c tal que a = bc. Si b divide a a escribimos b|a
Teorema 2.1 Sean a, b, d, p, q Z.
1. Si d|a y d|b entonces d|(ax +by) para cualquier x, y Z
2. Si d|(p+q) y d|p = d|q.
3. Si a, b Z
+
y b|a = a b
4. Si a|b, entonces a|mb, con m Z.
5. Si a, b Z, a|b y b|a = |a| =|b|
Prueba:
1. Sea a = nd y b = md, entonces ax +by = (nx +my)d =d|(ax +by)
2. Sea p = kd y p+q = k

d, entonces q = d(k

k) = d|q
3. Sea a = kb, como k 1 = bk b.
4. Sea b = ka = mb = mka = (mk)a = a|mb
5. El item (3) solo aplica si a y b son positivos. Usando el item (4), si a|b y
b|a entonces, |a|||b| y |b|||a|, luego: |a| |b| y |b| |a|. |a| =|b|
EJEMPLO 2.1 5| 5 y 5|5. |5| =| 5|
EJEMPLO 2.2 Sean a, b, d Z. Muestre que si a|d y d|b entonces a|b
Solucion: Si a|d d|b = d = k
1
a b = k
2
d, k
1
, k
2
Z.
Luego b = k
2
d = k
2
(k
1
a) = a|b
Introducci on a la Teora de N umeros.. Walter Mora F.
Derechos Reservados 2009 Revista digital Matem atica, Educacin e Internet (www.cidse.itcr.ac.cr/revistamate/)
9
10 DIVISIBILIDAD
2.1 Algoritmo de la division
Si la divisi on no es exacta, no todo esta perdido: Como hacamos en la escuela, la
divisi on de a por b la podemos expresar como un cociente y un resto. Por ejemplo,
la divisi on de 23 por 3 es 7 y queda un resto r = 2. Es decir, 23 = 7 3 + 2.
Gracamente,
7 3 23 (7+1) 3
r = 2
Teorema 2.2 (Algoritmo de la divisi on) Sean a, b Z con b = 0. Existen q, r Z
unicos tales que
a = bq+r con 0 r <|b|.
Gracamente,
qb a (q+1)b
r
Prueba: Vamos a demostrar el teorema para a, b Z con b > 0. Con esto, el teo-
rema sera tambien valido para el caso b <0, ya que tendramos que existen q, r Z
unicos tales que a =|b|q+r con 0 r <|b| con lo que a = b (q) + r con 0
r <|b|.
Consideremos la progresion aritmetica
..., 3b, 2b, b, 0, b, 2b, 3b, ...
existe q Z tal que
qb a < (q+1)b
Sea r =aqb, entonces a =bq+r. De qb a obtenemos 0 r y de a <(q+1)b =
aqb < b. a = bq+r con 0 r < b
Unicidad: Supongamos que existe q
1
, r
1
Z tal que
a = bq
1
+r
1
con 0 r
1
< b y a = bq+r con 0 r < b
Entonces b(q
1
q) = r r
1
= b|(r r
1
)
Ahora supongamos que r = r
1
y que r < r
1
. En este caso
0 < r r
1
< r < b = b (r r
1
) : Contradiccion (ejemplo )
Por lo tanto, r = r
1
. De aqu: b(q
1
q) = r r
1
= 0 = q
1
= q
EJERCICIOS 11
Nota 1: Si a, b Z
+
, el algoritmo de la divisi on corresponde a la divisi on usual. Si
a o b es negativo, la divisi on usual diere del algoritmo de la divisi on.
Nota 2: Para efectos de c alculo es mejor enunciar el algoritmo de la divisi on as:
Teorema 2.3 (Algoritmo de la divisi on) Sean a, b Z con b = 0. Existe r Z unico
tal que
Si b > 0, a = b[a/b_+r con 0 r < b.
Si b < 0, a =b [a/|b|_+r con 0 r <|b|.
En este contexto, a/b denota la divisi on usual en R.
El resto de la divisi on de a por b se denota rem(a, b) o tambien a mod b. En
forma compacta,
a mod b = rem(a, b) = ab sgn(b)[a/|b|_, b = 0.
Si a y b son positivos, esta funci on coincide con la funci on Mod de VBA y %
de Java.
EJEMPLO 2.3 Dividir 12 por 5 :
12 5,
10 2
2
En la divisi on ordinaria 12 = 2 5 2.
Desde el punto de vista del algoritmo de la di-
vision, como 3 5 12 <2 5, se tiene
12 =3 5+3
EJEMPLO 2.4 Si a = 2q+r con 0 r < 2. Si r = 0 a se dice par y si r = 1, a se dice
impar.
EJERCICIOS
2.1 De un contraejemplo de la armaci on: a|bc y a c entonces a|b
Solucion: 6|2 3 pero 6 2 y 6 3
2.2 Mostrar que si d|a d|(a+1) entonces |d| = 1
Solucion: si d|a d|(a+1) = d|(a+1a) Luego, d|1 = d =1.
2.3 Sean d, n Z. Si d no divide a n entonces ning un m ultiplo de d divide a n.
Solucion: Si kd|n = n = k

kd = d|n ()
12 DIVISIBILIDAD
2.4 Si d|a y d|b y si a = bq+r entonces d|r.
Solucion: Como d|a y d|b = d|(abq) = d|r
2.5 Sea b = 0 y a = qb +r con 0 r < |b|. Muestre que en el conjunto {a, a
1, ..., a|b| +1} hay un unico m ultiplo de b.
Solucion: Como a r = bq y 0 r < |b|, bq debe ser uno de los n umeros
{a, a1, ..., a|b| +1}
2.6 Muestre que si a, b, d Z, a impar y si d|a y d|(ab+2), entonces d = 1
Solucion: d|a y d|(ab+2) = d|ab d|(ab+2) = d|2 = d = 1, d = 2, pero
como a es impar, d = 1.
Denici on 2.2 (Primos y compuestos) Un entero p > 1 se dice primo si sus unicos
divisores son 1 y p. Si p no es primo, se dice compuesto.
EJEMPLO 2.5 Los primeros primos son {2, 3, 5, 7, 11, 13, 17, ...}
EJEMPLO 2.6 Sea p
i
el iesimo primo. El n umero N = p
1
p
2
p
n
+1 puede ser o
no ser primo. Por ejemplo,
N = 2 3 5 7 11 + 1 = 2311 es primo,
N = 2 3 5 7 11 13 + 1 = 30031 = 59 509 no es primo.
Lema 2.1 Todo entero positivo n > 1 tiene un divisor primo
Prueba: Si n es primo, tiene un divisor primo (el mismo). Supongamos que n es
compuesto. Por el principio del buen orden podemos suponer que existe un d > 1
que es el m as peque no divisor positivo de n. Entonces d es primo. En efecto, si
d fuera compuesto, d tendra un divisor 1 < d
1
< d. Pero si d
1
|d y d|n entonces
d
1
|n, en contradiccion con la suposici on de que d era el m as peque no divisor > 1,
de n.
Corolario 2.1 Sea n Z, n > 1. El m as peque no divisor positivo d > 1 de n es
primo.
C omo decidir si n es primo?
EJERCICIOS 13
El problema de decidir si es un n umero es primo no es, en general, facil. Si n es un
n umero muy grande, probar que n no es divisible por ning un n umero excepto 1 y
n, nos llevara a hacer un n umero nada razonable de c alculos. El siguiente teorema
nos dice que para determinar si un n umero n es primo o no, basta con probar con
los divisores primos inferiores a

n. Aunque

n es, en general, peque no respecto
a n, este metodo tiene un alcance muy limitado.
Teorema 2.4 Sean a, b, n N, n > 1 y m > 1.
(1) Si n = ab, entonces a

n b

n.
(2) Si n no tiene divisores primos

n, entonces n es primo.
Prueba: Probamos (1) por contradiccion, si a >

n b >

n = ab > n. ().
Prueba de (2): Sea m compuesto y m = ab y sea p el divisor primo m as peque no
de m y p >

m. Entonces ab p
2
> m ()
Corolario 2.2 Si n es compuesto, n tiene un divisor primo p

n.
EJEMPLO 2.7 n = 103 es primo?
Solucion: No pues 103 no es divisible por ning un primo inferior a

103 10.1 En
efecto, los primos inferiores a 10 son2, 3, 5 y 7. Para probar que n no es divisible por
alguno de estos n umeros calculamos los residuos: rem(103, 2) =1, rem(103, 3) =1,
rem(103, 5) = 3 y rem(103, 7) = 5.
n = 2311 es primo?
Solucion: Si es primo. En efecto, si no fuera primo, n =2311 tendra un divisor primo
p con p

2311 =48.07... . Los primos inferiores a 48 son {2, 3, 5, 7, 11, 13, 17, 19, 23, 29,
31, 37, 41, 43, 47} pero ninguno de ellos divide a 2311.
Cu antos primos hay?
Los primos son innitos. Es algo que se conoce desde la epoca d Euclides.
Teorema 2.5 (Euclides) Hay un n umero innito de primos
Prueba: La demostracion es por contradiccion: Si p
1
, . . . , p
n
fueran todos los primos,
el n umero N = p
1
p
2
p
n
+1 o es un nuevo primo o tiene un divisor primo diferente
de cada p
i
, i = 1, 2, ..., n. En efecto: Si N es primo, N > p
i
, i = 1, 2, ..., n y entonces
sera un nuevo primo. Si N no es un nuevo primo, tiene un divisor primo p
j
, pero
entonces como p
j
|(p
1
p
2
p
n
+ 1) y p
j
|(p
1
p
2
p
n
) = p
j
|1 lo cual es imposible
pues p
j
> 1.
14 DIVISIBILIDAD
La pregunta correcta es cuantos primos hay x?. (x) denota la cantidad de
primos inferiores o iguales x. Por ejemplo, (5) = (6) = 3. Al 2008, se conocen
todos los primos inferiores a x = 100, 000, 000, 000, 000, 000, 000, 000 = 10
23
. Se tiene
(10
23
) = 1925320391606803968923. Tambien se conocen algunos primos fuera
de estos, son primos especiales, por ejemplo 19249 2
13018586
+1 es un primo con
3918990 dgitos; fue encontrado en el 2007 por Samuel Yates.
Actualmente, la manera m as eciente de colar primos peque nos, es usando la
criba de Eratostenes.
2.2 Criba de Eratostenes: Como colar n umeros primos.
La criba
1
de Eratostenes es un algoritmo que permite colar todos los n umeros pri-
mos menores que un n umero natural dado n, eliminando los n umeros compuestos de
la lista {2, ..., n}. Es simple y razonablemente eciente hasta, digamos n =1000000.
Primero tomamos una lista de n umeros {2, 3, ..., n} y eliminamos de la lista los
m ultiplos de 2. Luego tomamos el primer entero despues de 2 que no fue borrado
(el 3) y eliminamos de la lista sus m ultiplos, y as sucesivamente. Los n umeros que
permanecen en la lista son los primos {2, 3, 5, 7, ...}.
EJEMPLO 2.8 Primos menores que n = 10
Lista inicial 2 3 4 5 6 7 8 9 10
Eliminar m ultiplos de 2 2 3

4 5

6 7

8 9
&&10
Resultado 2 3 5 7 9
Eliminar m ultiplos de 3 2 3 5 7

9
Resultado 2 3 5 7
Primer renamiento: Tachar solo los impares
Excepto el 2, los pares no son primos, as que podramos tachar solo sobre la
lista de impares n :
{3, 5, 9, ..., } =
_
2i +3 : i = 0, 1, ...
_
n3
2
__
El ultimo impar es n o n1. En cualquier caso, el ultimo impar es 2

n3
2

+3 pues:
1
Criba, tamiz y zaranda son sin onimos. Una criba es un herramienta que consiste de un cedazo
usada para limpiar el trigo u otras semillas, de impurezas. Esta acci on de limpiar se le dice cribar
o tamizar.
Introducci on a la Teora de N umeros.. Walter Mora F.
Derechos Reservados 2009 Revista digital Matem atica, Educacin e Internet (www.cidse.itcr.ac.cr/revistamate/)
EJERCICIOS 15
Si n es impar, n = 2k +1 y

n3
2

= k 1 = 2(k 1) +3 = n.
Si n es par, n = 2k y

n3
2

= k 2 = 2(k 2) +3 = 2k 1 = n1.
Segundo renamiento: Tachar de p
2
k
en adelante
En el paso kesimo hay que tachar los m ultiplos del primo p
k
desde p
2
k
en adelante.
Esto es as pues en los pasos anteriores se ya se tacharon 3 p
k
, 5 p
k
, ..., p
k1
p
k
.
Por ejemplo, cuando nos toca tachar los m ultiplos del primo 7, ya se han eliminado
los m ultiplos de 2, 3 y 5, es decir, ya se han eliminado 2 7, 3 7, 4 7, 5 7 y 6 7.
Por eso iniciamos en 7
2
.
Tercer renamiento: Tachar mientras p
2
k
n
En el paso kesimo hay que tachar los m ultiplos del primo p
k
solo si p
2
k
n. En
otro caso, nos detenemos ah. Porque?. En el paso kesimo tachamos los m ultiplos
del primo p
k
desde p
2
k
en adelante, as que si p
2
k
> n ya no hay nada que tachar.
EJEMPLO 2.9 Encontrar los primos menores que 20. El proceso termina cuando el
cuadrado del mayor n umero conrmado como primo es < 20.
1. La lista inicial es {2, 3, 5, 7, 9, 11, 13, 15, 17, 19}
2. Como 3
2
20, tachamos los m ultiplos de 3 desde 3
2
= 9 en adelante:
{2, 3, 5, 7,

9, 11, 13,
&
& 15, 17, 19}
3. Como 5
2
> 20 el proceso termina aqu.
4. Primos < 20 : {2, 3, 5, 7, 11, 13, 17, 19}
2.2.1 Algoritmo e implementaci on.
En este contexto, a/b divisi on entera, es decir, a/b es el cociente de dividir a y b.
1. Como ya vimos, para colar los primos en el conjunto {2, 3, ..., n} solo consid-
eramos los impares:
{2i +3 : i = 0, 1, ... (n3)/2[} ={3, 5, 7, 9, ...}
2. Por cada primo p = 2i +3 (tal que p
2
< n), debemos eliminar los m ultiplos
impares de p menores que n, a saber
(2k +1)p = (2k +1)(2i +3), k = i +1, i +2, ...
16 DIVISIBILIDAD
Observe que si k = i +1 entonces el primer m ultiplo en ser eliminado es
p
2
= (2i +3)(2i +3), como debe ser.
Esto nos dice que para implementar el algoritmo solo necesitamos un arreglo (booleano)
de tama no quo(n-3,2). En Java se pone (n-3)/2 y en VBA se pone (n-3)2.
El arreglo lo llamamos EsPrimo[i], i=0,1,...,(n-3)/2.Cada entrada del arreglo Es-
Primo[i] indica si el n umero 2i +3 es primo o no.
Por ejemplo
EsPrimo[0] = true pues n = 2 0+3 = 3 es primo,
EsPrimo[1] = true pues n = 2 1+3 = 5 es primo,
EsPrimo[2] = true pues n = 2 2+3 = 7 es primo,
EsPrimo[3] = false pues n = 2 3+3 = 9 no es primo.
Si el n umero p = 2i +3 es primo entonces i = (p3)/2 y
EsPrimo[(p-3)/2] = true.
Si sabemos que p = 2i +3 es primo, debemos poner
EsPrimo[((2k+1)(2i+3) - 3)/2] = false
pues estas entradas representan a los m ultiplos (2k +1)(2i +3) de p. Observe
que cuando i = 0, 1, 2 tachamos los m ultiplos de 3, 5 y 7; cuando i = 3 entonces
2i +3 = 9 pero en este momento esPrimo[3]=false as que proseguimos con i = 4,
es decir, proseguimos tachando los m ultiplos de 11.
En resumen: Antes de empezar a tachar los m ultiplos de p = 2i +3 debemos pre-
guntar si esPrimo[i]=true.
EJERCICIOS 17
Algoritmo 2.1: Criba de Eratostenes
Entrada: n N, n > 3
Resultado: Primos entre 2 y n
m ax= (n3)/2; 1
boolean esPrimo[i], i = 1, 2, ...,m ax; 2
for i = 1, 2, ..., m ax do 3
esPrimo[i] =True; 4
i = 0; 5
while (2i +3)(2i +3) n do 6
k = i +1; 7
if esPrimo(i) then 8
while (2k +1)(2i +3) n do 9
esPrimo[((2k +1)(2i +3) 3)/2] =False; 10
k = k +1; 11
i = i +1; 12
Imprimir; 13
for j = 1, 2, ..., m ax do 14
if esPrimo[ j] =True then 15
Imprima j 16
Nota: Es conveniente poner (2i +3) n/(2i +3) en vez de (2i +3)(2i +3) n, para
no operar con n umeros innecesariamente grandes.
2.3 Maximo com un divisor
Si a, b son enteros no ambos nulos, entonces d es un divisor com un de a y b si
d|a y d|b. Si denotamos D
a
al conjunto de divisores de a y a D
b
al conjunto de
divisores de b. Estos conjuntos no son vacos pues al menos 1 D
a
y 1 D
b
. El
m aximo com un divisor com un de a y a es el m as grande entero positivo del con-
junto D
a
D
b
.
EJEMPLO 2.10 Como D
3
={3, 1, 1, 3} y D
6
={6, 3, 2, 1}, entonces D
a

D
b
={3, 3, 1, 1}. Por tanto, el m aximo com un divisor de 3 y 6 es 3.
Antes de continuar, una pregunta porque a y b no pueden ser ambos nulos?
Una denicion m as tecnica y apropiada para el desarrollo te orica es,
Introducci on a la Teora de N umeros.. Walter Mora F.
Derechos Reservados 2009 Revista digital Matem atica, Educacin e Internet (www.cidse.itcr.ac.cr/revistamate/)
18 DIVISIBILIDAD
Denici on 2.3 (M aximo Com un Divisor) Sean a, b enteros con al menos uno de los
dos diferente de cero. El m aximo com un divisor de a y b, denotado mcd(a, b), es
el entero positivo d que satisface:
(1) d|a y d|b
(2) Si c|a y c|b, entonces c|d
Si mcd(a, b) = 1 se dice que a y b son relativamente primos o simplemente cop-
rimos.
EJEMPLO 2.11 mcd(3, 6) = mcd(3, 6) = mcd(3, 6) = 3.
Ahora establecemos algunas propiedades utiles. Otras propiedades seran enunci-
adas m as adelante, cuando tengamos m as herramientas para hacer las pruebas.
Teorema 2.6 Sean a, b Z, ambos no nulos.
(1) mcd(a, 0) =|a| si a = 0
(2) mcd(a, b) = mcd(|a|, |b|)
(3) Si d = mcd(a, b), entonces mcd(a/d, b/d) = 1
(4) Si d = mcd(a, b), entonces mcd(a, anb) = d con n Z
Prueba: : Para probar (1) sea d = mcd(a, 0). Como |a| |a y |a| |0, entonces |a| |d.
Pero d| |a| = d |a|. d =|a|.
Para probar (2) sea d =mcd(a, b) y d
1
=mcd(|a|, |b|). Como d|a d|b = d| |a|
d| |b| = d|d
1
. Ahora como d
1
| |a| d
1
| |b| = d
1
| a d
1
| b = d
1
|d. d = d
1
por ser ambos positivos.
Para probar (3), sea d

= mcd(a/d, b/d), entonces hay enteros k, k

tales que a/d =


kd

y b/d =k

. Por tanto, a =dkd

y b =dk

, es decir, dd

|a dd

|b = dd

d,
por denicion, entonces d

es un entero positivo 1, es decir, d

= 1.
Para probar (4), sea d
1
= mcd(a, b na). Como d|a d|b = d|(b na) por el
teorema 2.1, (1). Entonces d d
1
. Como d
1
|a d
1
|(b na) = d
1
|b por el teo-
rema 2.1, (2). Entonces d
1
d. d = d
1
EJEMPLO 2.12 Muestre que si p es primo, mcd(a, p) = p o 1.
Solucion: Si d = mcd(a, p), en particular, d|p, por tanto, como p es primo, d =
1 d = p.
EJERCICIOS 19
EJEMPLO 2.13 Muestre que si n >1, p es primo y p|n
2
+1, entonces mcd(p, n) =1.
Solucion: mcd(p, n) = 1 p. Si mcd(p, n) = p entonces p|n = p|n
2
y como
p|n
2
+1, p|1, contradiccion. mcd(p, n) = 1.
EJEMPLO 2.14 Sea d = mcd(a, b). Si a = kd y b = k

d, entonces mcd(k, k

) = 1.
Solucion: El teorema 2.6, (3) dice que mcd(a/d, b/d) = 1, es decir, mcd(k, k

) = 1.
En la practica necesitamos calcular el m aximo com un divisor de varios n umeros.
Esto no es problema, el siguiente teorema nos dice que el m aximo com un divisor de
varios n umeros se puede calcular de la misma manera en la que sumamos: De dos
en dos.
Teorema 2.7 mcd(a
1
, a
2
, ..., a
n
) = mcd(a
1
, mcd(a
2
, ..., a
n
)).
Prueba: Sea d = mcd(a
1
, a
2
, ..., a
n
) y d
1
= mcd(a
1
, d
2
) con d
2
= mcd(a
2
, ..., a
n
).
Como d|a
1
, d|a
2
, ..., d|a
n
, entonces d|d
2
y por tanto d|d
1
. Como d
1
|a
1
y d
1
|d
2
en-
tonces d
1
|a
1
, d
1
|a
2
, ..., d
1
|a
n
(por transitividad). Por tanto d
1
|d d = d
1
.
Corolario 2.3 Sean a
1
, ..., a
n
Z, si
mcd(a
1
, a
2
) = d
2
,
mcd(d
2
, a
3
) = d
3
,
...
mcd(d
n1
, a
n
) = d
n
,
_

_
= d
n
= mcd(a
1
, ..., a
n
).
EJEMPLO 2.15 mcd(3, 6, 12) = mcd(mcd(3, 6), 12) = mcd(3, 12) = 3.
2.4 Algoritmo de Euclides.
El algoritmo de Euclides se basa en la aplicaci on sucesiva del lema
Lema 2.2 Sean a, b, q, r Z tales que a = bq +r con b > 0 y 0 r < b. Entonces
mcd(a, b) = mcd(b, r).
Introducci on a la Teora de N umeros.. Walter Mora F.
Derechos Reservados 2009 Revista digital Matem atica, Educacin e Internet (www.cidse.itcr.ac.cr/revistamate/)
20 DIVISIBILIDAD
Prueba: Como r = abq el resultado es consecuencia de la parte (4) del teorema
(2.6).
Este resultado lo podemos usar para obtener un metodo muy eciente para calcular
el m aximo com un divisor de dos n umeros.
Teorema 2.8 (Algoritmo de Euclides) Sean a y b son n umeros naturales, b = 0.
Aplicando el algoritmo de la divisi on se obtiene una sucesion nita a, r
0
=b, r
1
, r
2
, ..., r
n
denida por
a = r
0
q
1
+r
1
, 0 r
1
< r
0
r
0
= r
1
q
2
+r
2
, 0 r
2
< r
1
r
1
= r
2
q
3
+r
3
, 0 r
3
< r
2
.
.
.
r
n2
= r
n1
q
n
+r
n
, 0 r
n
< r
n1
r
n1
= r
n
q
n+1
+0
Entonces r
n
= mcd(a, b).
Prueba: Aplicando el algoritmo de la divisi on obtenemos una sucesion decreciente
de residuos 0 ... < r
k
< r
k1
< ... < r
1
< r
0
. La sucesion es nita pues entre 0 y
r
0
solo puede haber un n umero nito de enteros. Por tanto, debe haber un residuo
mnimo. Si este mnimo es r
n
0, entonces r
n+1
= 0.
De acuerdo al lema (2.2) tenemos que
mcd(a, b) = mcd(ar
0
q, r
0
)
= mcd(r
1
, r
0
)
= mcd(r
1
, r
0
r
1
q
2
)
= mcd(r
1
, r
2
)
= mcd(r
1
r
2
q
2
, r
2
)
= mcd(r
3
, r
2
)
Usando induccion matematica,
mcd(a, b) = mcd(r
n1
, r
n
) = mcd(r
n
, 0) = r
n
. (2.1)
EJEMPLO 2.16 Vamos a aplicar el algoritmo de Euclides para calcular mcd(8, 2) y
mcd(78, 32).
EJERCICIOS 21
(1) mcd(8, 2) = 2. En efecto; (2) mcd(78, 32) = 2. En efecto;
8 = 2 4+0
As, r
0
= 2 y r
1
= 0.
78 = 32 2+14
32 = 14 2+4
14 = 4 3+2
4 = 2 2+0
2.4.1 Algoritmo e implementaci on.
Recordemos que ( mod a, b) denota el resto de la divisi on de a por b.
Algoritmo 2.2: Maximo com un divisor
Entrada: a, b Z.
Resultado: mcd(a, b)
if a = 0 y b = 0 then 1
return mcd(a, b) = 0 2
c =|a|, d =|b| ; 3
while d = 0 do 4
r = rem(c, d); 5
c = d; 6
d = r; 7
return mcd(a, b) =|c|; 8
2.5 Algoritmo Extendido de Euclides.
El siguiente teorema establece la llamada Identidad de Etienne Bezout aunque
el resultado lo descubri o primero el frances Claude Gaspard Bachet de Meziriac
(1581-1638).
Teorema 2.9 (Identidad de B ezout) Si a, b son dos enteros no ambos cero, existen
s, t Z (posiblemente no unicos) tales que
sa+tb = mcd(a, b)
Prueba: Considere el conjunto A = {ua + vb : u, v Z}. Este conjunto tiene
n umeros positivos, negativos y el cero. Sea m = ax + by el m as peque no entero
positivo en A.
Sea a = mq+r con 0 r < m. Entonces,
0 r = amq = a(ax + by)q = (1qx)a+(qy)b
Introducci on a la Teora de N umeros.. Walter Mora F.
Derechos Reservados 2009 Revista digital Matem atica, Educacin e Internet (www.cidse.itcr.ac.cr/revistamate/)
22 DIVISIBILIDAD
As, r es una combinacion lineal de a y b. Pero r < m, as que r = 0 por ser m el
mnimo entero positivo en A.
Luego, a =mq y m|a. De manera analoga podemos establecer que m|b. Por lo tanto
m es com un divisor de a y b y debe ser d m. Pero, como a = k
1
d y b = k
2
d en-
tonces m = ax +by = (xk
1
+yk
2
)d > 0 por lo tanto m d. As que m = d.
Prueba alternativa: Por induccion. Consideremos la sucesion r
1
, r
2
, ..., r
n
del algo-
ritmo de Euclides.
Como r
1
= abq
0
y b = r
1
q
1
+r
2
entonces r
2
es combinacion lineal de a y b.
Como r
1
y r
2
son combinaciones lineales de a y b y como r
1
= r
2
q
2
+r
3
entonces
r
3
es combinacion lineal de a y b.
Continuando de esta manera, como r
i1
y r
i2
son combinaciones lineales de a y
b y como r
i2
= r
i1
q
2
+r
i
(i = 2, ..., n), entonces r
n
es combinacion lineal de a y
b.
El siguiente corolario es sumamente util,
Corolario 2.4 El mcd(a, b) es la m as peque no n umero positivo de la forma sa+tb.
En particular, si ax +by = 1 = mcd(a, b) = 1.
EJEMPLO 2.17 Si n es entero positivo, verique que
n+2
n+1
es irreducible.
Solucion: Si n > 0, n+2(n+1) = 1; entonces, seg un el corolario (2.4), mcd(n+
2, n+1) = 1.
La ecuaci on sa+tb = mcd(a, b) no tiene soluci on unica para s, t enteros. Se puede
obtener una soluci on despejando los residuos, en el algoritmo de Euclides, y ha-
ciendo una sustitucion hacia atr as.
EJEMPLO 2.18 mcd(78, 32) =2. De acuerdo a la identidad d Bezout, existen s, t Z
tal que s 78 + t 32 = 2. En este caso, una posibilidad es 7 78 17 32 = 2, es
decir s = 7 y t =17.
s y t se pueden obtener as: primero despejamos los residuos en el algoritmo de
Euclides de abajo hacia arriba, iniciando con el m aximo com un divisor,
78 = 32 2+14 14 = 7832 2
32 = 14 2+4 4 = 3214 2
14 = 4 3+2 2 = 144 3
4 = 2 2+0
EJERCICIOS 23
Ahora hacemos sustitucion hacia atr as, sustituyendo las expresiones de los residuos.
En cada paso se ha subraya el residuo que se sustituye
2 = 144 3
= 14(3214 2)3
= 14 732 3
= (7832 2)732 3
= 7 78 17 32
EJEMPLO 2.19 Calcular s, t Z tal que s 8+t 22 = mcd(8, 22).
Solucion:
Calcular mcd(8, 22) Calculo de s y t
8 = 1 22+14
22 = 1 14+8
14 = 1 8+6
8 = 1 6+2
6 = 3 2+0
2 = 68
= 148 (2214)
= (8+22) 8 (22(8+22))
= 3 8 1 22
s =3 y t =1
2.5.1 Algoritmo e implementaci on.
Observaci on: En este contexto, a/b denota el cociente de dividir a y b.
Algoritmo 2.3: Algoritmo Extendido de Euclides
Entrada: a, b enteros no ambos nulos
Resultado: mcd(a, b), t y s
c =|a|, d =|b| ; 1
c
1
= 1, d
1
= 0; 2
c
2
= 0, d
2
= 1; 3
while d = 0 do 4
q = c/d, r = c qd,
r
1
= c
1
qd
1
, r
2
= c
2
qd
2
,
c = d, c
1
= d
1
, c
2
= d
2
,
d = r, d
1
= r
1
, d
2
= r
2
, 5
return mcd (a,b) =|c|, s = c
1
/sgn(a) sgn(c), t = c
2
/sgn(b) sgn(c); 6
En este algoritmo, sgn(x) =
_
1 si x 0
1 si x < 0
.
Validez del algoritmo
24 DIVISIBILIDAD
La validez de este algoritmo se establece probando que en todo el ciclo While
c = c
1
|a| +c
2
|b| (2.2)
d = d
1
|a| +d
2
|b|
Al nal, cuando d = 0, obtenemos s y t despejando en (2.2).
Como |x| = x/sgn(x) = x sgn(x), entonces
|c| =
c
1
sgn(c)
|a| +
c
2
sgn(c)
|b|
=
c
1
sgn(c) sgn(a)
. .
s
a+
c
2
sgn(c) sgn(b)
. .
t
b
2.6 Ecuaciones Diofanticas lineales.
Consideremos el problema de resolver ax+by = c en enteros. Aqu a, b, c son dados
y se debe determinar x, y. Las condiciones de existencia de soluciones y el metodo
para obtenerlas se basa en el algoritmo extendido de Euclides.
EJEMPLO 2.20 Consideremos la ecuaciones en enteros 2x +3y = 2 y 6x 3y = 1
Gracamente, las soluciones enteras son los pares (x, y) contenidos en la recta. En el
caso de 2x+3y =2, en la gura (2.1) se puede observar que (5, 4), (2, 2), (1, 0), (4, 2), (7, 4)
son algunas soluciones. En el caso de la recta 6x3y =1, no se observan soluciones;
tendra alguna?
-5
2
4
6
5
Figura 2.1 Algunas soluciones enteras de la ecuaci on 2x +3y = 2.
Teorema 2.10 La ecuaci on diofantica lineal ax +by = c tiene soluci on si y s olo si
mcd(a, b)|c
Introducci on a la Teora de N umeros.. Walter Mora F.
Derechos Reservados 2009 Revista digital Matem atica, Educacin e Internet (www.cidse.itcr.ac.cr/revistamate/)
EJERCICIOS 25
Prueba: Sea d = mcd(a, b)
: Si c = ax +by tiene soluci on, entonces d|c pues d|a y d|b.
: Si d|c, c = kd. Como podemos determinar, usando el algoritmo de Euclides,
s, t Z tal que d = sa +tb, entonces c = kd = (ks)a +(kt)b y una soluci on de la
ecuaci on diofantica lineal sera x = ks y y = kt.
EJEMPLO 2.21 La ecuaci on en enteros 2x+3y =2 tiene soluci on pues mcd(2, 3) =1|2
y 6x 3y = 1 no tiene soluciones enteras pues mcd(6, 3) = 3 1.
EJEMPLO 2.22 Calcule una soluci on para 8x +22y = 20
Solucion: En el ejemplo (2.18) establecimos que 2 = mcd(8, 22) =3 8 1 22.
Ahora, como 20 = 2 10,
2 =3 8 1 22 = 20 =30 810 22 =x =30 y =10.
Soluci on general.
La soluci on general se establece al estilo de las ecuaciones diferenciales y el algebra
lineal: Primero se busca la soluci on de la ecuaci on homogenea ax +by = 0 y la
soluci on general de la ecuaci on ax +by = c se expresa usando estas soluciones.
Teorema 2.11 Sea d = mcd(a, b). La ecuaci on diofantica lineal homogenea ax+by =
0 tiene soluciones x =
b
d
k, y =
a
d
k con k Z.
Prueba: ax +by = 0 = ax = by =
a
d
x =
b
d
y pues a/d y b/d son en-
teros. Como mcd (a/d , b/d) = 1 (ver ejercicio 2.13), entonces: (a/d)|(b/d)y =
(a/d)|y. Luego, si ponemos y =(a/d)k = (a/d)x =(b/d)(a/d)k = x =(b/d)k
Teorema 2.12 Sea d = mcd(a, b). Si la ecuaci on diofantica lineal ax +by = c tiene
una soluci on x = x
0
, y = y
0
, entonces la soluci on general es x = x
0
+
b
d
k, y = y
0

a
d
k
con k Z.
Prueba: Sean x = x
0
, y = y
0
, soluci on de ax +by = c. Tenemos,
_
ax +by = c
ax
0
+by
0
= c

Ecuaci on homogenea
..
a(x x
0
) +b(y y
0
) = 0
_
_
_
x x
0
= (b/d)k x = x
0
+(b/d)k
y y
0
= (a/d)k y = y
0
(a/d)k
26 DIVISIBILIDAD
EJEMPLO 2.23 Consideremos la ecuaci on en enteros 2x+3y = 2. Como una soluci on
particular es (1, 0), entonces la soluci on general es x = 1+3k, y = 02k, k Z.
-5
2
4
6
-5
5
Figura 2.2 Soluciones enteras de la ecuaci on
2x +3y = 2.
k x y

4 11 8
3 8 6
2 5 4
1 2 2
0 1 0
1 4 2
2 7 4
3 10 6
4 13 8

2.7 Teorema fundamental de la aritmetica
Un corolario muy util del teorema (2.9) es,
Corolario 2.5 Si m es el m as peque no entero positivo que es combinacion lineal de
a y b, entonces m = mcd(a, b).
EJEMPLO 2.24 mcd(a, b) = 1 x, y Z tal que xa+by = 1
Lema 2.3 Muestre que si a|bc y mcd(a, b) = 1 entones a|c.
Prueba: existen x, y Z tal que xa+by = 1. Multiplicando por c a ambos lados,
acx +bcy = c
Como a|ac y a|bc entonces a|(acx +bcy).
Lema 2.4 (Lema de Euclides) Si p es primo y p|ab entonces p|a o p|b.
Prueba: Supongamos que p|ab pero p a. En este caso mcd(p, a) = 1 por ser p
primo (el unico factor en com un sera p o 1). De el ejemplo (2.3) concluimos p|b.
Antes de enunciar el teorema fundamental de la aritmetica, veamos un ejemplo muy
familiar
EJEMPLO 2.25 Podemos factorizar 36 como un producto de primos: en cada paso
buscamos el divisor primo m as peque no:
Introducci on a la Teora de N umeros.. Walter Mora F.
Derechos Reservados 2009 Revista digital Matem atica, Educacin e Internet (www.cidse.itcr.ac.cr/revistamate/)
EJERCICIOS 27
36 = 2 18
= 2 2 9
= 2 2 3 3
El metodo que usamos es el procedimiento usual de la escuela. Obtener la factor-
izaci on prima de un n umero n dividiendo por los primos n
36
18 2
9 2
3 3
1 3
36 = 2
2
3
2
84
42 2
21 2
7 3
1 7
84 = 2
2
3 7
Teorema 2.13 (Fundamental de la aritm etica) Todo n umero natural n > 1 se puede
factorizar de manera unica como
n = p

1
1
p

2
2
p

k
k
donde p
1
, . . . , p
n
son primos distintos y
1
, . . . ,
n
son enteros positivos. Esta factor-
izaci on se llama la factorizaci on prima de n.
Prueba: La prueba se hace por induccion completa. El resultado es cierto para
n = 2. Supongamos ahora que el resultado es cierto para n = 3, 4, ..., k. Hay que
probar que es cierto para k +1. Si k +1 es primo, listo. Si k +1 es compuesto,
entonces existen a, b Z, 1 < a b < k +1, tal que k +1 = ab. Pero, por hip otesis
de induccion a y b factorizan como producto de primos, as que k +1 tambien
factoriza como producto de primos, a saber, los factores de a y b.
Unicidad: La prueba es por contradiccion. Supongamos que n =r
1
r
2
r
u
=q
1
q
2
q
v
donde todos los r

i
s y los q

j
s son primos, r
1
r
2
... r
u
y q
1
q
2
... q
v
. Si
cancelamos los primos iguales que hay en ambos lados y nos queda
r
i
1
r
i
2
r
i
n
= q
j
1
q
j
2
q
i
m
,
con n y m positivos, entonces r
i
1
debe dividir a alg un q
j
t
, es decir, r
i
1
= q
j
t
:
Contradiccion pues los asumimos distintos.
Nota: Observe que el n umero 1 no es ni primo ni compuesto. Esto garantiza la
unicidad de la factorizaci on.
mcd y mcm
28 DIVISIBILIDAD
Denici on 2.4 Si a, b Z
+
entonces el mnimo com un m ultiplo de a y b es el m as
peque no entero m > 0 tal que a|m y b|m. Se escribe mcm(a, b) = m
Teorema 2.14 Si a, b son enteros positivos, supongamos que
a =
k

i=1
p

i
i
,
i
0
b =
k

i=1
p

i
i
,
i
0
Es decir, solo por conveniencia, igualamos el n umero de factores con 1s si es nece-
sario. Entonces,
mcd(a, b) =
k

i=1
p

i
i
,
i
= mn{
i
,
i
}
mcm(a, b) =
k

i=1
p

i
i
,
i
= m ax{
i
,
i
}
En particular mcd(a, b) mcm(a, b) = ab, es decir
mcm(a, b) =
ab
mcd(a, b)
.
Prueba: Ejercicio.
Aunque para n umeros peque nos, el metodo de la factorizaci on prima sirve para
calcular mcd(n, m) y el mcm(n, m), en general, es computacionalmente ineciente,
por el costo de obtener esta factorizaci on. Un algoritmo m as adecuado esta basado
en el teorema (2.8).
EJEMPLO 2.26 A partir de la factorizaci on prima de dos n umeros n, m podemos cal-
cular el mcd(n, m) y el mcm(n, m).
36
18 2
9 2
3 3
1 3
36 = 2
2
3
2
7
0
84
42 2
21 2
7 3
1 7
84 = 2
2
3 7
Luego, mcd(36, 84) = 2
2
3 7
0
= 12 y el mcm(36, 84) = 2
2
3
2
7 = 252.
EJEMPLO 2.27 Realizar la suma
5
36
+
7
84
+
3
4
.
Solucion: Como mcm(36, 84, 4) = 252, entonces
EJERCICIOS 29
5
36
+
7
84
+
3
4
=
7 5
252
+
3 7
252
+
63 3
252
=
=
245
252
=
35
36
Para el mnimo com un m ultiplo de una lista de n umeros tenemos un teorema similar
al teorema del m aximo com un divisor.
Teorema 2.15 mcm(a
1
, a
2
, ..., a
n
) = mcm(a
1
, mcm(a
2
, ..., a
n
)).
Prueba: Ejercicio.
En la seccion que trata sobre el teorema chino del resto vamos a necesitar los dos
corolarios que siguen y, en su momento, haremos referencia a ellos.
Corolario 2.6 Si m
1
, m
2
, ..., m
k
son primos relativos dos a dos, entonces
mcm(m
1
, m
2
, ..., m
k
) = m
1
m
2
m
k
.
Prueba: Ejercicio.
Corolario 2.7 Si m
1
, m
2
, ..., m
k
, a Z
+
y si m
i
|a, i = 1, 2, ..., k; entonces
mcm(m
1
, m
2
, ..., m
k
)|a.
Prueba: Por induccion completa: La armaci on es claramente correcta para k =
1 y k = 2. Asumamos que es correcta para 1, 2, ..., t. Ahora, supongamos que
m
i
|a, i = 1, 2, . . . , t, t +1 entonces mcd(m
1
, m
2
, . . . , m
t
)|a por la hip otesis de induccion
y m
t+1
|a, pero entonces los dos n umeros mcd(m
1
, m
2
, . . . , m
t
) y m
t+1
dividen a, as
que la hip otesis de induccion nos dice que mcd(mcd(m
1
, m
2
, . . . , m
t
), m
t+1
)|a, i.e.
mcm(m
1
, m
2
, ..., m
k
)|a por el teorema (2.15).
EJEMPLO 2.28 El entero 290290 es divisible por 10, 77, y 13. Como mcd(10, 77) =
1, mcd(10, 13) = 1 y mcd(77, 13) = 1; entonces mcm(10, 77, 13) = 10 77 13 =
10010 y 10010| 290290.
30 DIVISIBILIDAD
EJEMPLO 2.29 Muestre que si p = 4k +1 y p = 3k

+1 entonces hay un k

Z tal
que p = 12k

+1
Solucion: Como mcd(4, 3) = 1, 4|p 1 y 3|p 1, entonces mcm(3, 4) = 12|p 1,
es decir, hay un k

Z tal que p1 = 12k

.
EJERCICIOS
2.7 Muestre que los divisores de n ocurren en pares, es decir, si d|n entonces
n/d|n.
2.8 Muestre que si d = mcd(a, b) entonces mcd(a/d, b/d) = 1 (Use la identidad
de Bezout).
Solucion:
a
d
,
b
d
Z. Por Bezout, d = ax +by = 1 =
a
d
x +
b
d
. Esta es la mnima
combinacion lineal positiva de (a/d b/d), por lo tanto mcd(a/d, b/d) = 1.
2.9 Muestre que mcd(ab, m)| mcd(a, m)mcd(b, m) (Use Id. Bezout).
Solucion: Sean d = mcd(ab, m), d
1
= mcd(a, m), d
2
= mcd(b, m). Por Bezout,
_
_
_
ax
1
+my
1
= d
1
= abx +my = d
1
d
2
= d|d
1
d
2
bx
2
+my
2
= d
2
2.10 Muestre que mcd(a, b) = 1 entonces mcd(a, m)mcd(b, m) = mcd(ab, m)
Solucion: Sean d = mcd(ab, m), d
1
= mcd(a, m), d
2
= mcd(b, m).
Por Bezout, ax +by = 1, luego axm+bym = m. Como d
1
es m ultiplo de a y d
2
es
m ultiplo de m, se sigue axm = k
1
d
1
d
2
. De manera analoga, bym = k
2
d
1
d
2
.
As, d
1
d
2
|m d
1
d
2
|ab y entonces d
1
d
2
|d. Usando el ejercicio anterior se concluye
que d
1
d
2
= d.
2.11 Muestre que si mcd(a, b) = 1 y si mcd(a, c) = 1, entonces mcd(a, bc) = 1
Solucion: Por Bezout, existen x, y, s, t Z tal que
_
ax +by = 1
as +ct = 1
, Multiplicando
obtenemos a(axs +xct +sby) +bc(yt) = 1, es decir, mcd(a, bc) = 1.
2.12 Muestre que si mcd(a
1
, m) =1, mcd(a
2
, m) =1, . . . , mcd(a
k
, m) =1 entonces
mcd(a
1
a
2
a
k
, m) = 1.
2.13 Muestre que si mcd(a, b) =d y si a =k
1
d y b =k
2
d, entonces mcd(k
1
, k
2
) =
1
Solucion: Por Bezout ax +by = d = k
1
x +k
2
y = 1, por (2.1, 4) mcd(k
1
, k
2
) = 1
2.14 Muestre que si d = mcd(a, b) y si ra+sb = d, entonces mcd(r, s) = 1.
Solucion: ra+sb = d = rk
1
d +sk
2
d = d = rk
1
+sk
2
= 1 = mcd(r, s) = 1 por
(2.1, 4).
EJERCICIOS 31
2.15 Muestre que si am+bn = h entonces mcd(a, b)|h
Solucion: Sea d = mcd(a, b), a y b son m ultiplos de d, entonces am+bn = h =
k
1
dm+k
2
dn = h = d|h.
2.16 Muestre que la ecuaci on diofantica ax+by =h tiene soluci on solo si mcd(a, b)|h
Solucion: : es el ejercicio anterior.
: Sea d = mcd(a, b) y sea h = kd. Usando el algoritmo extendido de Euclides
podemos calcular x
1
, y
1
Z tal que ax
1
+by
1
= d = ax
1
k +by
1
k = kd = h. Luego,
la soluci on de la ecuaci on diofantica es x = x
1
k y y = y
1
k.
2.17 Resuelva la ecuaci on diofantica 24 = 365x +1876y
Solucion: Por el algoritmo extendido de Euclides, 1 = 365 699+1876 136 luego
24 = 365 16776+1876 3264
2.18 Sea p un n umero primo. Determinar todos los enterosk Z tales que
_
k
2
kp
es natural. Ayuda: Si p
2
= ab = (a = p b = p) (a = p
2
b = 1)
Solucion: Sea
_
k
2
kp = d N. Luego k
2
kpd
2
= 0 de donde
k =
p
_
p
2
+4 1 d
2
2
()
k es entero, as que
_
p
2
+4d
2
debe ser cuadrado perfecto, sea p
2
+4d
2
= a
2
, en-
tonces
p = (a2d)(a+2d)
como p es primo, solo tenemos las dos posibilidades siguientes,
1. p = (a2d) y p = (a+2d)
2. p
2
= a+2d y a2d = 1 pues a+2d a2d.
En el primer caso d = 0 (y a = p). Entonces k = 0 o k = p
En el segundo caso, resolvemos el sistema y obtenemos d = (p
2
1)/2 (y a =
(p
2
+1)/2). Como a, d son naturales, este caso se cumple si p es impar, es decir
p = 2. Sustituyendo d en () y resolviendo queda k = (p+1)/2 y k =(p1)/2.
Note que si p = 2 solo puede suceder el primer caso y queda k = 0 o k = 2.
2.19 Sean q
1
, . . . , q
n
y p
i
todos n umeros primos distintos. Use induccion matematica
para probar que si p
i
|q
1
q
2
q
n
entonces p
i
= q
j
para alg un j 1, 2, ..., n.
Solucion: Para n = 2 es cierto, por el lema de Euclides.
Si es cierto para n =k y p
i
|(q
1
q
2
q
k
) q
k+1
, por el lema de Euclides, p
i
|(q
1
q
2
q
k
)
o p
i
|q
k+1
. Aplicando la hip otesis de induccion en el primer caso p
i
|q
j
para alg un
j {1, 2, ..., k}, sino p
i
|q
k+1
.
2.20 Sea p primo, si p|a
n
= p|a
2.21 Muestre que si p es primo, entonces
n

p no es racional. Ayuda: Por con-


tradiccion, suponga
n

p = a/b con mcd(a, b) = 1.


2.22 Sean mcd(a, b) = 1 y p primo, entonces p mcd(a
n
, b
n
).
32 DIVISIBILIDAD
2.23 Muestre que si mcd(a, p) = 1 con p primo, entonces mcd(a, p
s
) = 1, con
s > 0.
2.24 Sean m y n son primos relativos. Muestre que si mn = a
k
, k 0; entonces
existe x, y Z tal que m = x
k
y n = y
k
. Ayuda: Use la descomposici on en factores
primos de cada uno de los n umeros.
Solucion: Sean a =
i
p

i
i
, m=
j
q

j
j
y n =
s
r

s
s
la descomposici on prima de estos
n umeros. Luego, como mn y a
k
son iguales, su descomposici on prima es la misma
excepto por el orden de los factores, i.e.

j
q

j
j

s
r

s
s
=
i
p
k
i
i
Entonces para cada j, q

j
j
= p
k
i
j
i
j
y para cada s, r

s
s
= p
k
is
i
s
. Luego, m=
t
p
k
t
t
=x
k
y n =
d
p
k
d
d
= y
k
.
2.25 Consideremos la descomposici on prima n =
k
i
p

i
i
. mcm(p

1
1
, . . . , p

k
k
) =n?
2.26 Supongamos que los enteros m y n son primos relativos. Muestre que si
d|mn, entonces b, c unicos tal que d = bc con b|m y c|n.
2.27 Si 4|p3 y 3|p1, muestre que 12|p+1.
Solucion: Hay un k Z tal que p 3 = 4k, entonces p 7 = 4(k 1), por tanto
4|p 7. Usando la misma idea, comprobamos que 3|p 7. Como mcd(3, 4) =
1, mcm(3, 4) = 12 y entonces 12|p7.
2.28 Sea n > 1 y p el m as peque no divisor primo de n. Muestre que mcd(n, p
1) = 1
2.29 Encuentre tres n umeros a, b, c tal que mcd(a, b, c) =1 pero que mcd(a, b) =
1, mcd(a, c) = 1, mcd(b, c) = 1.
Solucion: Si p
1
, p
2
, p
3
son primos, mcd(p
1
p
2
, p
2
p
3
, p
1
, p
3
) =1 y mcd(p
i
p
j
, p
j
p
k
) =
1. Tambien, si a = 2 3 5, b = 5 7 11 y c = 11 2, mcd(a, b, c) = 1
mcd(a, b) = 5
mcd(a, c) = 2
mcd(b, c) = 11
2.30 Probar que dos enteros consecutivos son primos relativos
Solucion: Sea d = mcd(a, a+1). Como d|a d|(a+1) = d|1. Luego, d = 1.
Otra manera: d = mcd(a1, a) = d a(a1) = 1 pues d es la mnima com-
binacion lineal positiva de a y a1.
2.31 Probar que si mcd(a, b) = mcm(a, b) =a = b.
Solucion: m = mcd(a, b) = mcm(a, b) = m|a m|b a|m b|m = a = b (por
ser ambos positivos).
2.32 Muestre que mcd(mg, g) = g si g N.
Solucion: Sea d = mcd(mg, g), entonces d|g = d g. Pero g|g g|gm, entonces
EJERCICIOS 33
g = d.
2.33 Si a, b N, y si a|b calcule mcd(a, b) y mcm(a, b).
Solucion: mcd(a, b) = mcd(a, ka) =a seg un el ejercicio anterior. mcm(a, b) =
ab
a
=
b.
2.34 Muestre que (x, y Z tal que x+y = s mcd(x, y) = g) g|s, g Z
+
Ayuda: Una implicaci on es directa por Bezout, la otra requiere descomponer kg =
(k 1)g+g
Solucion: : g|x y g|y = g|s.
: g|s = s =kg =(k1)g+g. As, si x =(k1)g y y =g entonces
s = x +y y mcd(x, y) = mcd((k 1)g, g) = g, por ser g positivo.
2.35 Mostrar que si mcd(a, b) = mcd(c, d) = 1 y si
a
b
+
c
d
Z entonces |b| =|d|.
Ayuda: Si p|qc y mcd(p, q) = 1, entonces p|c.
Solucion: Si
a
b
+
c
d
=
ad +bc
bd
Z, entonces bd|(ad+bc). Como ad+bc =bd se tiene
que b|(ad +bc) y d|(ad +bc). Luego, b|ad d|bc. Finalmente, como mcd(a, b) =
mcd(c, d) = 1 se concluye que b|d d|b, es decir |a| =|b|.
2.36 Mostrar que mcd(a, b) = mcd(a, b, ax +by) con x, y Z
Solucion: Sea d = mcd(a, b) y m = mcd(a, b, ax + by). Como d|a d|b =
d|(ax +by) y por tanto d|m.
Luego, como m|a m|b = m|d.
d = m, por ser ambos positivos.
2.37 Muestre que mcd(a, a+2) = 1 o 2
Solucion: d = mcd(a, a+2) = d|a d|(a+2) = d|2 = d = 1 d = 2.
2.38 Muestre que si n = pq, con p, q factores no triviales de n y p|(y x) y
n (y x), entonces 1 < mcd(y x, n) < n.
2.39 Sea p
i
el iesimo primo y sea N = p
1
p
2
p
n1
+1. Muestre que N p
n
.
2.40 Sean m, a, b Z. Muestre que mcd(ma, mb) =|m| mcd(a, b)
Solucion: Sea d
m
= mcd(ma, mb) y d = mcd(a, b). Por Bezout, existen x, y, s, t Z
tal que
d
m
= amx +bmy = m(ax +by) = m(kd) pues d|(ax +by). Luego, md|d
m
.
d = as +bt = md = (ma)s +(mb)t = d
m
|md.
34 DIVISIBILIDAD
md|d
m
d
m
|md = |d
m
| =|md| = d
m
=|m|d, por ser d
m
y d positivos.
2.41 Sea mcd(a, b) = 1. Muestre que si d = mcd(a +b, a b) entonces d = 1 o
d = 2.
Sea mcd(a, b) =1 y d = mcd(a+2b, 2a+b). Muestre que d|3a y d|3b y por tanto,
d = 1 o d = 3.
Solucion:
d|(2(a+2b) (2a+b)), i.e. d|3b.
d|(2(2a+b) (a+2b)), i.e. d|3a
Luego d| mcd(3a, 3b) = d|3mcd(a, b) = d|3 por ejercicio(2.40). Luego, d = 1
o d = 3.
2.42 Muestre que para todo n N, n > 1; A = 1+
1
2
+
1
3
+... +
1
n
no es entero
Solucion: Asuma que A es entero. Sea 2

la m as grande potencia de 2 que es n,


i.e. 2

n pero 2

2 > n.
Considere todas las m aximas potencias p

i
i
, de los primos impares p
i
, que no exce-
den n, es decir, p

i
i
n pero p

i
+1
i
>n. Sea P es producto de todas estas potencias,
P =

i
p

i
i
.
Consideremos el producto
2
1
P
_
1+
1
2
+
1
3
+... +
1
n
_
= 2
1
P+
2
1
P
2
+
2
1
P
3
+... +
2
1
P
2

+... +
2
1
P
n
.
Analizamos ahora cada fracci on
2
1
P
k
. Si k tiene factorizaci on prima k =2

i
q

i
i
,
2
1
P
k
=
_

_
2
1
P
2

i
q

i
i
2
1
P
2

Por denicion de P, los q


i
aparecen en P pero con una potencia igual o mayor,
es decir, para cada i hay un j tal que q
i
= p
j
y
i

j
. Luego, como 1,
entonces
2
1
P
2

i
q

i
i
es entero.
Pero, por otra parte, el caso 2
1
P/2

= P/2 = m+1/2 con m entero, por ser P


impar. Resumiendo,
EJERCICIOS 35
2
1
PA = 2
1
P
_
1+
1
2
+
1
3
+... +
1
n
_
= 2
1
P+
2
1
P
2
+
2
1
P
3
+... +
2
1
P
2

+...
2
1
P
n
= Q+P/2 = Q

+1/2, con Q, Q

enteros.
Pero si A es entero, 2
1
PA es entero, mientras que Q

+1/2 no ().
2.43 Mostrar que si d|(n
2
+1) y d|((n +1)
2
+1) para alg un entero n, entonces
d = 1 d = 5.
2.44 Probar que la fracci on(21n+4)/(14n+3) es irreducible para cualquier n Z
2.45 Sea N = 2
p
1,
a) Probar que 2
ab
1 =(2
a
1)(1+2
a
+2
2a
+2
3a
+ +2
(b1)a
). Ayuda: Usar
la identidad 1+x +x
2
+x
3
+ +x
k
=
1x
k+1
1x
, x = 1, k N.
b) Muestre que si N es primo, entonces p es primo.
2.46 Considere los n umeros de Euler: T
n
= 2
2
n
+1 con n N.
a) Muestre que 2
2
n
1 = T
2
n1
2T
n1
Solucion: La vericaci on es directa:
T
2
n1
2T
n1
= (2
2
n1
+1)
2
2(2
2
n1
+1)
= 2
2
n1
2
+2 2
2
n1
+12 2
2
n1
2
= 2
2
n
1
b) Muestre, usando a), que T
n
2 = T
n1
T
n2
T
0
Solucion: La formula anterior es una formula recursiva:
2
2
n
1 = T
n
2 = T
2
n1
2T
n1
= T
n1
(T
n1
2), i.e.
T
n
2 = T
n1
(T
n1
2). Luego,
T
n
2 = T
n1
(T
n1
2)
= T
n1
(T
n2
(T
n2
2))
= T
n1
T
n2
T
n3
(T
n3
2)
.
.
.
= T
n1
T
n2
T
n3
T
0
(T
0
2)
= T
n1
T
n2
T
n3
T
0
, pues T
0
2 = 32 = 1
36 DIVISIBILIDAD
c) Muestre que si m > n, mcd(T
n
, T
m
) = 1
Solucion: T
m
= T
m1
T
n
T
0
+2. Si mcd(T
n
, T
m
) = d entonces d|T
m
=
d|(T
m1
T
n
T
0
+2) y como d|T
n
, d|2. As que d = 1 o d = 2. Pero por
denicion, T
m
y T
n
son impares: solo
2.47 Sea n entero positivo y S un conjunto con n+1 elementos distintos tomados
del conjunto {1, 2, ..., 2n}. Muestre que hay al menos dos elementos en S primos
relativos
2.48 n 2. Supongamos que tomamos n +1 enteros al azar. Muestre que hay
dos elementos tal que su diferencia es divisible por n. Ayuda: usar le principio del
palomar y el algoritmo de la divisi on: n +1 enteros producen n +1 restos, pero
dividir por n solo produce n restos...
2.49 Mostrar que hay un n umero innito de primos de la forma 4n+3.
Ayuda: Asuma que solo hay k primos de esa forma y considere el n umero N =
4p
1
p
2
p
k
+3.
3 CONGRUENCIAS
3.1 Congruencias modulo m
Denici on 3.1 Sea m Z, m1. Decimos que a es congruente con b m odulo m si
m|(ba). Escribimos
a b (mod m)
EJEMPLO 3.1 10 1 (mod 9) pues 9|(101)
10 1 (mod 3)
10 1 (mod 11)
Observe que si m = 1, a b (mod 1) para todo a, b Z, i.e. este m odulo no tiene
gracia.
Teorema 3.1 a b (mod m)
_
_
_
(i) ba 0 (mod m)
(ii) a = mk +b, para alg un k Z
(iii) a mod m = b mod m
Prueba: Probemos (iii) usando el algoritmo de la divisi on.
: Si a = mk +r
1
, 0 r
1
< m y b = mk

+r
2
, 0 r
2
< m, supongamos que r
2
r
1
,
entonces ba = m(k

k) +r
2
r
1
con 0 r
2
r
1
< m. Pero ba = mk

, como el
resto es unico, r
2
= r
1
. El caso r
1
r
2
es identico.
: Ejercicio.
EJEMPLO 3.2 Si x 1 (mod 8), entonces xmod8 = 1 y x = 8 k +1.
El smbolo se puede manipular como = excepto para la cancelacion:
Teorema 3.2 Sean a b (mod m) a

(mod m) y c, k Z.
(1) ka kb (mod m), en particular a
k
b
k
(mod m).
Introducci on a la Teora de N umeros.. Walter Mora F.
Derechos Reservados 2009 Revista digital Matem atica, Educacin e Internet (www.cidse.itcr.ac.cr/revistamate/)
37
38 CONGRUENCIAS
(2) aa

bb

(mod m)
(3) aa

bb

(mod m)
(4) a a (mod m) para toda a Z
(5) a b (mod m) = b a (mod m)
(6) a b (mod m) b c (mod m) = a c (mod m)
(7) Sea a = 0, ab ac (mod m) mcd(a, m) = d = b c (mod
m
d
)
(8) Sea a = 0, ab ac (mod m) mcd(a, m) = 1 = b c (mod m)
(9) Si a b (mod m) y d|m, entonces a b (mod d)
Prueba: Solo vamos a probar algunos items, el resto queda como ejercicio.
(2) aa

bb

(mod m) : En efecto, sean k, k

Z tal que b =mk+a y b

=mk

+a

,
multiplicando obtenemos bb

= m(mkk

+ka

+k

a) +aa

= m|(bb

aa

).
(7) ab ac (mod m) mcd(a, m) = d = b c (mod
m
d
): En efecto, recordemos
primero que si d = mcd(a, m) = mcd
_
a
d
,
m
d
_
= 1.
Entonces, como hay un k Z tal que ac ab = mk = a(c b) = mk =
a
d
(c b) =
m
d
k. As,
m
d

a
d
(c b) mcd
_
a
d
,
m
d
_
= 1 =
m
d
|(c b)
(8) Es un caso especial de 7.
EJEMPLO 3.3 Manipulacion algebraica m odulo m,
1. Muestre que si a b (mod m) y si c a + d (mod m) entonces c b +
d (mod m).
Solucion: Como 0 ba (mod m) y c a+d (mod m), sumamos miembro a
miembro y c b+d (mod m)
2. Muestre que si a b (mod m) y si c ad (mod m) entonces c bd (mod m)
Solucion: Existe k, k

Z tal que a =mk+b y cad =mk

, entonces cad =
c (mk +b)d = mk

= c bd = m(k

+kd) = c bd (mod m).


3. d

10
0
..
8+2+

10
0
..
3+2+5+

10
7
..
5+2 (mod 10) = d 7 (mod 10)
4. Calcular 9
5
mod 5
39
Solucion: 9
5
4
5
(mod 5) pues 9 4 (mod 5); ahora hacemos una reducci on
a potencias m as peque nas: 4
5
4
4+1
(4
2
)
2
4 1
2
4 4 (mod 5). Por tanto,
9
5
mod 5 = 4.
EJEMPLO 3.4 Calcular el resto de dividir 15
196
por 13.
Solucion: La idea es descomponer 15
196
en potencias m as peque nas. Si r es el resto
buscado, 15
196
r (mod 13).
15
196
2
196
mod 13, pues 15 2 (mod 13),

_
2
22
_
77
mod 13, pues 196 = 2 2 7 7,

_
3
7
_
7
mod 13, pues 2
4
= 16 3 (mod 13),

_
_
3
3
_
2
3
_
7
mod 13, pues
_
3
3
_
2
3 = 3
7
,

_
1
2
3
_
7
mod 13, pues 3
3
= 27 1 (mod 13),
3
7
mod 13
3 mod 13 pues 3
7
3 (mod 13).
As, el resto de dividir 15
196
por 13 es 3.
EJEMPLO 3.5 Resolver 4x 8 (mod 12) con x {0, 1, 2, ..., 11}.
Solucion: Podramos resolver esta congruencia por ensayo y error, pero la vamos a
resolver usando la propiedad 7 del teorema ( 3.2).
4x 8 mod 12,
x 2 mod 3 por el teorema 3.2, 7;
Luego, los x {0, 1, 2, ..., 12} que dejan resto 2 al dividir por 3 son x = 2, 5, 8, 11.
EJEMPLO 3.6 Calcular el resto de la divisi on de 12
201
por 13, es decir, calcular
12
201
mod 13
Como 12 1 (mod 13) = 12
201
(1)
201
(mod 13). Entonces, por transitividad
12
201
12 (mod 13). Esto dice que 12
201
mod 13 = 12 mod 13 = 12
EJEMPLO 3.7 Calcular 13
300
mod7.
Aunque 13 5 (mod 7) es mejor iniciar con 13 2 (mod 7) pues de esta congru-
encia obtenemos 13
3
8 (mod 7) y 8 1 (mod 7). As, 13
300
1 (mod 7) =
13
300
mod7 = 1 mod 7 = 1.
40 CONGRUENCIAS
3.2 Calendarios: Que d

ia nacio Ud?.
Supongamos que queremos saber el da de la semana correspondiente a una fecha
dada: Por ejemplo, que da fue el 9 de mayo de 1973?
En varios libros se hace un analisis de como resolver este problema, por ejemplo en
[?]; aqu solo vamos a dar la soluci on, seg un el calendario Gregoriano.
Primero debemos numerar los das y los meses, como se muestra en en la tabla que
sigue (a febrero se le asigna el 12; febrero es especial por ser el mes al que se agrega
un da en a no bisiesto).
Domingo = 0 Marzo = 1 Octubre = 8
Lunes = 1 Abril = 2 Noviembre = 9
Martes = 2 Mayo = 3 Diciembre = 10
Miercoles = 3 Junio = 4 Enero = 11
Jueves = 4 Julio = 5 Febrero = 12
Viernes = 5 Agosto = 6
Sabado = 6 Setiembre = 7
Tabla 3.1
Ahora, sea f = fecha, m= mes, a = a no, s = siglo y n = a nos en el siglo. Por ejem-
plo, si tenemos la fecha: abril 1, 1673 entonces: f = 1, m = 2, a = 1673 = 100s +n
donde s = 16 y n = 73.
Finalmente, si d denota el da de la semana correspondiente a la fecha ( f , m, a),
entonces
d f +
_
13m1
5
_
2s +n+
_
s
4
_
+
_
n
4
_
(mod 7)
Ahora ya puede calcular que da naci o Ud.
EJEMPLO 3.8 El 9 de mayo de 1973 fue miercoles=3; pues f =9, m =3, A = 1973 =
100s +n con s = 19 y n = 73. Usando la formula obtenemos,
d 9+738+73+4+18 3 (mod 7)
41
3.3 Trucos de divisibilidad.
Si a = a
n
10
n
+a
n1
10
n1
+... +a
1
10+a
0
, la suma de sus dgitos es congruente con
a m odulo 9, en efecto, como 10 1 (mod 9) entonces 10
i
1 (mod 9), i = 0, 1, 2, ...
Luego, multiplicando por a
i
y sumando
n

i=0
a
i
10
i

i=0
a
i
(mod 9) = a
n

i=0
a
i
(mod 9)
n

i=0
a
i
10
i

i=0
a
i
(mod 3) = a
n

i=0
a
i
(mod 3) pues 3|9
1. Divisibilidad por 9 : 9 divide a a si y s olo si 9 divide la suma de sus dgitos,
es decir, 9|a 9|
n
i=0
a
i
En efecto, como a
n

i=0
a
i
(mod 9) entonces 9|(
n
i=0
a
i
a) . Si 9|a entonces
divide la suma de sus dgitos y si 9 divide la suma de los dgitos de a entonces
divide a a.
2. Divisibilidad por 3 : 3 divide a a si y s olo si 3 divide la suma de sus dgitos.
La demostracion es igual a la anterior, cambiando 9 poe 3.
3. Divisibilidad por 2 y por 5 : tanto 2 como 5 dividen a a si y s olo si dividen
a
0
.
En efecto: Observemos que a =
n

i=1
a
i
10
i
+ a
0
. Tanto 2 como 5 dividen a
a
i
10
i
, por tanto, dividen a la suma
n

i=1
a
i
10
i
+ a
0
si y solo si tanto 2 y 5
dividen a a
0
4. Divisibilidad por 11 : 11 divide a a si y s olo si 11 divide la suma alternada
de sus dgitos, es decir, 11

n
i=0
(1)
i
a
i
En efecto, esto es consecuencia de que 10 1 (mod 11).
3.4 Cuadrados Magicos
Introducci on a la Teora de N umeros.. Walter Mora F.
Derechos Reservados 2009 Revista digital Matem atica, Educacin e Internet (www.cidse.itcr.ac.cr/revistamate/)
42 CONGRUENCIAS
Un cuadrado m agico es un arreglo de n n n umeros en el que la suma de las
entradas de cada la o columna siempre es la misma. Por ejemplo, consideremos el
cuadrado m agico 33
_
_
4 2 6
0 7 5
8 3 1
_
_
, tanto las las como las columnas suman 12
En la actualidad hay varios metodos para
cons-truir cuadrados m agicos. En 1693 De la
Loub`ere dio un metodo para construir cuadra-
dos m agicos para cualquier n impar, el metodo
es llamado el Metodo Siames. En 1929 D.N.
Lehmer investigo, por medio de congruencias,
una generali-zaci on de este metodo. El resul-
tado es una manera sencilla de colocar los
n umeros 0, 1, ..., n
2
1 en un arreglo nn de
tal manera que sea un cuadrado m agico. Este
metodo, llamado metodo del paso uniforme,
calcula la entrada (i, j), usando congruencias,
en la que se debe colocar cada uno de los
n umeros k = 0, 1, ..., n
2
1 para que el arreglo
resulte magico.
Denici on 3.2 Supongamos que n
2
enteros diferentes son colocados en un arreglo
n n. Si la suma de las entradas de cada la suma siempre lo mismo, decimos
que el cuadrado es magico por las. Si la suma de las entradas de cada columna
suma siempre lo mismo, decimos que el cuadrado es magico por columnas. Si el
cuadrado es ambos magico por las y magico por columnas, se dice cuadrado
m agico y la suma se dice suma m agica.
Teorema 3.3 Sea n entero positivo impar y a, b, c, d, e, f enteros, tal que mcd(c f
de, n) = 1 Sea A = (a
i j
) la matriz nn denida as: Para cada k = 0, 1, ..., n
2
1,
a
i+1, j+1
= k si i a+c k +e k/n[ (mod n) y j b+d k + f k/n[ (mod n)
Entonces,
Si mcd(c, n) = mcd(e, n) = 1, el cuadrado es magico por columnas.
Si mcd(d, n) = mcd( f , n) = 1, el cuadrado es magico por las.
Si mcd(c, n) = mcd(d, n) = mcd(e, n) = mcd( f , n) = 1, el cuadrado es m agico. En
cada caso la suma m agica es n(n
2
l)/2.
EJEMPLO 3.9 Sea n =7 y a =4, b =3, c =1, d =2, e =1, f =4. Como mcd(c, n) =
mcd(d, n) = mcd(e, n) = mcd( f , n) = 1, el metodo construye un cuadrado m agico
77 con suma m agica 168.
43
k = 0 : i 4+1 0+1 0/7[ (mod 7) y j 32 04 0/7[ (mod 7) a
5,4
= 0
k = 1 : i 4+1 1+1 1/7[ (mod 7) y j 32 14 1/7[ (mod 7) a
6,2
= 1
k = 2 : i 4+1 2+1 2/7[ (mod 7) y j 32 24 2/7[ (mod 7) a
7,7
= 2

_

_
15 40 9 34 3 21 46
10 28 4 22 47 16 41
5 23 48 17 35 11 29
42 18 36 12 30 6 24
37 13 31 0 25 43 19
32 1 26 44 20 38 7
27 45 14 39 8 33 2
_

_
La implementaci on, usando MATHEMATICA, es muy sencilla,
a = 4; b = 3; c = 1; d = -2; e = 1; f = -4; n = 7;
(*Vericar si son primos relativos?*)
GCD[c*f - d*e, n], GCD[c, n], GCD[d, n], GCD[e, n], GCD[f, n]
B = Array[A, n, n]; (*A[i,j]=k*)
Do[A[Mod[a + c*k + e*Floor[k/n], n] + 1,
Mod[b + d*k + f*IntegerPart[k/n], n] + 1] = k, k, 0, n2 - 1]
MatrixForm[B]
3.5 Clases residuales modulo m
La relacion congruente m odulo m, denotada por brevedad con
m
, se dene
as:
a b (mod m) m|(ba)
La relacion
m
es una relacion de equivalencia, es decir, particiona Z en clases (de
equivalencia.) El conjunto cociente Z/
m
, es el conjunto de clases de equivalencia.
Si a Z y a = mk +r con 0 r < m, entonces a r (mod m). Entonces es natural
tomar como representante de clase los residuos positivos m as peque nos, es decir
a = a mod m (recordemos que a mod m denota el m as peque no residuo positivo
de dividir a por m).
a = mk +b a = b = a mod m
EJEMPLO 3.10 12 2 (mod 7) k Z tal que 12 = 7k 2 12 = 2 =
12 mod 7 = 5
Introducci on a la Teora de N umeros.. Walter Mora F.
Derechos Reservados 2009 Revista digital Matem atica, Educacin e Internet (www.cidse.itcr.ac.cr/revistamate/)
44 CONGRUENCIAS
EJEMPLO 3.11 La relacion
5
particiona Z en 5 clases pues al dividir por 5 solo
hay posibilidad de cinco residuos: 0, 1, 2, 3 o 4.
El conjunto cociente es Z/
5
={0, 1, 2, 3, 4}.
0 ={5k : k Z} ={0, 5, 10, ...}
1 ={5k+1 : k Z} ={..., 9, 4, 1, 6, 11, ...}
2 ={5k+2 : k Z} ={..., 8, 3, 2, 7, 12, ...}
3 ={5k+3 : k Z} ={..., 7, 2, 3, 8, 13, ...}
4 ={5k+4 : k Z} ={..., 6, 1, 4, 9, 14, ...}
-5
0
5
10
-10
. . .
1
6
11
-4
-9
. . .
-8
-3
2
7. . .
3
8. . .
-2
-7
-12
4
9
-1...
-6
-11
=
4
0
1
2
3
Figura 3.1
En este contexto, a las clases de equivalencia se les denomina clases residuales
m odulo m y el conjunto cociente se denota con Z
m
(o tambien Z/mZ) en vez de
Z/
m
. Por ejemplo
Z
5
={0, 1, 2, 3, 4}
Por abuso del lenguaje, es usual poner
Z
5
={0, 1, 2, 3, 4}
EJEMPLO 3.12 Muestre que si p > 3 es primo, p es de la forma 6k 1.
Solucion: p debe estar en alguna de las clases de Z
6
. No esta en 0, 2 ni 4 pues estas
clases solo contienen pares.No esta en 3 pues esta clase solo contiene m ultiplos de
3. As que p 1 o p 5 = 1. Es decir, p es de la forma 6k 1. Observe que
2 2 y 3 3.
Conjunto de representantes de Z
m
.
En las aplicaciones a veces se usan otros conjuntos de representantes para las clases.
45
Si m es impar, la representaci on simetrica de Z
m
es
{
m1
2
, . . . , 1, 0, 1, . . .
m1
2
}
Si p es primo, existe b Z tal que Z
m
={0, b, b
2
, ..., b
p1
}
EJEMPLO 3.13
Z
5
= {0, 1, 2, 3, 4}
= {2, 1, 0, 1, 2} pues 2 3 (mod 5) y 1 4 (mod 5),
= {0, 3, 3
2
, 3
3
, 3
4
} pues 3
2
4 (mod 5), 3
3
2 (mod 5), 3
4
1 (mod 5)
Suma y producto en Z
m
.
Ahora nos interesa ver Z
m
desde el punto de vista de su estructura algebraica. Esto
no solo nos permite usar un lenguaje com un, sino que tambien nos permite usar
resultados generales de la teora de grupos, por ejemplo.
Podemos denir operaciones de suma y producto en Z
m
de la siguiente manera:
a+b = (a+b) mod m i.e. a+b es el resto de dividir a+b por m
a b = (a b) mod m i.e. a b es el resto de dividir a b por m
EJEMPLO 3.14 En Z
7
,
5+6 = 11 mod 7 = 4
5 30 = 150 mod 7 = 3
Propiedades de la suma y producto en Z
m
.
Con estas operaciones, si m2, Z
m
es anillo conmutativo con identidad. Si a, b, c
Z
m
,
(1) a+b Z,
(2) a+b = b+a,
(3) a+(b+c) = (a+b) +c,
(4) a+0 = 0+a = a,
(5) el inverso aditivo de a es
a
(1) a b Z,
(2) a (b+c) =a b+a c y (b+c) a =b a+c a,
(3) a 1 = a.
46 CONGRUENCIAS
Inversos m odulo m (unidades) y divisores de cero.
Sea a Z
m
, a es una unidad si tiene inverso, es decir, si existe b Z
m
tal que
ba ab 1 (mod m). En este caso ponemos a
1
=b. Por ejemplo, 2 3 1 (mod 5),
entonces el inverso de 2, m odulo 5, es 3 y viceversa.
Por otra parte, a = 0, es divisor de cero en Z
m
si existe b Z
m
, b = 0, tal que
ab = 0. Por ejemplo, 2 3 0 (mod 6), entonces 2 y 3 son divisores de cero en Z
6
.
Teorema 3.4 En Z
m
,
(i) a es una unidad si y solo si mcd(a, m) = 1;
(ii) a es divisor de cero si y solo si 1 < mcd(a, m) < m;
Prueba: (i) ab 1 (mod m) si y solo si existe k Z tal que ab+mk = 1, es decir,
si y solo si mcd(a, m) = 1.
(ii) Sea a > 1 y d = mcd(a, m).
: Como a es divisor de cero, sea b = 0, tal que ab = 0. Supongamos, por
contradiccion, que d = 1 o d = m. Si d = 1 m|ab = m|b, pero esto no puede ser
pues b 0 mod (m). Si d =m = m|a pero esto no puede ser pues a 0 mod (m).
: Como 1 < d < m y d|m, existe k tal que dk = m y 1 < k < m. Entonces k = 0
y dk = 0. Por tanto, si a = dk

, ak = dkk

= 0, es decir, a es divisor de cero.


EJEMPLO 3.15 Sea m=9. Si construimos una tabla de multiplicar para Z
9
podemos
detectar las unidades y los divisores de cero (si hubiera). La otra manera es usar el
teorema (3.4)
(Z
9
, ) 1 2 3 4 5 6 7 8
1 1 2 3 4 5 6 7 8
2 2 4 6 8 1 3 5 7
3 3 6 0 3 6 0 3 6
4 4 8 3 7 2 6 1 5
5 5 1 6 2 7 3 8 4
6 6 3 0 6 3 0 6 3
7 7 5 3 1 8 6 4 2
8 8 7 6 5 4 3 2 1
Figura 3.2 Tabla de multiplicar para Z
9
a 1 2 3 4 5 6 7 8
mcd(a, 9) 1 1 3 1 1 3 1 1
Figura 3.3 Aplicando el teorema (3.4)
47
As, las unidades de Z
8
son 1, 2, 4, 5, 7, 8 y los divisores de cero son 3, 6.
Si p es primo, entonces mcd(i, p) = 1 para todo i = 1, 2, ..., p 1. As, en Z
p
todo elemento tiene inverso y no hay divisores de cero. Z
m
es un campo si y solo
si m es primo. En el mundillo del algebra, si p es primo, se usa a Z
p
como el
representante de los campos nitos con p elementos y se le denota F
p
.
Sistemas de residuos.
Como ya dijimos, hay distintos conjuntos de representantes de Z
m
. Vamos a establecer
un par de lemas que seran de mucha utilidad al hora de establecer los teoremas
clasicos en teora de n umeros.
C omo determinar si Z
m
={a
1
, a
2
, ..., a
m
}?.
Sea |A| la cardinalidad del conjunto A. Si A y B son conjuntos nitos, A B y
|A| = |B|, entonces A = B. Este hecho lo podemos aplicar de la siguiente manera:
Como
A ={a
1
, a
2
, . . . , a
m
} ={a
1
mod m, a
2
mod m, . . . , a
m
mod m} {0, 1, . . . , m1},
entonces Z
m
= A si |A| = m. Para probar que |A| = m, necesitamos probar que A
tiene m elementos distintos m odulo m, es decir, a
i
a
j
(mod m) si i = j, 1 i, j m.
Lema 3.1 Sea mcd(a, m) = 1, entonces Z
m
={0, a, a 2, . . . , a (m1)}
Prueba: Es claro que A={0, a, a 2, . . . , a (m1)} {0, 1, . . . , m1}. Solo hay que
probar que A tiene m elementos, es decir, los elementos de A no se repiten m odulo
m. Para ver esto, supongamos por contradiccion que existe i, j {1, 2, ..., m1}, i =
j, tal que a i a j (mod m). Entonces, como mcd(a, m) =1, cancelando obtenemos
i j (mod m), contradiccion.
La aplicaci on practica que vamos a encontrar frecuentemente es la que se establece
en el siguiente corolario,
Corolario 3.1 Si mcd(a, m) = 1, entonces
a 1 a 2 a (m1) 1 2 (m1) (mod m)
Prueba: Ejercicio.
48 CONGRUENCIAS
3.6 Congruencias lineales
Es facil e inmediato resolver la ecuaci on x +a b (mod m) : x ba (mod m.)
Consideremos la ecuaci on ax b (mod m) : hay un k Z tal que b ax = mk o
b = mk +ax. Como ya vimos en la seccion sobre ecuaciones diofanticas lineales,
b = mk +ax tiene soluci on si y s olo si mcd(a, m)|b. Si esta es la situaci on, hay un
k

Z tal que b = k

mcd(a, m) y entonces, utilizando el algoritmo extendido de


Euclides, determinamos s, t Z tal que mcd(a, b) = sa+tm = b = k

sa+k

tm y
una soluci on sera x = k

s.
EJEMPLO 3.16 Determinar una soluci on de 2x 5 (mod 7).
Solucion: Como mcd(2, 7) = 1 y 1|5, la ecuaci on tiene soluci on. Como el m odulo
es peque no, podemos encontrar una soluci on por ensayo y error: Sustituimos los
valores x = 0, 1, 2, ..., 6 y buscamos los valores de x que satisfacen la congruencia.
En este caso obtenemos la soluci on x = 6. Esta soluci on es unica m odulo 7.
Si el m odulo es muy grande, podemos encontrar una soluci on usando algoritmo
extendido de Euclides: 1 = 3 2 + 1 7 = 5 = 15 2 + 5 7. As, x = 15 es
una soluci on. La reducci on m odulo 7 nos da x = 6.
Inversos m odulo m
Teorema 3.5 Si mcd(a, m) =1 entonces ax 1 (mod m) tiene soluci on unica x =a
1
m odulo m.
Prueba: Resolver la congruencia ax 1 (mod m) es equivalente a resolver la ecuaci on
ax +my = 1. Como mcd(a, m) = 1, existen s, t Z tal que sa+tm = 1, con lo que
tenemos la soluci on x = s para la ecuaci on ax 1 (mod m).
La unicidad m odulo m signica que si as 1 (mod m) y as

1 (mod m), entonces


s s

(mod m). Para vericar que la soluci on es unica m odulo m, supongamos que
as

1 (mod m), luego, restando tenemos a(s s

) 0 (mod m) = m|a(s s

) pero
como mcd(a, m) = 1 entonces m|(s s

) = s s

(mod m).
Si sa+tm = 1, en la practica tomamos a
1
= s mod m.
EJEMPLO 3.17 Como mcd(27, 31) =1 entonces 27 tiene inverso m odulo 31. Aplicando
el algoritmo extendido de Euclides obtenemos 8 27+7 31 =1. As a
1
=8. En
49
la practica nos interesa la soluci on a
1
=8 mod 31 =23 (pues 8 =1 31+23).
Soluci on general.
Podemos aplicar la teora de ecuaciones diofanticas lineales para obtener el siguiente
resultado,
Teorema 3.6 ax b (mod m) tiene soluci on si y s olo si d = mcd(a, m)|b. Si x
0
es una
soluci on particular, la soluci on general es x x
0
(mod m/d), es decir, obtenemos las
d soluciones m odulo m, x = x
0
+
m
d
t con 0 t < d
Prueba: Para la prueba vamos a usar los teoremas (2.10) y (2.12) de la seccion de
ecuaciones diofanticas. Si ax b (mod m), entonces hay un k Z tal que axmk =
b. Esta ecuaci on diofantica tiene soluci on si y solo si d = mcd(a, m)|b. Si una
soluci on particular es x =x
0
, entonces la soluci on general es x =x
0
+
m
d
t, con t Z
(aqu solo interesa x). Solo falta probar que solo hay d soluciones distintas m odulo
m.
Si x
0
+
m
d
t
1
x
0
+
m
d
t
2
(mod m), usando el hecho de que (m/d)|m, obtenemos que
t
1
t
2
(mod d), es decir, x
0
+
m
d
t
1
y x
0
+
m
d
t
2
son soluciones distintas m odulo m si y
solo si t
1
y t
2
estan en clases distintas de Z
d
. Esto nos deja solo las d posibilidades
t = 0, 1, ..., d 1.
Corolario 3.2 Si p es primo y mcd(a, p) = 1, la ecuaci on lineal ax b (mod m)
tiene soluci on unica x a
1
b (mod p).
Prueba: Ejercicio.
EJEMPLO 3.18 Resolver 2x 5 (mod 7).
Solucion: como mcd(2, 7) =1, la ecuaci on tiene soluci on unica m odulo 7. La soluci on
es x 2
1
5 (mod 7); como 4 2 1 (mod 7), x 4 5 6 (mod 7). As, la soluci on
es x = 6.
EJEMPLO 3.19 Resolver 42x 50 (mod 76).
Solucion: Usando el algoritmo extendido de Euclides obtenemos la soluci on particular
x = 225 3 (mod 76). Ahora, como mcd(42, 76) = 2, la soluci on general es x
3 (mod 38), es decir, x = 3 +38t. La ecuaci on tiene dos soluciones m odulo 76, a
saber x = 3 y x = 41.
50 CONGRUENCIAS
EJEMPLO 3.20 Resolver 12x 48 (mod 18).
Solucion: Una soluci on particular es x =1, ahora, como mcd(12, 18) =6, la soluci on
general es x 1 (mod 3), es decir, x = 1 +3t. La ecuaci on tiene seis soluciones
m odulo 18, a saber x = 1, 4, 7, 10, 13 y 16.
EJEMPLO 3.21 La ecuaci on 2x 3 (mod 4) o tiene soluci on pues mcd(2, 4) = 2 3
3.7 Teorema Chino del resto
Un ejemplo concreto de un sistema de congruencias lineales se describe en el ejemplo
que sigue,
EJEMPLO 3.22 Calcule x tal que
_
_
_
x 1 (mod 3),
x 2 (mod 5),
x 3 (mod 7).
Solucion: Hay un n umero innito
de soluciones para este sistema, por ejemplo x = 52, 53, 157, ... como se puede
vericar rapidamente. Una manera de resolver este sistema es despejar y sustituir
x hasta que la ultima congruncia se a usada,
x 1 (mod 3) = x = 1+3t
1
= 1+3t
1
2 (mod 5)
= 3t
1
1 (mod 5)
= t
1
2 (mod 5) pues 3 2 1 (mod 5)
= t
1
= 2+5t
2
= x = 7+15t
2
= 7+15t
2
3 (mod 7)
= 15t
2
3 (mod 7)
= t
2
3 (mod 7) pues 15 1 1 (mod 7)
= t
2
= 3+7t
= x = 7+15 (3+7t) = 52+105t, t Z.
As, x = 52 +105t, t Z; es la soluci on general del sistema. Aqu debemos notar
que 105 = 3 5 7, es decir, la soluci on es unica m odulo 3 5 7.
Introducci on a la Teora de N umeros.. Walter Mora F.
Derechos Reservados 2009 Revista digital Matem atica, Educacin e Internet (www.cidse.itcr.ac.cr/revistamate/)
51
Teorema 3.7 (Teorema Chino del resto) Consideremos el sistema lineal de congruencias
x a
1
(mod m
1
)
x a
2
(mod m
2
)
. . .
x a
k
(mod m
k
)
con mcd(m
i
, m
j
) = 1, 1 i, j k; entonces, si M = m
1
m
2
m
k
y M
i
= M/m
i
, el
sistema tiene soluci on unica x = a
1
M
1
y
1
+a
2
M
2
y
2
+ +a
k
M
k
y
k
, m odulo M.
Prueba: La prueba
2
es en dos partes, primero se muestra una soluci on de manera
explcita y luego se prueba que es unica.
Sean M = m
1
m
2
m
k
y M
i
= M/m
i
, 1 i k. Como los m odulos son primos
relativos dos a dos, mcd(M
i
, m
i
) = 1 para cada i. Tambien, M
i
0 (mod m
j
) j = i.
Como mcd(M
i
, m
i
) =1, entonces M
i
y
i
1 (mod m
i
) tiene soluci on unica y
i
M
1
i
(mod m
i
).
Sea x = a
1
M
1
y
1
+a
2
M
2
y
2
+ +a
k
M
k
y
k
. Vamos a mostrar que x es soluci on del
sistema de congruencias.
x =
k

i=1
i=j
a
i
M
i
y
i
+ a
j
M
j
y
j

i=1
i=j
a
i
0 y
i
+ a
j
1 (mod m
j
)
0+a
j
(mod m
j
)
a
j
(mod m
j
), 1 j k
Por tanto, x satisface todas las congruencias del sistema, es decir, es una soluci on
del sistema.
Para probar la unicidad m odulo M supongamos que x
1
y x
2
son soluciones del
sistema, vamos a demostrar que x
1
x
2
(mod M).
Puesto que x
1
a
j
(mod m
j
) y x
2
a
j
(mod m
j
) para 1 j k, restando x
1
x
2

0 (mod m
j
), luego m
j
|(x
1
x
2
) para cada j.
2
Para seguir la prueba debemos recordar que si m
1
, m
2
, ..., m
k
son primos relativos dos a dos,
entonces mcm(m
1
, m
2
, ..., m
k
) = m
1
m
2
m
k
y si m
1
, m
2
, ..., m
k
, a Z
+
y m
i
|a, i = 1, 2, ..., k; entonces
mcm(m
1
, m
2
, ..., m
k
)|a.
52 CONGRUENCIAS
Entonces mcm(m
1
m
2
m
k
)|(x
1
x
2
), es decir, M|(x
1
x
2
) pues tambien M =
mcm(m
1
m
2
m
k
). Por tanto, x
1
x
2
0 (mod M), es decir x
1
x
2
(mod M).
EJEMPLO 3.23 Resolver el sistema que sigue, usando el metodo del teorema anterior.
x 1 (mod 3),
x 2 (mod 5),
x 3 (mod 7).
Solucion: M = 3 5 7 = 105, M
1
= 35, M
2
= 21, M
3
= 15. Luego,
y
1
M
1
1
(mod 3) = y
1
2 (mod 3)
y
2
M
1
2
(mod 5) = y
2
1 (mod 5)
y
3
M
1
3
(mod 7) = y
3
1 (mod 7)
As,
x = a
1
M
1
y
1
+a
2
M
2
y
2
+a
3
M
3
y
3
= 1 35 2+2 21 1+3 15 1
= 157 52 (mod M)
Podemos decir, la soluci on unica es x 52 (mod M).
Sistemas con m odulos no coprimos dos a dos
Si los m odulo no son coprimos dos a dos, el sistema podra tener soluci on en las
condiciones del siguiente teorema,
Teorema 3.8 El sistema
x a
1
(mod m
1
)
x a
2
(mod m
2
)
. . .
x a
k
(mod m
k
)
tiene soluci on si y solo si mcd(m
i
, m
j
)|(a
i
a
j
), 1 i, j k. Cuando hay soluci on,
es unica m odulo mcm(m
1
m
2
m
k
).
Si hay soluci on, se puede obtener despejando y sustituyendo como en el ejemplo
(3.22).
53
3.8 Congruencias de Orden Superior
Consideremos de manera general el problema de resolver la congruencia P(x)
0 (mod m) con P(x) un polinomio con coecientes enteros. La manera directa (y no
muy eciente) de resolver este problema es probar con x = 0, 1, ..., m1.
EJEMPLO 3.24 Resolver x
2
+x 2 0 (mod 10).
Solucion: Probamos sustituyendo x = 0, 1, ..., 9 y encontramos las soluciones x =
1, 3, 6, 8.
Si m no es la potencia de un primo, el problema se puede reducir a resolver un
sistema con m odulos menores que m, usando el teorema chino del resto.
Teorema 3.9 Sea m = m
1
m
2
m
k
con m
1
, m
2
, , m
k
primos relativos dos a dos.
x = a es una soluci on de P(x) 0 (mod m) si y solo si a es soluci on del sistema
P(x) 0 (mod m
1
)
P(x) 0 (mod m
2
)
. . .
P(x) 0 (mod m
k
)
Prueba: Si P(a) 0 (mod M), entonces P(a) 0 (mod m
i
), i = 1, 2, ..., k.
Ahora supongamos que x = a es soluci on del sistema, es decir,
P(a) 0 (mod m
1
)
P(a) 0 (mod m
2
)
. . .
P(a) 0 (mod m
k
)
El teorema chino del resto nos dice que P(a) 0 (mod m
1
m
2
m
k
), es decir, x = a
es soluci on de P(x) 0 (mod m).
EJEMPLO 3.25 Resolver x
2
+x 2 0 (mod 10).
Solucion: Como 10 = 2 5, podemos resolver el sistema
P(x) 0 (mod 2)
P(x) 0 (mod 5)
La ganancia sera resolver congruencias con un m odulo m as peque no. Por ensayo y
error,
P(x) 0 (mod 2) tiene soluciones x = 0, 1
P(x) 0 (mod 5) tiene soluciones x = 1, 3
La soluci on del problema requiere resolver los cuatro sistemas
54 CONGRUENCIAS
_
x 0 (mod 2)
x 1 (mod 5)
,
_
x 0 (mod 2)
x 3 (mod 5)
,
_
x 1 (mod 2)
x 1 (mod 5)
,
_
x 1 (mod 2)
x 3 (mod 5)
.
Las soluci on de cada uno de los cuatro sistemas son x = 6, 8, 1, 3, respectivamente.
Por tanto, la soluci on de la congruencia x
2
+x 2 0 (mod 10) es x = 1, 3, 6, 8.
EJERCICIOS
3.1 Hacer las pruebas que se dejaron como ejercicio.
3.2 Calcule el inverso de a = 7 m odulo m = 211.
3.3 Si llamamos a los das de la semana por un n umero 0 d < 7 (0 = domingo,
6 = s abado), describa un algoritmo (usando congruencias) que, sabiendo que hoy
es el da d, nos diga que da sera en n das? Por ejemplo, si hoy es domingo (0), en
7 das es domingo. Por ejemplo, si hoy es lunes(d = 1), que da es en 374 das?.
3.4 Probar que todo entero es congruente con exactamente uno de los residuos
{0, 1, 2, ..., m1} m odulo m.
3.5 Muestre que todo primo p >3 es congruente con 1 o con 5 m odulo 6. Ayuda:
Use el ejercicio anterior.
3.6 Sea S ={2, 3, 5, 7, 11, 13, 17, ...} ={2, 3} {6k 1 : k = 1, 2, ...}. Muestre que
S contiene a todos los primos.
Ayuda: 2, 3 S. Solo falta vericar que si p es primo > 3, entonces p S.
3.7 Muestre que si p es primo y p a, la ecuaci on ax b (mod p) tiene soluci on
unica, m odulo p, x a
p2
b (mod p).
3.8 Z
7
={0, 1, 2, 4, 8, 16, 32}?
3.9 Mostrar que si a Z, Z
m
={a, a+1, ... , a+m1}.
3.10 Mostrar que si m es impar, Z
m
={
m1
2
, . . . , 1, 0, 1, . . .
m1
2
}
3.11 Muestre que si mcd(b, m) > 1, entonces Z
m
={0, b 1, b 2, . . . , b (m1)}
3.12 Sea p primo, muestre que si mcd(a, p) = 1, entonces mcd(a
1
, p) = 1.
3.13 Muestre que si k 5 entonces k! 0 (mod 15)
3.14 Muestre que 3! +4! +5! + +100! es divisible por 15.
3.15 Muestre que N = 11 14
n
+1 es compuesto.
Ayuda: Si n es par, iniciar con 14 1 (mod 3) y recalcular N m odulo 3. Si n es
impar, iniciar con 14 1 (mod 5) y recalcular N m odulo 5.
3.16 Muestre que si P(x) es un polinomio con coecientes enteros y si a b (mod m)
entonces P(a) P(b) (mod m).
3.17 De un ejemplo que muestre que (a b (mod p) y si p|m) = a b mod (m)
EJERCICIOS 55
3.18 Sea p un divisor no trivial de m. Muestre que si x
i
x
j
(mod p) x
i
x
j
mod (m), entonces mcd(x
i
x
j
, m) es un divisor no trivial de m.
3.19 Muestre que si p 1 (mod 4) y p 1 (mod 3), entonces p 1 (mod 12).
Ayuda: Corolario (2.7).
3.20 Muestre que si p 3 (mod 4) y p 2 (mod 3), entonces p 7 (mod 12).
Ayuda: Corolario (2.7).
3.21 Muestre que 4
3q
1 (mod 9), q N.
3.22 Muestre que para cada n N, 4
n
1 (mod 9) o 4
n
4 (mod 9) o 4
n

7 (mod 9)
Ayuda: Calcule primero 4
t
mod9 para t = 0, 1, 2, 3. Luego use el algoritmo de la
divisi on: 4
n
= 4
3q+r
.
3.23 Muestre que 6 4
n
6 (mod 9) pata todo n 0.
3.24 Muestre que (a
1
+a
2
)
2
(a
2
1
+a
2
2
) (mod 2)
3.25 Muestre que (a
1
+a
2
+... +a
n
)
2
(a
2
1
+a
2
2
+... +a
2
n
) (mod 2)
3.26 Muestre que si 0 r < m y a r (mod m), entonces amodm = r
3.27 Muestre que m
1
, m
2
, ..., m
k
Z
+
y si a b (mod m
i
), i = 1, 2, ..., k; entonces
a b (mod mcm(m
1
m
2
m
k
))
3.28 Verique que 11
2
1 (mod 5) y 11 1 (mod 5) pero 11 1 mod 5
3.29 Muestre que si p es primo y si mcd(a, p) =1, entonces si a
2
1 (mod p) =
a 1 (mod p) o a 1 (mod p). Indique adem as, porque se requiere la hip otesis
mcd(a, p) = 1, ?
3.30 Muestre que a
2
a (mod 2) para todo a Z
3.31 Muestre que si ra rb (mod rm) entonces a b (mod m).
3.32 Sean a, b enteros positivos < m. Muestre que a b mod m
3.33 De un ejemplo en el que mcd(a, m) =1 y mcd(b, m) =1 y que a b (mod m).
3.34 Calcule el m as peque no entero positivo n que deja residuo 3 al dividir por
7, residuo 4 al dividir por 9 y residuo 8 cuando se divide por 11.
3.35 Resuelva el sistema
x 21 (mod 3),
x 32 (mod 5),
x 3 (mod 7),
x 9 (mod 11),
x 2 (mod 2),
x 1 (mod 97).
3.36 Un ni no tiene una bolsa con bolinchas. Si las agrupa en pu nos de 7, le sobran
5, Si las agrupa en pu nos de 11, le sobran 6, Si las agrupa en pu nos de 13, le sobran
8. Determine el mnimo n umero de bolinchas que podra tener el ni no.
56 CONGRUENCIAS
3.37 Muestre que el sistema
_
x 2 (mod 4)
x 3 (mod 6)
no tiene soluci on. Ayuda: Use el
metodo de sustitucion y Bezout.
3.38 Calcule el m as peque no entero positivo n tal que 2|n, 3|n+1, 5|n+2, 7|n+3,
y 11|n+4.
3.39 Resuelva el sistema
x 4 (mod 6),
x 2 (mod 8),
x 1 (mod 9).
3.40 Resolver x
5
3x
4
+x 2 0 (mod 165).
4 POTENCIAS mod m
4.1 Orden de un elemento modulo m.
Sea mcd(a, p) = 1, entonces p a
s
si s 1. Como a, a
2
, a
3
, ..., a
p
son p elementos
no nulos del conjunto de p 1 residuos {1, 2, ..., p 1}, entonces al menos dos se
tienen que repetir: Existen s =t tal que a
s
a
t
(mod p).
La parte importante aqu es ver que si s =t +r, entonces, como mcd(a
t
, p) = 1, a
t
tiene inverso, as
a
s
a
t
(mod p) = a
t+r
a
t
(mod p) = a
r
1 (mod p).
EJEMPLO 4.1 En Z
7
, {2, 2
2
, 2
3
, 2
4
, 2
5
, 2
6
, 2
7
} ={2, 4, 1, 2, 4, 1, 2}. En particular, 2
2

2
5
(mod 7), entonces 2
3
1 (mod 7)
Teorema 4.1 Si mcd(a, m) = 1 entonces a
t
1 (mod m) para alg un 1 t < m.
Prueba: Se trata de renar un poco el argumento que se dio m as arriba, as que se
deja como ejercicio.
Denici on 4.1 Sea m 2. Si mcd(a, m) = 1, el orden de a m odulo m es el m as
peque no entero positivo t tal que a
t
1 (mod m)
Si denotamos t = Ord
m
(a), a
t
1 (mod m) pero a
s
1(modm) si 0 < s <t
EJEMPLO 4.2 El orden de 2 m odulo 7 es t = 3 pues 2
3
1 (mod 7) pero 2
2

1(mod7) y 2
1
1(mod7). Observemos que 2
6
1 (mod 7) y 3|6.
Teorema 4.2 Si Ord
m
(a) =t y a
s
1 (mod m), entonces t|s.
Introducci on a la Teora de N umeros.. Walter Mora F.
Derechos Reservados 2009 Revista digital Matem atica, Educacin e Internet (www.cidse.itcr.ac.cr/revistamate/)
57
58 POTENCIAS mod m
Prueba: Si s = kt +r con 0 r < t, se tiene que a
s
a
kt+r
(mod m) = a
r

1 (mod m). Pero como 0 r <t y t es el orden de a, a


r
1 (mod m) solo si r = 0.
Si conocemos el orden de un n umero m odulo m, podramos ganar algo en el c alculo
del orden de otros elementos de Z
m
: Si a tiene orden t m odulo m y queremos
calcular el orden de a
d
, ya se sabe que a
dt
1 (mod m) pero el orden de a
d
es
dt, en realidad tenemos,
Teorema 4.3 Si Ord
m
(a) =t, entonces el orden de a
d
es q =
t
mcd(d, t)
si d > 0
Prueba: a
mcm(d,t)
1 (mod m) pues t| mcm(d, t). Tenemos,
(a
d
)
mcm(d,t)
d
1 (mod m)
Como mcm(d, t) =
dt
mcd(d, t)
=
mcm(d, t)
d
=
t
mcd(d, t)
, entonces
(a
d
)
t
mcd(d,t)
1 (mod m)
Falta probar que el orden de a
d
es
t
mcd(d, t)
: Si (a
d
)
s
= a
ds
1 (mod m) entonces
t|ds. As, ds es m ultiplo de t y es m ultiplo de d, por tanto,
ds mcm(d, t) = s
mcm(d, t)
d
=
t
mcd(d, t)
.
EJEMPLO 4.3 El orden de 2 m odulo 7 es t = 3, entonces el orden de 4 = 2
2
es
3
mcd(2, 3)
= 3, i.e. 4
3
1 (mod 7.)
El orden de 3 m odulo 163 es 162, entonces el orden de 3
26
es
162
mcd(26, 162)
= 81.
4.2 El Teorema peque no de Fermat.
Introducci on a la Teora de N umeros.. Walter Mora F.
Derechos Reservados 2009 Revista digital Matem atica, Educacin e Internet (www.cidse.itcr.ac.cr/revistamate/)
59
Para establecer el teorema peque no de Fermat
3
, observemos que
(a 1)(a 2) (a (p1)) = a
p1
(1 2 (p1))
Pero podemos probar que a 1, a 2, , a (p1) es una permutaci on del conjunto
{1, 2, ..., p1}. Luego, si mcd(a, p) = 1, cancelando tendramos a
p1
1 (mod p.)
EJEMPLO 4.4 Sea a = 270 y p = 7.
270 1 4 (mod 7) 270 2 1 (mod 7)
270 3 5 (mod 7) 270 4 2 (mod 7)
270 5 6 (mod 7) 270 6 3 (mod 7)
As,
270
6
(1 2 3 4 5 6) (270 1)(270 2)(270 3)(270 4)(270 5)(270 6) (mod7)
(4 5 6 1 2 3)(mod7)
entonces, 270
6
(1 2 6) (1 2 6) (mod 7) = 270
6
1 (mod 7)
Antes de enunciar el teorema, establecemos el lema
Lema 4.1 Sea p primo y mcd(a, p) = 1, entonces
a 1 a 2 a (p1) 1 2 (p1) (mod p)
Prueba: Aplicacion directa del lema (3.1) y el corolario (3.1)
Teorema 4.4 (Teorema peque no de Fermat) Sea p primo y a Z. Si mcd(a, p) =
1 entonces,
a
p1
1 (mod p)
si p|a entonces,
a
p
a (mod p.)
3
P. Fermat (1601-1665)
El 18 de octubre de 1640, Fermat escribi o una carta a Bernhard Frenicle de Bessy (1605-
1675), un funcionario de la Casa de la Moneda francesa, excelente alumno en teora de
los n umeros.
En su carta, Fermat comunica el resultado siguiente: Si p es primo y p a
entonces p|a
p1
1. Fermat no present o una prueba de este resultado, pero una nota ad-
junta prometa enviar una demostraci on, siempre que no resultara demasiado extensa.
Sin embargo, la primera prueba conocida la dio Euler un siglo despues. Este resultado es
conocido como el peque no teorema de Fermat para diferenciarlo del ultimo teorema
de Fermat (1637): La ecuaci on x
n
+y
n
= z
n
no tiene soluciones enteras positivas
si n > 2 (demostrado por A.Wiles en 1995.)
60 POTENCIAS mod m
Prueba: Usando lema anterior,
a
p1
(1 2 (p1)) (a 1)(a 2) (a (p1)) (mod p)
1 2 (p1) (mod p)
Entonces a
p1
(1 2 (p1)) 1 2 (p1) (mod p) = a
p1
1 (mod p) pues
mcd(1 2 (p1), p) = 1.
Para probar la segunda armaci on solo hay que observar que si p|a, a 0 (mod p) =
a
p
a (mod p.)
EJEMPLO 4.5 Calcule manualmente el resto de dividir 24
1937
por 17.
Solucion: Por el teorema de Fermat, como mcd(24, 17) = 1, 24
16
1 (mod 17).
Luego, como 1937 = 16 121+1, entonces
24
1937
= 24
16121+1
= 24
16121
24
1
24 (mod 17), es decir, 24
1937
7 (mod 17).
EJEMPLO 4.6 La congruencia x
6
+x
4
+x3 0 (mod 5) tiene las mismas soluciones
que la congruencia x
2
+x 3 0 (mod 5) pues, por el teorema de Fermat, x
4

1 (mod 5) y x
6
= x
4
x
2
x
2
(mod 5)
Teorema 4.5 Sea p primo y a cualquier entero tal que p a. Entonces, a
p2
es el
inverso de a m odulo p.
Prueba: Por el teorema de Fermat, a
p1
1 (mod p), entonces a a
p2
1 (mod p),
es decir, a a
p2
= 1.
Corolario 4.1 Sea p > 1. Si existe a Z, mcd(a, p) = 1, tal que a
p
a (mod p),
entonces p es compuesto.
Prueba: Ejercicio.
El teorema de Fermat no se puede usar, en principio como una prueba de primalidad
pues solo nos da una condici on necesaria pero no suciente. En efecto, hay n umeros
61
compuestos que pasan la prueba del teorema de Fermat para alguna base a.
EJEMPLO 4.7 341 = 11 31 es compuesto, mcd(341, 2) = 1 y 2
340
1 (mod 341), es
decir, 341 pasa la prueba de Fermat en base 2 pero no es primo.
4 no es primo pues 2
4
2 (mod 4).
4.3 Teorema de Euler
El teorema de Euler es uno de los grandes hitos en le desarrollo de la teora
de n umeros. Fue probado por Euler en 1760. Este teorema extiende el teorema
peque no de Fermat a un m odulo arbitrario. Antes de enunciarlo y probarlo,
necesitamos algunos detalles tecnicos.
Denici on 4.2 Para cada n 1, denotamos con (n) la cantidad de enteros positivos
menores que n y coprimos con n. A se le llama funci on phi
4
de Euler.
EJEMPLO 4.8 (24) = 8 pues 1, 5, 7, 11, 13, 17, 19 y 23 son los coprimos con 24
inferiores a 24.
Recordemos que a Z
m
tiene inverso si mcd(a, m) = 1. Luego, (m) calcula la
cantidad de unidades en Z
m
. As, si p es primo, entonces (p) = p1.
EJEMPLO 4.9 Sea m = 9,
a 1 2 3 4 5 6 7 8
mcd(a, 9) 1 1 3 1 1 3 1 1
Figura 4.1 Unidades de Z
9
: (9) = 6.
As, (9) = 6
Teorema 4.6 p es primo si y solo si (p) = p1
4
Euler parece que no usaba una notaci on funcional para esta funci on, el us o en alg un momento
la notaci on n. Gass introdujo la notaci on (n) aunque tambien se usa (n). Sylverter
introdujo la notaci on Totient(n) que a veces aparece en la literatura actual.
Introducci on a la Teora de N umeros.. Walter Mora F.
Derechos Reservados 2009 Revista digital Matem atica, Educacin e Internet (www.cidse.itcr.ac.cr/revistamate/)
62 POTENCIAS mod m
Prueba: Si p es primo, los enteros 1, 2, ..., p1 son primos relativos con p y menores
que p, entonces (p) = p1.
Hay exactamente p1 enteros positivos inferiores a p. Como (p) = p1, ninguno
de estos p1 enteros divide a p, es decir, p es primo.
Corolario 4.2 Sea m es compuesto, (m) < m1.
Prueba: Ejercicio.
Teorema 4.7 Sea p primo y > 1. (p

) = p

p
1
= p
1
(p1)
Prueba: Debemos contar los coprimos con p

e inferiores a p

. Los n umeros divisibles


por p

e inferiores a el son los p


1
n umeros
p, 2p, . . . , p
1
p.
Entonces, en el conjunto {1, 2, . . . , p

} hay p

p
1
elementos coprimos con p

.
EJEMPLO 4.10 (9) = (3
2
) = 3 2 = 6 y (4) = (2
2
) = 2 1 = 1
Si fuera multiplicativa, es decir, si (nm) = (n)(m) cuando mcd(m, n) = 1,
entonces (36) =(2
2
3
2
) =(2
2
)(3
2
) =2 6 =12. Y efectivamente, es multiplicativa.
Para tener una gua para la demostracion de este hecho, necesitamos un lema previo
y un ejemplo numerico.
Lema 4.2 Sean (n, m) = 1 y r un entero, entonces Z
m
= {r, n +r, 2n +r, ..., (m
1)n+r}
Prueba: Solo necesitamos probar que los m elementos de A={r, n+r, 2n+r, ..., (m
1)n+r} no se repiten m odulo m. En efecto, si
jn+r in+r (mod m), i = j, 0 i, j m1;
entonces, como mcd(n, m) = 1, j i (mod m) lo cual es imposible.
EJEMPLO 4.11 Sea n = 4 y m = 9. mcd(4, 9) = 1. Para establecer una gua para la
prueba del teorema, hacemos un arreglo con los n umeros 1, 2, ..., 36,
_

_
1 5 9 13 17 21 25 29 33
2 6 10 14 18 22 26 30 34
3 7 11 15 19 23 27 31 35
4 8 12 16 20 24 28 32 36
_

_
63
La idea es eliminar n umeros hasta que nos quede un arreglo rectangular (n)(m).
La la i es i n+i 2n+i ... (m1)n+i. Como 2 no es primo relativo con
n, entonces 2 ni la la 2 n+2 2n+2 ... (m1)n+2 es prima relativa con
n, as que podemos quitar esta la y, con el mismo argumento, podemos quitar la
la 4.
_

_
1 5 9 13 17 21 25 29 33

3 7 11 15 19 23 27 31 35

_

_
Las las que quedan son las las que inician con primos relativos de n, es decir
quedan (n) = 2 las.
Ahora quitamos, en cada la, los n umeros que no son primos relativos con m,
_

_
1 5 13 17 25 29

3 7 11 19 23 31

_

_
1 5 13 17 25 29
3 7 11 19 23 31
_
En cada la quedan (m) elementos, esto es as pues si hacemos reducci on m odulo
m, por el lema anterior, la la {i, n + i 2n + i ... (m1)n + i} se convierte en
{0, 2, ..., m1}, y en este conjunto solo hay (m) elementos primos relativos con
m. Finalmente el arreglo queda (n)(m). Recordemos que si mcd(a, b) = 1 y
mcd(a, c) = 1, entonces mcd(a, bc) = 1. As, el arreglo tiene todos los primos
relativos con mn e inferiores a mn, es decir (nm).
Teorema 4.8 es multiplicativa, i.e. si mcd(m, n) = 1 = (nm) = (n)(m)
Prueba: Consideremos el arreglo
_

_
1 m+1 2m+1 . . . (n1)m+1
2 m+2 2m+2 . . . (n1)m+2

i m+i 2m+i . . . (n1)m+i

m1
m 2m 3m . . . mn
_

_
Si mcd(i, n) = 1, entonces los elementos de la la i no son primos relativos con n.
Si quitamos estas las, quedan (n) las
r
j
n+r
j
2n+r + j ... (m1)n+r
j
, j = 1, ..., (n)
con mcd(r
j
, n) = 1. Ahora, como Z
m
= {r
j
, n +r
j
, 2n +r + j, ..., (m1)n +r
j
},
entonces en cada la de estas solo hay (m) n umeros primos relativos con m.
64 POTENCIAS mod m
Finalmente nos queda un arreglo de (n)(m) con n umeros ambos primos relativos
con n y m y por tanto, primos relativos con nm. Como todos son inferiores a nm,
(nm) = (n)(m).
El teorema (4.8) nos permite calcular (n) de manera directa, si conocemos la
factorizaci on prima de n,
(n) =
_

k
i=1
p

i
i
_
=
k
i=1
p

i
1
i
(p
i
1).
EJEMPLO 4.12
(6566304875) = (5
3
13
2
310831)
= 5
2
(51) 13(131) (3108311)
= 4848948000.
(15) = (3 5) = (3)(5) = 2 4 = 8, pues mcd(3, 5) = 1.
EJEMPLO 4.13 Calcular Ord
39
(4).
Solucion: Como (39) = 24, Ord
39
(4)|24. Por tanto, debemos probar solo con los
divisores d
i
de 24 hasta que 4
d
i
1 (mod 39).
4 1 mod 39
4
2
16 mod 39
4
3
25 mod 39
4
6
1 mod 39
Ord
39
(4) = 6.
Teorema 4.9 Sea n =
k
i=1
p

i
i
, p
i
primo. Entonces
(n) = n
_
1
1
p
1
__
1
1
p
2
_

_
1
1
p
k
_
65
Prueba: Si n = p

1
, (n) = (p

1
) = p

p
1
= p

1
_
p
1

1
p
1
_
= n
_
p
1

1
p
1
_
.
Si n =
k
i=1
p

i
i
,
(n) =
_
k

i=1
p

i
i
_
=
k

i=1
p

i
i
_
1
1
p
i
_
= n
k

i=1
_
1
1
p
i
_
Este teorema parece algo extra no, para que usar fracciones si podemos calcular
(n) con enteros?. Es cierto. Pero esta forma de expresar sera de mucha utilidad
m as adelante cuando aparezcan los factores 11/p
i
en productos innitos.
Por ahora, vamos a establecer un lema analogo al lema (4.1): a
(m)
1 (mod m) si
mcd(a, m) = 1. Para familiarizarnos, veamos primero un ejemplo.
EJEMPLO 4.14 (12) = 4 cuenta los primos relativos con 12 que son inferiores a 12,
es decir 1, 5, 7, 11.
Ahora, mcd(12, 35) =1 y 35 1 11 (mod 12), 35 5 7 (mod 12), 35 7 5 (mod 12)
y 35 11 1 (mod 12).
As, {35 1, 35 5, 35 7, 35 11} es una permutaci on de {1, 5, 7, 11}.
EJEMPLO 4.15 Muestre que (n) = n/2 n = 2

.
Solucion: Si n = 2

el resultado es directo. En la otra direccion, si n =


k
i=1
p

i
i
y
(n) = n/2, entonces (n) = n
k
i=1
(11/p
i
) = n/2, es decir,
k
i=1
(11/p
i
) = 1/2,
por tanto k =1 y p
1
=2, sino tendramos una contradiccion pues si k >1, entonces

k
i=1
(11/p
i
) < 1/2. Finalmente, n = 2

Lema 4.3 Sea m entero positivo y mcd(a, m) = 1. Sea r


1
, r
2
, . . . , r
(m)
(m) enteros
positivos m y primos relativos con m. Entonces {a r
j
mod m : j = 1, ..., (m)}
es una permutaci on de los enteros r
1
, r
2
, . . . , r
(m)
.
Prueba: Como R = {a r
j
mod m : j = 1, ..., (m)} {r
1
, r
2
, . . . , r
(m)
}, solo hay que
demostrar que R tiene (m) elementos distintos y todos primos relativos con m.
As, estos (m) n umeros son inferiores a m y coprimos con m, entonces constituyen
una permutaci on de r
1
, r
2
, . . . , r
(m)
.
Sea i = j con i, j {1, ..., (m)}. Si a r
i
a r
j
(mod m) entonces, como mcd(a, m) =
1, r
i
r
j
(mod m), pero esto no puede pasar pues m >|r
i
r
j
|.
66 POTENCIAS mod m
Si mcd(a r
i
, m) > 1 entonces sea p un divisor primo de a r
i
y de m. Si p|a r
i
,
entonces p|r
i
o p|a. Pero esto no puede pasar pues si p|r
i
, como p|m entonces
contradice el hecho de que mcd(r
i
, m = 1). Por otra parte p|a contradice el hecho
de que mcd(a, m = 1).
De nuevo, vamos a dar un ejemplo antes de enunciar el teorema de Euler.
EJEMPLO 4.16 (12) = 4 cuenta los primos relativos con 12 que son inferiores a 12,
es decir 1, 5, 7, 11.
Ahora, mcd(12, 35) =1 y 35 1 11 (mod 12), 35 5 7 (mod 12), 35 7 5 (mod 12)
y 35 11 1 (mod 12).
Luego,
35
4
(1 5 7 11) (35 1)(35 5)(35 7)(35 11) (mod 12)
(11 7 5 1) (mod 12)
35
4
1 (mod 12)
pues mcd(11 7 5 1, 12) = 1.
Teorema 4.10 (Teorema de Euler) Sea m entero positivo y mcd(a, m) = 1, entonces
a
(m)
1 (mod m)
Prueba: Sea r
1
, r
2
, . . . , r
(m)
(m) enteros positivos m y primos relativos con m.
Entonces, por el lema (4.3),
a
(m)
(r
1
r
2
r
(m)
) (a r
1
)(a r
2
) (a r
(m)
) (mod m)
(r
1
r
2
r
(m)
) (mod m)
a
(m)
1 (mod m)
pues mcd(r
1
r
2
r
(m)
, m) = 1.
67
EJEMPLO 4.17 El conjunto de unidades de Z
m
se denota (Z/mZ)

. A este conjunto
es un ejemplo de un sistema reducido de residuos.
Es claro entonces que |(Z/mZ)

| = (m) y, en particular, |(Z/mZ)

| = m1 si y
solo si m es primo.
4.3.1 El recproco del Teorema peque no de Fermat
Si p es primo, (p) = p1 y si p no es primo (p) < p1. Entonces si n1 es
el m as peque no entero positivo tal que a
n1
1 (mod n), n debera ser primo.
Teorema 4.11 Si existe un entero a tal que a
n1
1 (mod n) y si a
s
1 (mod n),
para todo s < n1, entonces n es primo.
Prueba: Si n no fuera primo, (n) < n1, pero a
(n)
1 (mod n), contradiccion.
Una version renada es
Teorema 4.12 Si a y n son enteros tales que a
n1
1 (mod n) y a
(n1)/q
1 (mod n)
para todos los divisores primos q de n1, entonces n es primo.
Prueba: Como los divisores propios de n 1 dividen (n 1)/q para alg un primo
q, entonces Ord
m
(a) no divide al entero (n1)/q, seg un la hip otesis del teorema.
Por tanto, la unica posibilidad es que Ord
m
(a) = n 1. As, (Z/nZ)

tiene n 1
elementos, entonces n debe ser primo.
Con este teorema podramos decidir si n es primo si conocemos la factorizaci on de
n1.
EJEMPLO 4.18 Usar el teorema (4.12) para probar que n = 229 es primo.
Solucion: n1 = 2
2
3 19. Para probar que n es primo debemos encontrar 2 a
228 tal que a
228
1 (mod 229) y a
(228)/q
1 (mod 229) para q = 2, 3, 19. Ahora
hacemos una b usqueda exhaustiva:
a a
n1
mod 229 a
(n1)/2
mod 229 a
(n1)/3
mod 229 a
(n1)/19
mod 229
2 1 228 1 203
3 1 1 134 161
4 1 1 1 218
5 1 1 94 61
6 1 228 134 165
Tabla 4.1 n = 229 es primo seg un el teorema (4.12).
68 POTENCIAS mod m
n =229 cumple las condiciones del teorema (4.12) para a =6. Por tanto, n =229
es primo.
4.4 Teorema de Wilson
Sean n, r enteros no negativos. Recordemos que 0! =1 y si n r,
_
n
r
_
=
n!
r!(nr)!
.
Se puede probar que
_
n
r
_
es un entero procediendo por induccion y usando la
identidad de Pascal:
_
n
r
_
=
_
n1
r 1
_
+
_
n1
r
_
El teorema del binomio establece que si x, y R y n no negativo,
(x +y)
n
=
n

k=0
_
n
k
_
x
k
y
nk
Se asume como convenio que 0
0
= 1 para el caso especial x = 0, y = 0, n = 0.
Teorema 4.13 Si p es primo y 0 < r < p, entonces p

_
p
r
_
Prueba:
_
p
r
_
es un entero as que r!(pr)!|p!. Como p r! y p ((pr)!), entonces
p r!(pr)!. As r!(pr)!|p! mcd(p, r!(pr)!) = 1 = r!(pr)!|(p1)!.

_
p
r
_
= p
(p1)!
r!(pr)!
.
Teorema 4.14 Si p es primo, entonces (x +y)
p
x
p
+y
p
(mod p) si x, y Z.
Prueba: Como p

_
p
r
_
si r = 1, 2, . . . , p1, entonces
(x +y)
p

_
p
0
_
y
p
+
_
p
1
_
x
1
y
p1
+ +
_
p
p1
_
x
p1
y
1
+
_
p
p
_
x
p
(mod p)
1 y
p
+0+ +0+1 x
p
(mod p)
En general, si p es primo, entonces (x
1
+x
2
+ +x
n
)
p
x
p
1
+x
p
2
+ +x
p
n
(mod p)
si x
i
Z.
Introducci on a la Teora de N umeros.. Walter Mora F.
Derechos Reservados 2009 Revista digital Matem atica, Educacin e Internet (www.cidse.itcr.ac.cr/revistamate/)
69
Teorema 4.15 Sea p primo y P(x) = a
n
x
n
+ +a
1
x+a
0
Z[x] con a
n
0 (mod p).
Entonces P(x) 0 (mod p) tiene a lo sumo n soluciones (enteras) distintas m odulo
p.
Prueba: La prueba es por induccion sobre el grado de P. .
Si n = 0 no hay soluciones pues a
0
0 (mod p).
Si n = 1, la congruencia a
1
x +a
0
0 (mod p) tiene una solo soluci on (m odulo p)
pues a
1
x a
0
(mod p) tiene soluci on unica m odulo p si mcd(a
1
, p) = 1.
Supongamos que el resultado es cierto para polinomios de grado n 1. Para
el resto de la prueba vamos a razonar por contradiccion: Supongamos que P(x) =
a
n
x
n
+ +a
1
x+a
0
tiene s >n soluciones a
1
, a
2
, ..., a
s
distintas m odulo p. Consideremos
ahora
Q(x) = P(x) a
n
(x a
1
)(x a
2
) (x a
n
).
Observe que Q es de grado n1 pues
Q(x) = P(x) a
n
(x a
1
)(x a
2
) (x a
n
) = a
n
x
n
+... (a
n
x
n
+...)
y tiene al menos n races pues Q(x) 0 (mod p) si x =a
1
, x =a
2
, ..., a
n
. Por hip otesis
de induccion, como Q tiene grado n 1, la unica posibilidad es que Q sea el
polinomio nulo, es decir, Q(x) 0 (mod p) para toda x Z. En particular, Q(a
s
)
0 (mod p), Entonces
Q(a
s
) P(a
s
) a
n
(a
s
a
1
)(a
s
a
2
) (a
s
a
n
) (mod p)
a
n
(a
s
a
1
)(a
s
a
2
) (a
s
a
n
) (mod p)
0 (mod p)
Luego, p|a
n
(a
s
a
1
)(a
s
a
2
) (a
s
a
n
) y como p a
n
, p divide alg un factor (a
s

a
j
) con lo que a
j
a
s
(mod p) en contradiccion con nuestra hip otesis.
Teorema 4.16 Sea p primo. Entonces,
x
p1
1 (x 1)(x 2) (x p+1) (mod p)
Prueba: Como p es primo y como p es coprimo con 1, 2, ..., p1, por el teorema
de Euler tenemos, x
p1
1 (mod p) para x = 1, 2, ..., p 1. Entonces, x
p1
1 es
un polinomio con p1 races. Usando el teorema (4.15) tenemos
x
p1
1 (x 1)(x 2) (x p+1) (mod p) si x = 1, 2, ..., p1
70 POTENCIAS mod m
Pero, como Q(x) =x
p1
1(x1)(x2) (xp+1) tiene grado p2 y p1
races, Q(x) 0 (mod p), es decir,
x
p1
1 (x 1)(x 2) (x p+1) (mod p) si x Z
Teorema 4.17 (Teorema de Wilson) p es primo si y solo si (p1)! 1 (mod p)
Prueba: Si p es primo, por el teorema (4.16),
x
p1
1 (x 1)(x 2) (x (p1)) (mod p) para cualquier x Z.
Poniendo x = 0, 1 (1)
p1
1 2 (p1) (mod p). Si p es impar, obtenemos el
resultado. Si p = 2 el resultado es directo.
Si (p1)! 1 (mod p) y p tiene un divisor d, 1 < d < p, entonces d|(p1)! +1
pero d|(p1)! pues 1 < d < p, as que d|1, contradiccion.
Es claro que no es practico usar este teorema para vericar si p es o no primo.
4.5 Teorema de Carmichael
El c alculo del orden de un n umero a puede ser complicado. Algo que nos puede
ayudar es saber que este orden es inferior a (a) y un factor de la funci on (a) de
Carmichael.
Denici on 4.3 (Funci on de Carmichael) Sean p, p
1
, p
2
, ..., p
s
primos, la funci on se
dene as:
(1) = 1,
(2) = 1,
(4) = 2,
(2

) = 2
2
si 3,
(p

) = (p

) = p
1
(p1) si p
i
3 y 1,
(n) = mcm((p

1
1
), (p

2
2
), . . . , (p

k
k
)) si n =
k
i=1
p

i
i
_

_
EJEMPLO 4.19 (1) = 1, (2) = 1, (3) = 2, etc.
n 1 2 3 4 5 6 7 8 9 10 100 101 102 103
(n) 1 1 2 2 4 2 6 2 6 4 20 100 16 102
Introducci on a la Teora de N umeros.. Walter Mora F.
Derechos Reservados 2009 Revista digital Matem atica, Educacin e Internet (www.cidse.itcr.ac.cr/revistamate/)
71
Teorema 4.18 (Teorema de Carmichael) Sean a, n Z
+
y mcd(a, n) = 1. Entonces
a
(n)
1 (mod n)
Prueba: n =
k
i=2
2

i
i
, p
i
primo y mcd(a, n) = 1. Por la denicion de la funci on
, se tiene
(1) a
(2

)
1 (mod 2

)
(2) a
(p

1
1
)
1 (mod p

1
1
)
(3) a
(p

2
2
)
1 (mod p

2
2
)
. . .
(k) a
(p

k
k
)
1 (mod p

k
k
)
Ahora, en la primera congruencia, elevamos a ambos lados a la potencia entera
(n)/(2

) y en la congruencia iesima elevamos a ambos lados a la potencia


entera (n)/(p

i
i
), obtenemos
a
(n)
1 (mod 2

)
a
(n)
1 (mod p

1
1
)
a
(n)
1 (mod p

2
2
)
. . .
a
(n)
1 (mod p

k
k
)
Para concluir, recordemos que si m
1
, m
2
, ..., m
k
Z
+
y si a b (mod m
i
), i =1, 2, ..., k;
entonces a b (mod mcm(m
1
m
2
m
k
)). Usando este hecho, podemos concluir que
a
(n)
1 (mod n).
Se gana algo usando (n) en vez de (n)?
Con un esfuerzo razonablemente peque no, podemos obtener, en general, mejores
resultados con .
EJEMPLO 4.20 Sea n =65520 =2
4
3
2
5 7 13. Entonces (n) =8 6 4 6 12 =13824
mientras que (n) = mcm(4, 6, 4, 6, 12) = 12. Entonces, si mcd(a, n) = 1,
a
(n)
1 (mod 65520) = a
12
1 (mod 65520)
a
(n)
1 (mod 65520) = a
13824
1 (mod 65520)
Para encontrar el orden de a, se puede probar con los divisores de 12 en vez de
calcular y usar los divisores de 13824
EJEMPLO 4.21 Calcular Ord
39
(4).
72 POTENCIAS mod m
Solucion: Como (39) = 12, Ord
39
(4)|12. Por tanto, debemos probar solo con los
divisores d
i
de 12 hasta que 4
d
i
1 (mod 39).
4 1 mod 39
4
2
16 mod 39
4
3
25 mod 39
4
6
1 mod 39
_

_
= Ord
39
(4) = 6.
Uno de los resultados importantes es: (n) es el m as peque no entero positivo para
tal quea
(n)
1 (mod m) para todo a tal que mcd(a, m) = 1 ([1]).
(n) y (n) si mcd(a, n) > 1
El teorema de Euler y Carmichael requieren mcd(a, n) = 1. Sin embargo hay una
version util para el caso en que mcd(a, n) no sea necesariamente 1.
Teorema 4.19 Sea d = mcd(a, n),
a
(n)+1
a (mod n) mcd
_
d,
n
d
_
= 1.
Si n es producto de primos distintos,
a
(n)+1
a (mod n) para cualquier a Z.
Prueba: Ver ([9], pag. 274).
EJEMPLO 4.22 Sea a =7 y n =210, mcd(a, n) =7 >1, (n) =48, 7
49
7 (mod 210)
y mcd(7, 30) = 1 (tambien 7
1
7 (mod 210), 7
5
7 (mod 210), etc.).
Sea n = 2 3 5 7, (n) = 12, 2
13
2 (mod 210), 3
13
3 (mod 210), etc.
Sea n = 2
2
5, en este caso (n) = 4 y 2
5
2 (mod n), 3
5
3 (mod n), . . . , 6
5

6 (mod n), etc.


EJERCICIOS
4.1 Verique, usando el teorema de Fermat, que 2
340
1 (mod 11).
4.2 Verique que 2
340
1 (mod 31). Ayuda: 2
5
1 (mod 31)
4.3 Verique que mcd(341, 2) = 1 y 2
340
1 (mod 341). Es 341 primo?
4.4 Verique que (666) = 6 6 6
EJERCICIOS 73
4.5 Consideremos los n umeros de Fermat, F
n
= 2
2
n
+1. Vamos a probar, usando
congruencias, que si n = m, mcd(F
n
, F
m
) = 1. Para probar esto, vamos a suponer,
por contradiccion, que mcd(F
n
, F
m
) =d >1. Entonces hay un primo p tal que p|F
n
y p|F
m
. Bajo esta suposici on,
a) verique que 2
2
n
1 (mod p),
b) verique que 2
2
n+1
1 (mod p).
c) Sea Ord
p
(2) = 2
s
. Porque 2
s
n+1?
d) Verique que s n. Ayuda: considere 2
n
= 2
s
2
t
con s +t = n y obtenga
una contradiccion con 2
2
n
1 (mod p).
e) Deduzca que el orden de 2 m odulo p es 2
n+1
f) Deduzca que el orden de 2 m odulo p debera ser tambien 2
m+1
g) Cu al es la contradiccion?
4.6 Use el resultado anterior para dar otra prueba de que los primos son un
conjunto innito. Ayuda: para cada F
n
considere uno de sus divisores primos.
4.7 Sea mcd(a, m) = 1 y Ord
m
(a) = t. Si i, j Z, a
i
a
j
(mod m) i
j (mod t).
4.8 Muestre que si m es compuesto y mcd(a, m) = 1, entonces Ord
m
(b) < m1.
Ayuda: Use el teorema de Euler.
4.9 Muestre que si p es primo y t (p1), entonces no pueden haber elementos
de orden t en Z
p
. Ayuda: Use Fermat.
4.10 Sea Ord
m
(a) =t. Muestre que Ord
m
(a
i
) =t si y solo si mcd(i, t) = 1.
4.11 Calcule Ord
13
(5) y Ord
13
(7)
4.12 Sea mcd(a, m) = 1. Muestre que Ord
m
(a) divide a (m).
4.13 Muestre que si p es primo y mcd(a, p) = 1, entonces Ord
p
(a)|p1.
4.14 Muestre que si Ord
m
(a) =t y k es cualquier entero positivo, entonces Ord
m
(a
k
) =
1 si y solo si mcd(t, k) = 1.
4.15 Sean a = 7 y m = 310.
a) Calcule (m) y (m).
b) Se puede armar, sin calcular, que a
(m)
1 (mod m)?
c) Obtenga Ord
m
(a) Ayuda: solo debe probar con los divisores de (m).
4.16 Sean a = 7 y m = 210.
a) Tiene sentido hablar de Ord
m
(a)?
b) Calcule s tal que a
s
a (mod m).
4.17 Calcule Ord
2337
(2)
4.18 Muestre que si p es primo y Ord
p
(a) =t, entonces las soluciones, m odulo p,
de x
t
1 0 (mod p) son {1, a, a
2
, ..., a
t1
}. porque no hay m as soluciones m odulo
p?
74 POTENCIAS mod m
4.19 Sea p primo impar. Muestre que si P(x) es un polinomio con coecientes
enteros de grado n 1 y coeciente principal a
n
0 (mod p), entonces hay un
polinomio Q(x) Z[x] de grado 0 < m < p tal que P(x) Q(x) (mod p).
4.20 Muestre el teorema peque no de Fermat usando el teorema de Euler.
4.21 Muestre el teorema de Euler usando el teorema de Carmichael.
4.22 Muestre que si n es par entonces (2n) =2(n) y que si n es impar entonces
(2n) = 2(n). Ayuda: Teorema (4.8).
4.23 Calcule (25) usando el teorema (5.2).
4.24 Factorizar n = 2337 y calcular (n) y (n)
4.25 Calcule las races (si hubiera) de P(x) = x
5
+1 m odulo 5
4.26 Calcule las races (si hubiera) de P(x) = x
5
1 m odulo 5
4.27 Calcule 96
1
m odulo 97. Luego calcule el resto de dividir 95! por 97. Ayuda:
Fermat y Wilson.
4.28 Sea p primo impar y mcd(a, p) = 1. Mostrar que a
(p1)/2
1 (mod p) o
a
(p1)/2
1 (mod p). Ayuda: Usar la tercera formula notable y el teorema de
Fermat.
4.29 Resuelva 7x 1 (mod 2
6
3 5 17) usando primero el teorema peque no de
Fermat y luego usando el teorema de Carmichael.
4.30 Mostrar que si p es primo, entonces (x
1
+x
2
+ +x
n
)
p
x
p
1
+x
p
2
+ +
x
p
n
(mod p) si x
i
Z.
4.31 Sean mcd(a, b) = 1 y S = a
(b)
+b
(a)
. Muestre que S 1 (mod ab)
4.32 Use el teorema peque no de Fermat para probar que si p es primo y
mcd(p, n) = 1 y p|4n
2
+ 1, entonces p 1 (mod 4). Ayuda: Muestre que p
3 (mod 4) por contradiccion: Si p = 4k +1 y si y = 2n, y
2
1 (mod p) luego,
como mcd(p, y) = 1, aplique Fermat.
4.33 Muestre que si p es primo y mcd(p, n) =1 y p|n
2
+1, entonces p 1 (mod 4)
o p = 2.
4.34 Muestre que en Z
8
, el polinomio P(x) = x
2
1 tiene 4 races: x = 1, 3, 5, 7,
es decir, P(1) 0 (mod 8), etc. Contradice esto el teorema (4.15)?
4.35 Muestre el teorema peque no de Fermat usando el teorema del binomio.
Ayuda: x
i
= 1.
4.36 Muestre que si a 1 (mod 2), entonces a
2
1 (mod 2
3
). Ayuda: a = 2h+1,
eleve al cuadrado y observe que h(h+1) es par.
4.37 Muestre que si a
2
1 (mod 2
3
) entonces a
2
2
1 (mod 2
4
).
4.38 Use induccion para demostrar que si > 2, entonces a
2
2
1 (mod 2

).
4.39 Verique que si > 2, entonces a
1
2
(2

)
1 (mod 2

).
EJERCICIOS 75
4.40 Muestre que si n =
k
i=1
p

i
i
, p
i
primo; entonces (p

i
i
)|(n)
4.41 Muestre que (n)|(n)
4.42 Muestre que (n) = (n) si n = 1, 2, 4, p

, 2p

5
RA

ICES PRIMITIVAS Y
LOGARITMO
DISCRETO
5.1 Introduccion
Se puede demostrar que si p es primo, existe b Z tal que Z
p
={Z
p
={0, b, b
2
, ..., b
p1
}.
A b se le llama raz primitiva m odulo p. Como cualquier elemento a Z
p
debe
ser una potencia de b, tiene sentido denir un logaritmo discreto (indicador) que
resulta tener propiedades similares al logaritmo usual. Es muy util en el c alculo
de residuos y para resolver algunos tipos de ecuaciones congruenciales. Las races
primitivas m odulo n son usadas a menudo en criptografa.
5.2 Ra

ices primitivas
Denici on 5.1 (Races primitivas) Sea m Z
+
y mcd(a, m) =1. Si Ord
m
(a) =(m)
entonces a se dice raz primitiva m odulo m
Teorema 5.1 Si p es primo y b raz primitiva m odulo p, entonces
Z
p
={0, b, b
2
, ..., b
p1
}.
Prueba: Ejercicio.
EJEMPLO 5.1 Ord
5
(3) = 4 pues 3
2
4 (mod 5), 3
3
2 (mod 5) y 3
4
1 (mod 5).
Entonces,
Z
5
={0, 3, 3
2
, 3
3
, 3
4
}
Existencia de las races primitivas.
Para establecer la existencia de las races primitivas en cualquier Z
p
, p primo;
necesitamos algunos resultados.
El teorema que sigue establece que

d|n
d>0
(d) = n. Para la demostracion, se usa un
conjunto S
d
={
1
d
,
2
d
, . . . ,
d
d
} y otros conjuntos de fracciones irreducibles (disjuntos):
T
d
=
_
i
d
S
d
tal que mcd(i, d) = 1
_
.
La idea es contar la cantidad de primos relativos como el n umero de fracciones
irreducibles. El siguiente ejemplo muestra la idea de la prueba.
EJEMPLO 5.2 Sea n = 4. Los divisores positivos de 4 son 1, 2, 4. Entonces,
S
4
=
_
1
4
,
2
4
,
3
4
,
4
4
_
= |S
4
| = 4
T
4
=
_
1
4
,
3
4
_
= |T
4
| = (4) = 2
T
2
=
_
1
2
_
= |T
2
| = (2) = 1
T
1
=
_
1
1
_
= |T
1
| = (1) = 1
Observemos que T
i

T
j
= / 0 y que |S
4
| =|T
4
| +|T
2
| +|T
1
|, es decir,
4 = (1) +(2) +(4)
Teorema 5.2 Sea n un entero positivo, entonces

d|n
d>0
(d) = n
Prueba: Sea S
n
={
1
n
,
2
n
, . . . ,
n
n
} y sea T
n
={
i
n
S
n
tal que mcd(i, n) =1}. Claramente
|S
n
| = n y |T
n
| = (n). Ahora, si d|n, d > 0; T
d
={
i
d
S
d
tal que mcd(i, d) = 1},
entonces
Introducci on a la Teora de N umeros.. Walter Mora F.
Derechos Reservados 2009 Revista digital Matem atica, Educacin e Internet (www.cidse.itcr.ac.cr/revistamate/)
77
78 RA

ICES PRIMITIVAS Y LOGARITMO DISCRETO


S
n
=
_
d|n
d>0
T
d
= T
1
. . . T
n
En efecto,
:
i
n
S
n
tenemos dos casos. Si mcd(i, n) = 1,
i
n
T
n
. Si
i
n
no esta en forma
reducida, entonces, usando la factorizaci on prima de i y de n, simplicamos y nos
queda
i
n
=
k
h
con mcd(k, h) =1 y 1 k h. Como h|n, entonces
k
h
T
h
y por tanto
esta en la uni on de los T
d
s.
: Ahora si s

d|n
d>0
T
d
, entonces s T
h
para alg un h|n. Por tanto, s =
j
h
con
mcd( j, h) = 1 y 1 j h. Si n = k

h, se tiene jk

n, as s =
j
h
=
k

j
k

h
S
n
.
Si d y d

son divisores distintos de n, T


d
T
d
= / 0. Esto es as pues si s esta en esta
la intersecci on, s =
j
d
=
i
d

= id = jd

y entonces, como mcd( j, d) = mcd(i, d

) =1,
d|d

y d

|d. Esto contradice que d = d

.
Finalmente, |S
n
| =
d|n
d>0
|T
d
|, entonces, como |T
d
| = (d), n =

d|n
d>0
(d).
El teorema (5.2) nos da una formula recursiva para calcular (n). No es un formula
adecuada para c alculos porque requiere todos los divisores (primos y compuestos)
de n
EJEMPLO 5.3 Como (1) = 1, (3) = 2 y (5) = 4, entonces
(1) +(3) +(5) +(15) = 15 = (15) = 8.
Teorema 5.3 Sea p primo y t un entero positivo. Si t (p1) entonces Z
p
no tiene
elementos de orden t. Si t|(p1), hay exactamente (t) elementos de orden t en
Z
p
Prueba: De acuerdo al teorema peque no de Fermat, para cada a Z
p
, a = 0,
a
p1
1 (mod p). Luego, si a es de orden t, t|(p 1), o lo que es lo mismo, si
t (p1) no hay elementos de orden t.
Para probar la segunda parte, denimos una nueva funci on (t) : Para cada entero
positivo s que divide a p1, sea (s) el n umero de elementos de orden s en Z
p
.
Ahora, como cada elemento en Z

p
tiene alg un orden s que divide a p1, entonces

t|(p1)
(t) = p1
79
Por el teorema (5.2),
t|(p1)
(t) = p1, entonces

t|(p1)
((t) (t)) = 0
Pero, para cualquier entero t, no hay elementos de orden t o hay exactamente (t)
elementos de orden t; entonces (t) (t) 0 para todo t. Como los sumandos
son 0 y la suma da cero, cada sumado vale cero: (t) =(t) para cada t|(p1).
Corolario 5.1 Si p es primo, en Z
p
hay (p1) elementos de orden p1, es decir,
hay p1 races primitivas.
EJEMPLO 5.4 Cuando tenemos un primo peque no, podemos localizar las races primitivas
por ensayo y error, construyendo una tabla de potencias.
a a
1
a
2
a
3
a
4
a
5
a
6
Ord
7
(a)
1 1 1 1 1 1 1 1
2 2 4 1 2 4 1 3
3 3 2 6 4 5 1 6
4 4 2 1 4 2 1 3
5 5 4 6 2 3 1 6
6 6 1 6 1 6 1 2
Tabla 5.1 Potencias de los elementos de Z
7
Solo hay (6) =2 races primitivas m odulo 7, 3 y 5 tienen orden 6, es decir, son las
unicas dos races primitivas m odulo 7. Tambien, por ejemplo, Z
7
={0, 5, 5
2
, 5
3
, 5
4
, 5
5
, 5
6
}.
EJEMPLO 5.5 La tabla que sigue e un listado de las races primitivas de los primeros
seis primos.
p (p1) Races primitivas
2 1 1
3 1 2
5 2 2, 3
7 2 3, 5
11 4 2, 6, 7, 8
13 4 2, 6, 7, 11
Tabla 5.2 Races primitivas m odulo p
80 RA

ICES PRIMITIVAS Y LOGARITMO DISCRETO


Si b es raz primitiva del primo p, hay (p1) races primitivas no congruentes, a
saber, b

1
, b

2
, ..., b

(p1)
, donde
1
,
2
, ...,
(p1)
son los (p1) enteros menores
que p1 y coprimos con p1.
EJEMPLO 5.6 Determinar las races primitivas de 17 sabiendo que 3 es raz primitiva
m odulo 17.
Solucion: Como (17) = 8, los ocho enteros menores que 16 y coprimos con 16 son
1, 3, 5, 7, 9, 11, 13 y 15. As, la races primitivas son 3
3
, 3
5
, 3
7
, 3
9
, 3
1
1, 3
1
3. Haciendo
la reducci on a m odulo 17, no queda 3, 10, 5, 11, 14, 7, 12, 6.
Ya probamos la existencia de races primitivas para p primo. El siguiente teorema
dene la situaci on general.
Teorema 5.4 (Gauss) Un entero n > 1 tiene races primitivas m odulo n si y solo si
n = 2, 4, p

o 2p

donde p es primo impar y entero positivo.


EJEMPLO 5.7 Los primeros n para los que hay races primitivas son 2, 3, 4, 5, 6, 7, 9, 10, 11, 13,
14, 17, 18, 19, 22, ... . En particular, no hay races primitivas m odulo n = 2
4
= 16.
En resumen, podemos determinar si hay o no hay races primitivas m odulo n y el
c alculo de estas races se hace usando prueba y error (aunque hay unas pocas
tecnicas generales de c alculo).
5.3 Logaritmo discreto o Indicador
El problema que queremos resolver es el siguiente: Si sabemos que a b
k
(mod m),
C omo determinar k?
Recordemos que en Z
n
hay (n) elementos primos relativos con n.
Denici on 5.2 Sea n Z
+
. El conjunto {a
1
, a
2
, ..., a
(n)
} es un sistema reducido de
resi-duos m odulo n, si contiene exactamente un elemento de cada una de las clases
r
i
Z
n
para las que mcd(r
i
, n) = 1.
EJEMPLO 5.8 Si n = p es primo, {1, 2, ..., p1} es un sistema reducido de residuos
m odulo p.
Introducci on a la Teora de N umeros.. Walter Mora F.
Derechos Reservados 2009 Revista digital Matem atica, Educacin e Internet (www.cidse.itcr.ac.cr/revistamate/)
81
Si n = 10, (10) = 4. En este caso, {1, 3, 7, 9} es un sistema reducido de residuos
m odulo 10.
Claramente, si b es una raz primitiva m odulo n, el conjunto {b, b
2
, ..., b
(n)
} es
un sistema reducido de residuos. Entonces, si a Z
n
con mcd(a, n) = 1, existe
1 k (n) tal que a b
k
(mod n). En particular 1 b
(n)
(mod n), por el teorema
de Euler.
Denici on 5.3 Sea b una raz primitiva m odulo n. Si mcd(a, n) = 1, entonces el
m as peque no entero positivo k tal que a b
k
(mod n) se denota con Ind
b
(a) y se
llama indicador de a respecto a la base b m odulo n.
Entonces tenemos,
a b
Ind
b
(a)
(mod n)
A veces se pone Ind
b
(a) = log
b
a y se le llama logaritmo discreto.
Propiedades.
Las propiedades de Ind
b
(a) son similares a las de la funci on logaritmo.
Teorema 5.5 Sea b raz primitiva m odulo n y mcd(a, n) = mcd(c, n) =1. Entonces,
b
x
b
y
(mod n) x y (mod (n)) (5.1)
Ind
b
(1) 0 (mod (n)) (5.2)
Ind
b
(ac) [Ind
b
(a) +Ind
b
(c)] (mod (n)) (5.3)
Ind
b
(a
k
) k Ind
b
(a) (mod (n)), si k es entero positivo. (5.4)
Prueba: Ejercicio.
La propiedad (5.1) nos dice que,
a b
k
(mod n) Ind
b
(a) = k mod (n) (5.5)
La reducci on m odulo (n) es necesaria para obtener el menor exponente positivo,
excepto cuando 1 b
(n)
(mod n), es claro que, como 1 b
(n)
(mod n), Ind
b
(1) =
(n).
EJEMPLO 5.9 b = 5 es raz primitiva m odulo 7 y (7) = 6,
82 RA

ICES PRIMITIVAS Y LOGARITMO DISCRETO


2 5
10
(mod 7) Ind
5
(2) =10 mod 6 =4, es decir, efectivamente 2 5
4
(mod 7).
1 5
6
(mod 7) Ind
5
(1) = 6 mod 6 = 0, es decir, como efectivamente indica
(5.3), 6 = Ind
5
(1) 0 (mod 6).
Observe que (5.1) nos dice que Ind se puede aplicar igual que se aplican los
logaritmos para resolver ecuaciones,
g(x) f (x) (mod m) = Ind
b
(g(x)) Ind
b
( f (x)) (mod (m)).
Por supuesto, la aplicaci on de esta parte del teorema requiere tener a la mano una
tabla de indicadores. En el ejemplo que sigue construimos una breve tabla para
Ind
2
(a) m odulo 13.
EJEMPLO 5.10 Como es usual, para usar el teorema (5.5) necesitamos una tabla de
logaritmos discretos. Por ejemplo, para construir una tabla parcial en base b = 2
m odulo 13, calculamos las potencias de 2 m odulo 13.
2 2
1
(mod 13), 11 2
7
(mod 13),
4 2
2
(mod 13), 9 2
8
(mod 13),
8 2
3
(mod 13), 5 2
9
(mod 13),
3 2
4
(mod 13), 10 2
10
(mod 13),
6 2
5
(mod 13), 7 2
11
(mod 13),
12 2
6
(mod 13), 1 2
12
(mod 13).
Luego, ponemos la informaci on en una tabla,
a 1 2 3 4 5 6 7 8 9 10 11 12
Ind
2
(a) 12 1 4 2 9 5 11 3 8 10 7 6
Tabla 5.3 Logaritmos discreto base b = 2 m odulo 13
EJEMPLO 5.11 Resolver 8x
5
2 (mod 13).
Solucion: Como b = 2 es raz primitiva m odulo 13 y como conocemos una tabla de
loga-ritmos discretos para esta base, podemos aplicar Ind
2
() a ambos lados de
la ecuaci on 8x
5
2 (mod 13),
8x
5
2 (mod 13) =Ind
2
(8x
5
) Ind
2
(2) mod (13), por (5.1)
Ahora operamos,
83
Ind
2
(8x
5
) Ind
2
(2) mod (13) =
Ind
2
(8) +5Ind
2
(x) 1 mod 12, por (5.2), (5.3), y (5.4)
3+5Ind
2
(x) 1 mod 12, pues Ind
2
(8) = 3
5Ind
2
(x) 2 mod 12,
Ind
2
(x) 10 mod 12, pues 5 5 1 (mod 12)
Ind
2
(x) 2 mod 12, pues 10 2 (mod 12)
x 2
2
mod 13, por 5.5
Y, efectivamente, 8 4
5
= 8192 2 (mod 13).
EJEMPLO 5.12 Resolver 2
3x
8 (mod 13).
Solucion: Como b = 2 es raz primitiva m odulo 13, podemos aplicar Ind
2
() a
ambos lados,
2
3x
8 (mod 13) = Ind
2
(2
3x
) Ind
2
(8) (mod 12)
= 3x 3 (mod 12)
= x 1 (mod 4)
Ahora, los elementos en Z
13
que son congruentes con 4 m odulo 13 son x = 1, 5, 9.
EJEMPLO 5.13 Calcule el residuo de dividir 12
729
7
97
por 17.
Solucion: 3 es raz primitiva m odulo 17. As que podemos tomar logaritmo discreto
en base 3. En particular, Ind
3
(12) = 13 y Ind
3
(7) = 11.
Sea x 12
729
7
97
(mod 17), aplicando logaritmo discreto,
Ind
3
(x) Ind
3
(12
729
7
97
) (mod 16),
entonces Ind
3
(x) 729 Ind
3
(12) +97 Ind
3
(7) 9 13+1 11 0 (mod 16), es decir,
x 1 (mod 17.)
EJEMPLO 5.14 Probar que la congruencia 25x
5
17 (mod 71) no tiene soluci on
Solucion: Para aplicar logaritmo discreto a ambos lados, necesitamos una raz primitiva
m odulo 71. Como Ord
71
(7) =70, g =7 es raz primitiva m odulo 71. En particular,
Ind
7
(17) = 49 y Ind
7
(25) = 15. Ahora,
Ind
7
(25x
5
) Ind
7
(17) (mod 70) 5 Ind
7
(x) Ind
7
(17) Ind
7
(25) (mod 70), es
decir, 5 Ind
7
(x) 34 (mod 70). Esta congruencia no tiene soluci on pues mcd(5, 70) =
5 34.
84 RA

ICES PRIMITIVAS Y LOGARITMO DISCRETO


Comparado con el logaritmo com un, el logaritmo discreto tiene dos defectos: (1) las
tablas se deben construir para cada m odulo primo y hay (p 1) posibles bases;
(2) los datos en las tablas no estan en orden ascendente.
El siguiente teorema establece algunas formulas utiles para el c alculo de ndices.
Teorema 5.6 Sea b una raz primitiva m odulo m.
(i) Si mcd(a, m) = 1, entonces Ind
b
(a
1
) = (m) Ind
b
(a)
(ii) Si m 3, Ind
b
(m1) = (m)/2
(iii) Si p es primo impar, Ind
b
(p1) = (p1)/2
(iv) Si m 3 y mcd(a, m) = 1, entonces Ind
b
(m1) = Ind
b
(a) +(m)/2
(v) Si m es primo impar y mcd(a, m) =1, entonces Ind
b
(m1) =Ind
b
(a)+(m
1)/2
EJEMPLO 5.15 Podemos usar la parte (v) del teorema (5.6) para construir una tabla
para logaritmo discreto en base 3 m odulo 7,
a 1 2 3 4 5 6
Ind
3
(a) 6 2 1 . . .
Tabla 5.4 Logaritmos discreto base b = 3 m odulo 7
EJERCICIOS
5.1 Muestre que en Z
7
, las unicas races primitivas son 3 y 5.
5.2 Calcule las races primitivas de m odulo 71.
5.3 Muestre que si b es raz primitiva m odulo p y b c (mod p), entonces c es
raz primitiva m odulo p.
5.4 Sea p = 2
n
+ 1. Verique que si p es primo, entonces n es par Ayuda:
Solamente puede pasar p 2 (mod 3). Ahora use logaritmo discreto.
5.5 Verique que no hay races primitivas m odulo 8.
5.6 Verique que 8 no es raz primitiva m odulo 13.
EJERCICIOS 85
5.7 Hay races primitivas en Z
12
?
5.8 Calcule las races primitivas de Z
11
5.9 Si p es primo y t un entero, muestre que si en Z
p
hay elementos de orden t,
entonces hay exactamente (t) elementos de este orden.
5.10 Sea p primo y Ord
p
(a) =t. Muestre que si b Z
p
y b
t
1 (mod p), entonces
b debe ser una potencia de a.
5.11 Construya una tabla para el logaritmo discreto en base 11
5.12 Resolver
a) 7x 13 (mod 18)
b) 2x
4
5 (mod 13)
c) 8
5x
5 (mod 13)
d) 3
4x+1
10 (mod 19)
e) 8x
2
2 (mod 13)
5.13 Sea m = m
1
m
2
con mcd(m
1
, m
2
) = 1 y m
i
3.
a) Muestre que si m 3, (m) es par
b) Sea n =(m
1
)(m
2
)/2 y mcd(a, m) = mcd(a, m
1
) = mcd(a, m
2
) =1. Muestre
que a
(m
1
)(m
2
)/2
1 (mod m
1
) y que a
(m
2
)(m
1
)/2
1 (mod m
2
)
c) Muestre que a
n
1 (mod m)
5.14 Usar logaritmo discreto para encontrar el residuo de dividir 23
1001
por 13.
6
RESIDUOS
CUADR

ATICOS
6.1 Congruencias cuadraticas modulo m
Denici on 6.1 Una congruencia cuadratica es una congruencia de la forma
x
2
a (mod m)
donde mcd(a, n) =1. Si la congruencia tiene soluci on, a es llamado residuo cuadratico
m odulo m.
EJEMPLO 6.1 Calcular los residuos cuadraticos m odulo 7.
Solucion: Una manera directa es hacer una tabla de cuadrados,
a 1 2 3 4 5 6
a
2
mod 7 1 4 2 2 4 1 residuos cuadraticos
Tabla 6.1 Residuos cuadr aticos m odulo 7
Tenemos,
1
2
= 6
2
1 mod 7
2
2
= 5
2
4 mod 7
3
2
= 4
2
2 mod 7
= 1, 2, 4 son los residuos cuadraticos mod 7.
La congruencia Ax
2
+Bx +C 0 (mod p)
Observemos que si p es primo y p A; la congruencia Ax
2
+Bx +C 0 (mod p) es
equivalente a (2Ax +B)
2
B
2
4AC (mod p) (ver ejercicios), o lo que es lo mismo,
u
2
a (mod p) con u = 2Ax +B y a = B
2
4AC.
Representaci on sim etrica.
En principio podemos decidir si la congruencia x
2
a (mod p) tiene soluci on o
no, por ensayo y error. La teora que sigue esta orientada a buscar respuestas a
preguntas como cuando es soluble o no, esta congruencia?, si es soluble, cuantas
soluciones tiene m odulo p?. La teora requiere trabajar con la representaci on simetrica
de Z
p
.
Z
p
=
_

_
_

p1
2
, ..., 1, 0, 1, ...
p1
2
_
si p es impar
_

p1
2
+1, ..., 1, 0, 1, ...
p1
2
_
si p es par
En las aplicaciones, el caso com un es cuando p es impar. La regla de conversion es
sencilla:
_

_
i i si 0 i
p1
2
,
p1
2
+k
p1
2
+k 1 con 1 k
p1
2
.
EJEMPLO 6.2 Consideremos Z
7
,
p1
2
= 3.
0, 1, 2, 3, 4, 5, 6

0, 1, 2, 3, 3, 2, 1
Si volvemos a calcular como en el ejemplo (6.1), pero esta vez usando la representaci on
simetrica, se nos har a evidente nuestro siguiente teorema,
a 3 2 1 1 2 3
a
2
mod 7 2 4 1 1 4 2 residuos cuadraticos mod 7
Tabla 6.2 Residuos cuadr aticos m odulo 7
Tenemos,
(1)
2
= 1
2
1 mod 7
(2)
2
= 2
2
4 mod 7
(3)
2
= 3
2
2 mod 7
Teorema 6.1 Sea p primo impar y mcd(a, p) = 1.
(i.) x
2
a (mod p) no tiene soluci on o tiene exactamente dos soluciones mod p,
Introducci on a la Teora de N umeros.. Walter Mora F.
Derechos Reservados 2009 Revista digital Matem atica, Educacin e Internet (www.cidse.itcr.ac.cr/revistamate/)
87
88 RESIDUOS CUADR

ATICOS
(ii.) Hay exactamente (p1)/2 residuos cuadraticos
1
2
, 2
2
, ..., ((p1)/2)
2
,
y (p1)/2 residuos no cuadraticos.
Prueba: (i.) Si x
2
a (mod p) y y
2
a (mod p) entonces p|x
2
y
2
= p|x +y
p|x y = x y (mod p).
(ii.) Sea Z
p
= {(p 1)/2, ..., 1, 0, 1, ..., (p 1)/2}. Entonces tenemos (p 1)/2
residuos cuadraticos a
j
con (j)
2
= j
2
a
j
(mod p), (p 1)/2 j (p 1)/2.
Claramente son residuos cuadraticos distintos puesto que i j (mod p) si |i| =| j|
e i, j {(p 1)/2, ..., 1, 0, 1, ..., (p 1)/2} (pues 0 < |i j| < i + j < p 1). Hay
exactamente (p1)/2 residuos cuadraticos pues ya agotamos los cuadrados en Z
p
.
En el caso p = 2 el teorema no aplica: s olo hay un residuo cuadratico m odulo 2 :
1
2
1 (mod 2).
6.2 Criterio de Euler
Euler diviso un criterio sencillo, para decidir si un n umero es residuos cuadratico
m odulo p. El criterio no es muy practico computacionalmente pero si de gran valor
te orico.
La idea es la siguiente: Si a es residuo cuadratico m odulo p, hay un entero x tal
que a
p
x
2
, entonces, por el teorema peque no de Fermat se tiene
a
(p1)/2
(x
2
)
(p1)/2
x
p1
1 (mod p),
es decir, si a es residuo cuadratico m odulo p,
a
(p1)/2
1 (mod p).
EJEMPLO 6.3 Sea p = 11. Vamos a calcular todas las potencias a
(p1)/2
m odulo p
en representaci on estandar y en representaci on simetrica,
a 1 2 3 4 5 6 7 8 9 10
a
(p1)/2
mod p 1 10 1 1 1 10 10 10 1 10 estandar
a
(p1)/2
mod p 1 1 1 1 1 1 1 1 1 1 simetrica
Tabla 6.3 Residuos cuadr aticos y no cuadr aticos m odulo p = 11
Introducci on a la Teora de N umeros.. Walter Mora F.
Derechos Reservados 2009 Revista digital Matem atica, Educacin e Internet (www.cidse.itcr.ac.cr/revistamate/)
89
Teorema 6.2 (Criterio de Euler) Sea p primo impar y mcd(a, p) = 1, entonces
(1) a es residuo cuadratico a
(p1)/2
1 (mod p),
(2) a no es residuo cuadratico a
(p1)/2
1 (mod p),
Prueba: Para la demostracion usamos el teorema peque no de Fermat y logaritmo
discreto.
Parte (1).
: Si a es un residuo cuadratico m odulo p, existe x Z tal que x
2
a (mod p.)
Ahora aplicamos el teorema peque no de Fermat,
1 x
p1
mod p
(x
2
)
(p1)/2
mod p
a
(p1)/2
mod p
: Si a
(p1)/2
1 (mod p), sea b una raz primitiva de p y sea t Z tal que
a b
t
(mod p). Entonces,
b
t
a mod p
= b
t(p1)/2
a
(p1)/2
1 mod p
= Ind
b
(b
t(p1)/2
) Ind
b
(1) 0 mod p
= t(p1)/2 0 mod (p1)
= t(p1) = 2k(p1), i.e. t es par.
=
_
b
t/2
_
2
b
t
a (mod p), i.e. a es residuo cuadratico m odulo p.
Parte (2).
Para probar esta parte se suciente observar que
(a
(p1)/2
1)(a
(p1)/2
+1) = a
(p1)/2
1 0 (mod p).
As, si a no es residuo cuadratico m odulo p, la unica opcion que queda es a
(p1)/2

1 (mod p). La otra implicaci on es consecuencia de la parte (1).


El criterio de Euler, en su version cruda, es util en el c alculo directo si p es peque no,
dado que tenemos que calcular la potencia a
(p1)/2
.
EJEMPLO 6.4 Es a = 72 residuo cuadratico m odulo 229?
90 RESIDUOS CUADR

ATICOS
Solucion: Tenemos que calcular 72
114
mod 229. Para simplicar el c alculo descomponemos
en potencias de 2,
72
114
= 72
2
((((72
2
)
2
)
2
)
2
)
7
228 1 mod 229; no es residuo cuadratico.
EJEMPLO 6.5 Sea p primo impar. Muestre que si b es raz primitiva m odulo p,
entonces b no es residuo cuadratico m odulo p.
Solucion: Como Ord
p
(b) = p1, entonces b
(p1)/2
1 (mod p) y por el criterio de
Euler, la unica posibilidad es que b
(p1)/2
1 (mod p), es decir, b no es residuo
cuadratico m odulo p.
6.3 S

imbolos de Legendre y Jacobi


El smbolo de Legendre nos permite establecer si un n umero a es o no es residuo
cuadratico m odulo un primo p, mediante un c alculo automatico. La ley de la
reciprocidad cuadratica, una de las joyas de la teora de n umeros, simplica notablemente
este c alculo.
El smbolo de Jacobi es una generalizaci on del smbolo de Legendre que permite
una simplicacion del c alculo cuando el m odulo no es primo.
Los estudios en residuos cuadraticos de Euler fueron extendidos por Legendre. El
smbolo de Legendre nos proporciona una serie de reglas para el c alculo automatico.
Estas reglas en el fondo, son aplicaciones simplicadas del criterio de Euler.
Denici on 6.2 Sea p un primo impar y mcd(a, p) = 1. El smbolo de Legendre
_
a
p
_
es denido por,
_
a
p
_
=
_
_
_
1 si a residuo cuadratico m odulo p
1 si a no es residuo cuadratico m odulo p
En algunos textos se usa una denicion alternativa: Si p es primo impar,
_
a
p
_
=
_
_
_
1 si a residuo cuadratico m odulo p
0 si p|a
1 si a no es residuo cuadratico m odulo p
Introducci on a la Teora de N umeros.. Walter Mora F.
Derechos Reservados 2009 Revista digital Matem atica, Educacin e Internet (www.cidse.itcr.ac.cr/revistamate/)
91
EJEMPLO 6.6 Los residuos cuadraticos de Z
7
son 1, 2, 4, es decir,
_
1
7
_
=
_
2
7
_
=
_
4
7
_
= 1 y
_
3
7
_
=
_
5
7
_
=
_
6
7
_
=1
Para el c alculo del smbolo de Legendre es necesario establecer las siguientes propiedades,
Teorema 6.3 Sea p primo impar y mcd(p, a) = mod(p, b) = 1. Entonces,
(1)
_
a
p
_
= a
(p1)/2
mod p (en representaci on simetrica).
(2)
_
a
2
p
_
= 1. En particular
_
1
p
_
= 1.
(3) Si a b (mod p) entonces
_
a
p
_
=
_
b
p
_
, En particular,
_
a
p
_
=
_
a mod p
p
_
.
(4)
_
ab
p
_
=
_
a
p
__
b
p
_
.
(5)
_
1
p
_
= (1)
(p1)/2
=
_
1 si p 1 (mod 4)
1 si p 1 (mod 4)
Prueba: El item (1) es el criterio de Euler.
(2): Si a

= a
2
, a
2
a

(mod p), es decir,


_
a
2
p
_
= 1.
(3): a b (mod p) = a
(p1)/2
b
(p1)/2
(mod p). Luego, por (1),
_
a
p
_
=
_
b
p
_
.
(4): Aplicamos (1),
_
ab
p
_
(ab)
(p1)/2
mod p
a
(p1)/2
b
(p1)/2
mod p

_
a
p
__
b
p
_
(5): Por (1)
_
1
p
_
(1)
(p1)/2
mod p
92 RESIDUOS CUADR

ATICOS
Ahora, para demostrar la segunda igualdad, notemos que, por el algoritmo de la
divisi on, p 1 (mod 4) o p 3 (mod p). Si p = 4k +1 para alg un entero k, (p
1)/2 = 2k = par. Si p = 4k 1 para alg un entero k, (p 1)/2 = 2k 1 = impar.
Por lo tanto,
_
1
p
_
=
_
1 si p 1 (mod 4)
1 si p 1 (mod 4)
Corolario 6.1 Sea p primo impar,
(1) Si n =
k
i=1
p

i
i
es la descomposici on prima de n,
_

k
i=1
p

i
i
p
_
=
k

i=1
_
p
i
p
_

i
(2) El producto de dos residuos cuadraticos m odulo p es residuo cuadratico
m odulo p.
(3) El producto de dos residuos no cuadraticos m odulo p es residuo cuadratico
m odulo p.
(4) El producto de un residuo cuadratico y otro no cuadratico m odulo p, es un
residuo no cuadratico m odulo p.
Prueba: Ejercicio.
EJEMPLO 6.7
_
2
5
_
= 2
(51)/2
mod 5 = 4 mod 5 =1 por el teorema (6.3), (1).
EJEMPLO 6.8 El criterio de Euler, bajo el smbolo de Legendre, nos da un criterio
rapido para decidir si a =1 es o no es residuo cuadratico m odulo p.
a =1 no es residuo cuadratico m odulo 3 pues
_
1
3
_
= (1)
(31)/2
=1
a =1 es residuo cuadratico m odulo 229 pues
_
1
229
_
=(1)
(2291)/2
=(1)
114
=
1.
EJEMPLO 6.9 En este ejemplo vamos a aver como se aplican algunas de las propiedades
del smbolo de Legendre.
93
(1) Es 10 residuo cuadratico del primo 3?
Solucion:
_
10
3
_
=
_
1
3
__
10
3
_
=
_
1
3
_
= 1 por el teorema (6.3), (3).
(2) Es 72 residuo cuadratico del primo 229?
Solucion: Por el corolario (6.1),
_
72
229
_
=
_
2
3
3
2
229
_
=
_
2
229
_
3
_
3
229
_
2
= 1
_
2
229
_
1 =1
(3) Es 63 residuo cuadratico del primo 11?
Solucion:
_
63
11
_
=
_
8
11
_
por (6.3), (3)
=
_
2
11
__
2
2
11
_
por (6.3), (4)
=
_
2
11
_
1 por (6.3), (1)
= 1 1 por c alculo directo.
por tanto 63 no es residuo cuadratico m odulo 11.
EJEMPLO 6.10 Probar que hay una cantidad innita de primos de la forma 4k +1.
Solucion: Por contradiccon, supongamos que solo hay una cantidad nita P =
{p
1
, p
2
, ..., p
s
} de primos de la forma 4k+1. Sea N =(2p
1
p
2
p
s
)
2
+1. Observemos
que si p
i
P, N = k

p
i
+1, es decir, los p
i
s no dividen N. Como N es de la forma
4k +1 y no es un p
i
, es compuesto. Entonces sera divisible por un primo p P.
Por tanto, 1 es residuo cuadratico m odulo p y p debera ser de la forma 4k +1.
Contradiccion.
94 RESIDUOS CUADR

ATICOS
6.3.1 Lema de Gauss
El lema de Gauss es una herramienta te orica que al igual que el criterio de Euler,
nos provee de metodo para calcular el smbolo de Legendre va un conteo de signos.
La idea es la siguiente: Si p es primo impar y mcd(a, p) = 1, entonces
Z
p
={0, a 1, a 2, ..., (p1) a}
Los n umeros a 1, a 2, ..., a
p1
2
son distintos m odulo p. Si consideramos Z
p
en
representaci on simetrica, tenemos
{a 1, a 2, ..., a
p1
2
}
_

p1
2
, ..., 1, 0, 1, ...,
p1
2
_
Por ejemplo, si p = 13 y a = 3, tenemos
3 i 3 1 3 2 3 3 3 4 3 5 3 6
3 i mod 13 3 6 4 1 2 5
Tabla 6.4 Representaci on sim etrica de 3 i, i = 1, ..., 6
En representaci on simetrica los n umeros aparecen con una copia positiva y otra
negativa, es decir, aparece el 1 y el 1, el 2 y el 2, etc. Pero, al pasar cada
elemento del conjunto {a i, i = 1, ..., (p 1)/2} a representaci on simetrica, solo
aparece una copia: aparece el 1 o el 1, el 2 o el 2, etc.
Ahora, sacando a factor com un a y los signos, tenemos
a
(p1)/2
1 2
p1
2
(1)

= a 1 a 2. . . a
p1
2
1 2
p1
2
mod p,
por tanto, cancelando: a
(p1)/2
(1)

(mod p). As, el n umero de signos


en {a, a 2, ..., a
p1
2
} dene si a es residuo cuadratico o no.
Lema 6.1 (Lema de Gauss) Sea p primo impar y mcd(a, p) =1. Si es la cantidad
de enteros en el conjunto
{a 1 mod p, a 2 mod p, ..., a
p1
2
mod p}
que son m as grandes que (p1)/2 (negativos en representaci on simetrica), entonces
_
a
p
_
= (1)

Prueba: : Sea R={a mod p, 2 a mod p, ...,


p1
2
a mod p}. En R no hay elementos
congruentes pues mcd(a, p) = 1. Vamos a denotar con r
1
, r
2
, ..., r
k
los elementos de
95
R que son (p1)/2 y s
1
, s
2
, ..., s

los elementos de R que son > (p1)/2. Por


tanto, k + = (p1)/2.
Los (p 1)/2 enteros r
1
, r
2
, ..., r
k
, p s
1
, p s
2
, ..., p s

son positivos y (p
1)/2. Todos estos n umeros son distintos m odulo p : En efecto, ya conocemos que
r
1
, r
2
, ..., r
k
, s
1
, s
2
, ..., s

son distintos m odulo p, como s


i
s
j
(mod p) entonces p
s
i
ps
j
(mod p). Tambien ps
i
r
j
(mod p) si i = j, 1 i, j (p1)/2; para
probarlo, supongamos que ps
i
r
j
(mod p), entonces s
i
r
j
(mod p) = s
i
+
r
j
0 (mod p) pero esto es imposible pues 0 < s
i
+r
j
(p1)/2. Esto demuestra
que
{r
1
, r
2
, ..., r
k
, ps
1
, ps
2
, ..., ps

} ={1, 2, ..., (p1)/2}.


Entonces
1 2 (p1)/2 r
1
r
2
r
k
(ps
1
) (ps
2
) (ps

) (mod p)
r
1
r
2
r
k
s
1
s
2
s

(mod p), pues p 0


sacamos los signos a factor com un,
(r
1
r
2
r
k
s
1
s
2
s

)(1)

(mod p),
(a 1 a 2 a (p1)/2)(1)

(mod p),
sacamos a a factor com un,
(a
(p1)/2
(1 2 (p1)/2)(1)

(mod p),
cancelamos,
1 a
(p1)/2
(1)

(mod p), es decir,


a
(p1)/2
(1)

(mod p).
Ahora, por el criterio de Euler,
_
a
p
_
= (1)

.
Nota: En la practica, en vez de contar los signos negativos, contamos los residuos
a i mod p > p/2. Usamos p/2 en vez de (p1)/2 pues
p1
2
<
p
2
<
p1
2
+1 =
p+1
2
.
El siguiente ejemplo ilustra el c alculo. Recordemos que la importancia del lema es
de orden te orico no computacional.
EJEMPLO 6.11 Es a = 63 residuo cuadratico m odulo p = 11?
Solucion: {63 i, i = 1, ..., 5} ={8, 5, 2, 10, 7}. Hay = 3 n umeros > p/2[ = 5. Por
tanto
_
63
11
_
= (1)
3
=1. 63 no es residuo cuadratico m odulo 11.
96 RESIDUOS CUADR

ATICOS
Ya sabemos c omo decidir si 1 es residuo cuadratico m odulo p. Podemos aplicar
el lema de Gauss para decidir si 2 es residuo cuadratico m odulo p.
Teorema 6.4 Si p es primo impar, entonces
_
2
p
_
= (1)
(p
2
1)/8
=
_
1 si p 1 (mod 8)
1 si p 3 (mod 8)
(6.1)
En particular, si a es par,
_
a
p
_
= (1)
(p
2
1)/8
_
a/2
p
_
.
Prueba: Para calcular
_
2
p
_
contamos los n umeros en {2 i, i =1, ..., (p1)/2} tales
que 2 i > p/2, es decir, i > p/4 (aqu no hay que hacer reducci on m odulo p pues
0 2i p1). Entonces 2i > p/2 si p/4[ < i (p1)/2. Por lo tanto,
=
p1
2

_
p
4
_
.
Esto nos da
_
2
p
_
= (1)
(p1)/2
_
p
4
_
Aqu lo que interesa es saber si (p 1)/2
_
p
4
_
es par o impar, as que hacemos
una reducci on m odulo 2 :
p1
2

_
p
4
_

p
2
1
8
(mod 2). (6.2)
Para probar esto ultimo usamos el hecho de que, como p es primo impar, k Z
tal que p = 8k +r con r = 1, 1, 3 o 3. Luego,
_
p
4
_
=
_

_
2k si r = 1
2k 1 si r =1
2k si r = 3
2k 1 si r =3
En estos cuatro casos se cumple (6.2) y adem as si r =1,
p
2
1
8
es par y si r =3,
p
2
1
8
es impar.
Aqu solo vamos a probar los casos r = 3 y r =1, los otros casos son similares.
Si p = 8k +3, entonces
p1
2

_
p
4
_
= 1+4k 2k = 1+4k 1 (mod 2)
97
p
2
1
8
= 1+6k +8k
2
1 (mod 2). Esto prueba (6.2) para este caso.
Como
p
2
1
8
es impar,
_
2
p
_
= (1)
(p
2
1)/8
=1 si p 3 (mod 8)
Si p = 8k 1, entonces
p1
2

_
p
4
_
= 4k 1(2k 1) = 2k 0 (mod 2)
p
2
1
8
=2k +8k
2
0 (mod 2). Esto prueba (6.2) para este caso.
Como
p
2
1
8
es par,
_
2
p
_
= (1)
(p
2
1)/8
= 1 si p 1 (mod 8).
La idea de la congruencia es muy util: es m as facil vericar la congruencia que
calcular la potencia.
EJEMPLO 6.12 (1) Es 2 residuo cuadratico m odulo 97?
Solucion: S,
_
2
97
_
= 1 pues 97 1 (mod 8)
(2) Es 2 residuo cuadratico m odulo 229?
Solucion: No,
_
2
97
_
=1 pues 229 3 (mod 8)
6.3.2 Ley de Reciprocidad Cuadr atica.
La ley de reciprocidad cuadratica establece una sorprendente relacion entre
_
p
q
_
y
_
q
p
_
. Esta ley fue conjeturada, basandose en evidencia numerica, por Euler en
1783 y Lagrange en 1785. Legendre le dio la forma actual a esta ley, pero no pudo
dar una prueba completa. La primera prueba rigurosa fue dada por Gauss en a la
edad de 18 a nos. Hasta el 2004 se conocan 190 pruebas diferentes. Gauss llamo a
este teorema Aureum Theorema (el Teorema de oro). Su importancia en la teora
de n umeros no tienen discusion. Al respecto, Hecke armo al respecto: La teora
de n umeros moderna comenz o con el descubrimiento de la Ley de Reciprocidad
Cuadr atica.
98 RESIDUOS CUADR

ATICOS
La prueba del teorema sigue es la tercera prueba que dio Gauss. La prueba se basa
e un argumento geometrico.
Primero veamos un ejemplo concreto. Sea p = 11 y q = 7. El n umero
_
4 q
p
_
= 2
cuenta la cantidad de n umeros
4 q
p
. Geometricamente corresponde a la cantidad
de pares ordenados con componentes enteros (llamados punto reticulares) sobre la
parte positiva de la recta x = 4 y por debajo de la recta y =
q
p
x. Estos puntos son
de la forma (4, y) con y
4 q
p
.
2 1
y = x
7
11
}
y
7
11
.
4
3 4 5
Y
Figura 6.1 Puntos reticulares (4, y) con y
4 11/7
2 1
y = x
q
p
3 4 (p -1)/2=5
A B
C
D
3=(q -1)/2
Figura 6.2 Puntos reticulares en ABCD
La suma
(p1)/2

k=1
_
k q
p
_
= 7 corresponde a los puntos reticulares en el polgono
ABCD de la gura (6.2).
Podemos cambiar el punto de vista y ver las cosas desde el eje Y de una manera
totalmente simetrica: El n umero
_
3 p
q
_
=4 cuenta la cantidad de n umeros
3 p
q
.
Geometricamente corresponde a la cantidad de de puntos reticulares sobre la parte
derecha de la recta y = 3 y por debajo de la recta x =
p
q
y. Estos puntos son de la
forma (x, 3) con x
3 p
q
.
La suma
(q1)/2

k=1
_
k p
q
_
= 8 corresponde a los puntos reticulares en el polgono
APQR de la gura (6.3).
99
2 1
x = y
p
q
3 4 (p -1)/2=5
A
Q
C
P
3=(q -1)/2
R
Figura 6.3 Puntos reticulares en ABCD
Finalmente, la gura (6.3) tambien nos sugiere que
(p1)/2

k=1
_
k q
p
_
+
(q1)/2

k=1
_
k p
q
_
= 7+8 = 15 =
p1
2

q1
2
Observemos adem as, que si p >q entonces
_
q
p
p1
2
_
=
q1
2
. La prueba se puede
hacer de manera directa y esta en los ejercicios. Note que, por simetra, si q > p,
entonces
_
p
q
q1
2
_
=
p1
2
.
Ahora que hemos establecido el signicado geometrico de estas sumas, consideremos
el siguiente lema,
Lema 6.2 Sean p, q primos impares distintos. Entonces,
(p1)/2

k=1
_
k q
p
_
+
(q1)/2

k=1
_
k p
q
_
=
p1
2

q1
2
Prueba: Sea p > q. El n umero k q/p[ cuenta la cantidad de n umeros (k q)/p.
Geometricamente corresponde a la cantidad de puntos reticulares sobre la parte
positiva de la recta x = k y por debajo de la recta y = q/px. Estos puntos son de
la forma (k, y) con y k q/p.
Cuando x = (p1)/2, la cantidad de puntos es y = (q1)/2 pues
q1
2

q
p
p1
2
<
q1
2
+1.
La suma S(p, q) =
(p1)/2

k=1
k q/p[ corresponde a la suma de los puntos reticulares
en el polgonoABCD de la gura (6.4).
100 RESIDUOS CUADR

ATICOS
x = y,
q
p
(p -1)/2
A
P
C
R
(q -1)/2
p>q
. . .
. . .
.

.

.
.

.

.
. . .
.

.

.
B
D Q
Figura 6.4 Puntos reticulares en ABCD
De manera simetrica, la suma S(q, p) =
(q1)/2

k=1
k p/q[ corresponde a la suma de
los puntos reticulares en el polgonoAPQR de la gura (6.4).
Hay que notar que no hay puntos reticulares sobre la recta y =
q
p
x pues mcd(p, q) =
1.
Finalmente, la suma de los puntos reticulares en el polgono ABCR se puede expresar
de dos formas:
p1
2

q1
2
y S(q, p) +S(p, q).
Teorema 6.5 (Ley de Reciprocidad Cuadr atica) Sea p y q primos impares distintos.
Entonces
_
p
q
__
q
p
_
= (1)
p1
2
q1
2
En particular,
_
p
q
_
= (1)
p1
2
q1
2
_
q
p
_
.
Prueba: Sea R = {q mod p, 2 q mod p, ...,
p1
2
q mod p}. Vamos a denotar con
r
1
, r
2
, ..., r
k
los elementos de R que son p/2 y s
1
, s
2
, ..., s

los elementos de R que


son > p/2. Claramente k + = (p1)/2 y
_
q
p
_
= (1)

.
Los elementos de R son todos distintos. Si i, j R y i = j, entonces i p
j (mod p). As, los (p1)/2 n umeros r
1
, r
2
, ..., r
k
, ps
1
, ps
2
, ..., ps

son todos
distintos e inferiores a p/2, por tanto estos n umeros corresponden a los n umeros
1, 2, ..., (p1)/2 en alg un orden. Entonces
101
k

i=1
r
k
+

i=1
(ps
i
) =
(p1)/2

i=1
i
=
(p1)(p+1)
8
Por tanto,
k

i=1
r
k
+

i=1
(ps
i
) =
k

i=1
r
k
+ p

i=1
s
i
=
p
2
1
8
Sea S(p, q) =
(p1)/2
k=1
k q/p[, S
1
=
k
i=1
r
k
y S
2
=

i=1
s
i
. Con esta notacion,
p
2
1
8
= S
1
+ pS
2
(6.3)
Por el algoritmo de la divisi on,
kq =[kq/p_ p+t
k
con 0 t
k
< p.
Entonces,
(p1)/2

k=1
k q =
(p1)/2

k=1
[kq/p_ p +
(p1)/2

k=1
t
k
.
Esto es,
q
(p1)/2

k=1
k = p S(p, q) +S
1
+S
2
q
p
2
1
8
= p S(p, q) +S
1
+S
2
(6.4)
Ahora restando (6.3) con (6.4) obtenemos,
(q1)
p
2
1
8
= p (S(p, q) ) +2S
2
De aqu se sigue que S(p, q) es par. Por tanto, (1)
S(p,q)
= 1; es decir
(1)
S(p,q)
=(1)

. Pero, el lema de Gauss dice que


_
q
p
_
=(1)

, entonces
_
q
p
_
=
(1)
S(p,q)
. De manera similar,
_
p
q
_
= (1)
S(q,p)
. En conclusion,
_
p
q
__
q
p
_
= (1)
S(q,p)
(1)
S(p,q)
= (1)
p1
2

q1
2
, por el lema de Gauss.
102 RESIDUOS CUADR

ATICOS
El corolario que sigue es una reformulacion de la Ley de Reciprocidad Cuadr atica
en terminos de congruencias.
Corolario 6.2 Sea p y q primos impares distintos. Entonces
_
q
p
_
=
_

_
_
p
q
_
si p 1 (mod 4) o q 1 (mod 4)
_
p
q
_
si p q 3 (mod 4)
Prueba: Ejercicio.
EJEMPLO 6.13 Es 152 residuo cuadratico m odulo 43?
Solucion: 152 23 (mod 43), entonces
_
153
43
_
=
_
23
43
_
=
_
43
23
_
, pues 43 23 3 (mod 4), (Corolario 6.2)
=
_
20
23
_
, pues 43 20 (mod 23)
=
_
2
2
23
_

_
5
23
_
=
_
5
23
_
=
_
23
5
_
pues 5 1 (mod 4)
=
_
2
5
_
=1 pues
_
2
5
_
= (1)
(251)/8
= 1.
EJEMPLO 6.14 Muestre que si p = 2
n
+1 es primo =3 es raz primitiva m odulo
p.
Solucion: p 1 (mod 3) y, como p es primo, p 0 (mod 3). As, p 2 (mod 3).
Por tanto,
_
p
3
_
=
_
2
3
_
=1. Ahora, por el criterio de Euler, 3
2
n1
1 (mod p)
y tambien 3
2
n
1 (mod p). Sea ahora Ord
p
(3) = s, entonces s|2
n
s = 2
k
con
k n. Si k < n, entonces
103
1 =
_
3
2
k
_
2
nk1
= 3
2
n1
1 mod p,
lo cual es una contradiccion. 3 es raz primitiva.
6.4 Smbolo de Jacobi.
El smbolo de Jacobi es una extensi on del smbolo de Legendre. La notacion es la
misma, el smbolo de Jacobi se denota
_
a
m
_
solo que esta vez m debe ser impar y
mcd(a, m) = 1. No hay peligro de confusi on pues si m es primo impar, el smbolo
de Jacobi coincide con el smbolo de Legendre. Si m no es primo, estamos en el
contexto del smbolo de Jacobi.
Denici on 6.3 (Smbolo de jacobi) Sea m entero positivo impar con descomposici on
prima m =
k

i=1
p
e
i
i
, y sea a entero tal que mcd(a, m) = 1. El smbolo de Jacobi se
dene por
_
a
m
_
=
_
a

k
i=1
p
e
i
i
_
=
k

i=1
_
a
p
i
_
e
i
En esta denicion, formalmente
_
a
p
i
_
corresponde al smbolo de Legendre.
EJEMPLO 6.15
_
2
15
_
=
_
2
3
__
2
5
_
= (1)(1) = 1.
Si m es primo, el smbolo de Jacobi coincide con el smbolo de Legendre.
Si m no es primo, el smbolo de Jacobi no decide si a es residuo cuadratico
m odulo m : 2 no es residuo cuadratico m odulo 15 pero, usando el smbolo de
Jacobi,
_
2
15
_
= 1.
El smbolo de Jacobi si decide residuos no cuadraticos. Si
_
a
m
_
= 1, a no
es residuo cuadratico m odulo m. Esto es as pues si
_
a
m
_
=1, entonces por
denicion, si m es compuesto, hay un divisor primo impar p
i
de m tal que
_
a
p
i
_
=1. Si suponemos que a es residuo cuadratico m odulo m tendramos
Introducci on a la Teora de N umeros.. Walter Mora F.
Derechos Reservados 2009 Revista digital Matem atica, Educacin e Internet (www.cidse.itcr.ac.cr/revistamate/)
104 RESIDUOS CUADR

ATICOS
una contradiccion pues x
2
a (mod m) = x
2
a (mod p
i
).
El smbolo de Jacobi simplica el c alculo del smbolo de Legendre cuando a
es compuesto impar y p primo, como veremos m as adelante.
Teorema 6.6 (Propiedades del smbolo de Jacobi) Sea m un entero positivo impar,
a, b, n enteros con mcd(a, m) = mcd(b, m) = 1 entonces,
(1)
_
a
m
_
=
_
a mod m
m
_
(2)
_
ab
m
_
=
_
a
m
_
_
b
m
_
(3)
_
a
2
m
_
= 1. En particular,
_
1
m
_
= 1
(4)
_
1
m
_
= (1)
(m1)/2
(5)
_
2
m
_
= (1)
(m
2
1)/8
(6) Ley generalizada de reciprocidad cuadratica.
_
n
m
_
=
_
m
n
_
(1)
(m1)(n1)/4
si
tambien n es impar y mcd(m, n) = 1
EJEMPLO 6.16
_
391
439
_
corresponde al smbolo de Legendre pues 439 es primo.
Como mcd(391, 439) =1, podemos usar la ley generalizada de reciprocidad cuadratica
calculando como smbolo de Jacobi.
_
391
439
_
= (1)
(4391)(3911)/4
_
439
391
_
(Reciprocidad cuadratica generalizada)
= 1
_
439
391
_
=
_
439 mod 391
391
_
=
_
48
391
_
=
_
4
2
391
__
3
391
_
=
_
3
391
_
=(1)
(3911)(31)/4
_
391
3
_
=
_
1
3
_
= 1
EJERCICIOS 105
EJERCICIOS
6.1 Calcule los residuos cuadraticos m odulo 9.
6.2 Muestre que si p es primo impar,
p1

a=1
_
a
p
_
= 0
6.3 Use el smbolo de Jacobi para vericar si 48 no es residuo cuadratico m odulo
391.
6.4 Use el smbolo de Legendre para vericar que si q es el m as peque no residuo
no cuadratico m odulo p (primo impar), entonces q debe ser primo.
6.5 Muestre que
_
1
p
_
= 1 p 1 (mod 4).
6.6 Sea p es primo impar. Muestre que p 1 es residuo cuadratico m odulo p,
si y solo si p 1 (mod 4). Ayuda: Verique que si x
2
p 1 (mod p), entonces
x
2
1 (mod p).
6.7 Sea p es primo impar y
_
a
p
_
= 1. Muestre que pa es residuo cuadratico
m odulo p, si y solo si p 1 (mod 4). Solucion: Veamos que a es residuo cuadratico
si y solo si pa es residuo cuadratico, pues x
2
a (mod p) x
2
pa (mod p).
Ahora vamos a probar, usando el smbolo de Legendre, que a es residuo cuadratico
m odulo p si y solo si 1 es residuo cuadratico m odulo p ( p 1 (mod 4)).
_
a
p
_
= 1
_
1
p
__
a
p
_
=
_
1
p
_

_
a
p
_
=
_
1
p
_
.
As , p a es residuo cuadratico m odulo p si y solo si 1 es residuo cuadratico
m odulo p, es decir, si y solo si p 1 (mod 4). .
6.8 Muestre que
_
a
5
_
= 1 si a 1 (mod 5), y
_
a
5
_
= 1 si a 2 (mod 5).
Ayuda: reciprocidad cuadratica y reducci on m odulo 5.
6.9 Sea n >1. Muestre que si p es factor primo de n
2
+1, entonces p 1 (mod 4).
6.10 Sea p es primo impar y
_
a
p
_
= 1. Muestre que p a es no es residuo
cuadratico m odulo p, si y solo si p 3 (mod 4).
6.11 Sea p primo impar. Muestre que si
_
a
p
_
= 1, entonces el inverso de a es
residuo cuadratico m odulo p.
6.12 Sea p primo y p A. Si Ax
2
+Bx+C 0 (mod p), muestre que (2Ax+B)
2

B
2
4AC (mod p) Ayuda: En Ax
2
+Bx+C 0 (mod p) multiplique por 4A y agrupe.
6.13 Resolver la congruencia 3x
2
4x +7 0 (mod 13)
6.14 Muestre que 3x
2
+7x +5 0 (mod 13) no tiene soluci on.
106 RESIDUOS CUADR

ATICOS
6.15 Si p es primo impar, probar que
p1
2

_
p
4
_

p
2
1
8
(mod 2) para los
casos p = 8k +1, p = 8k 3.
6.16 Sea p primo impar, mcd(a, p) = 1 y b raz primitiva m odulo p. Sea a
b
s
(mod p). Muestre que si s es par, entonces a es residuo cuadratico; sino, a no
es residuo cuadratico.
6.17 Usar el criterio de Euler para determinar si a = 54 es residuo cuadratico
m odulo p = 97.
6.18 Que puede decir de
_
a
2
_
?
6.19 Es 2 residuo cuadratico m odulo 3181?
6.20 Sean p >q ambos primos impares. Muestre que
_
q
p
p1
2
_
=
q1
2
. Ayuda:
Muestre que
q1
2

q
p
p1
2
<
q1
2
+1.
6.21 Es 3797 residuo cuadratico m odulo 7297?
6.22
_
1
17
_
=
_
1
17
_
?
6.23 Sea p primo > 3. Muestre que si p 1 (mod 4), entonces
_
3
p
_
= 1 si
p 1 (mod 3) y que si p 3 (mod 4), entonces
_
3
p
_
= 1 si p 2 (mod 3).
Ayuda: Ley de Reciprocidad Cuadr atica.
6.24 Sea p primo impar.
_
3
p
_
= 1 si y solo si p 1 (mod 12).
6.25 Sea p primo impar. Muestre que la congruencia x
2
+3 0 (mod p) tiene
soluci on si y solo si p es un primo de la forma 3h+1.
6.26 Probar que x
(p1)/2
1 (mod p) tiene (p1)/2 soluciones m odulo p.
6.27 Sea p
1
, ..., p
s
primos de la forma 8k +7 y sea N = (4p
1
p
2
p
s
)
2
2.
a) Probar, usando residuos cuadraticos, que los divisores primos impares de
N tienen la forma 8k +1 o 8k +7.
b) Probar que no todos los divisores primos impares de N tienen la forma
8k +1
c) Probar que hay innitos primos de la forma 8k +7.
7
ESTIMACIONES,
ESTAD

ISTICAS Y
PROMEDIOS
El prop osito de este captulo es estudiar comportamiento tpico de algunas funciones
aritmeticas para tomar decisiones a la hora de crear heursticas para resolver alg un
problema. Por ejemplo, nos interesa tener estimaciones para poder responder preguntas
tales como: La mayora de los n umeros tienen factores primos peque nos?, Los
n umeros altamente compuestos son escasos?, Cu al es el tama no tpico del factor
m as grande de un n umero?, etc.
7.1 Funciones Aritmeticas
La funci on aritmetica (n) cuenta cu antos divisores positivos, primos y compuestos,
tiene n. Por ejemplo, los divisores de 8 son 1, 2, 4; as (8) = 4. La funci on (n) es
denida como la suma de los divisores, tanto primos como compuestos, de n. Por
ejemplo, (8) = 1+2+4 = 7. Formalmente,
(n) =

d|n
1 y (n) =

d|n
1
Sea s(n) = (n) n, es decir, s(n) es la suma de divisores propios de n (a veces se
usa el arcasmo parte alcuota en vez de divisor propio). Decimos que un entero
n es deciente si s(n) > n, que abundante si s(n) > n y que es perfecto si s(n) = n.
Por ejemplo, s(8) = 7 < 8, as que 8 es deciente. s(12) = 16, as 12 es abundante.
Los divisores de 6 son 1, 2, 3 y 6, por tanto s(6) = 1+2+3+6 6 = 6, por tanto
6 es perfecto.
y se calculan facil si n es potencia de un primo.
Lema 7.1 Sea p primo y n = p
k
, , entonces (n) =k+1 y (n) =(p
k+1
1)/(p1)
Prueba: Los divisores de d de p
k
son 1, p, p
2
, ..., p
k
. Hay k +1 divisores. La suma
es
1+ p+ p
2
+... + p
k
=
1p
k+1
1p
=
p
k+1
1
p1
Introducci on a la Teora de N umeros.. Walter Mora F.
Derechos Reservados 2009 Revista digital Matem atica, Educacin e Internet (www.cidse.itcr.ac.cr/revistamate/)
107
108 ESTIMACIONES, ESTAD

ISTICAS Y PROMEDIOS
y son multiplicativas, esto nos permite calcular y si conocemos la descomposici on
prima de n.
Lema 7.2 Si mcd(m, n) = 1, entonces
(nm) = (n)(m) y (nm) = (n)(m)
Prueba: La a idea la podemos ver con un ejemplo: Sea n = 9 y n = 4, ambos son
primos relativos. Ahora, hacemos un arreglo rectangular como el que esta a la
izquierda de la tabla (7.1). Las unicas las en consideraci on son las las que inician
con un divisor de 9. Luego, solo marcamos las entradas
5
d
i
d
j
si d
i
|9 y d
j
|4.
Simplicando, lo que nos queda es un arreglo rectangular (9)(4).
1 2 3 4
1 1 1 1 2 1 4
2
3 3 1 3 2 3 4
4
5
6
7
8
9 9 1 9 2 9 4

1 2 4
1 1 1 1 2 1 4
3 3 1 3 2 3 4
9 9 1 9 2 9 4
Tabla 7.1 Si mcd(9, 4) = 1, entonces (9 4) = (9)(4)
La prueba para (nm) es una modicaci on de la prueba de (m)(n). Solo necesitamos
notar que (nm) es la suma de todas las entradas de la tabla simplicada.
La prueba formal queda como ejercicio.
Teorema 7.1 Si la factorizaci on prima de n es p
k
1
1
p
k
2
2
p
k
s
s
, entonces
(n) = (k
1
+1)(k
2
+2) (k
s
+1) y (n) =
s

i=1
p
k
i
i
1
p
i
1
Prueba: Como p
k
1
1
, p
k
2
2
, , p
k
s
s
son primos relativos dos a dos, entonces
(n) =
_
p
k
1
1
_

_
p
k
2
2
_

_
p
k
s
s
_
= (k
1
+1)(k
2
+2) (k
s
+1)
y similarmente para (n)
y son ejemplos de funciones denidas sobre los n umeros naturales. En vez de
considerar este tipo de funciones como objetos aislados, es de mucha ayuda verlas
5
Recordemos que si mcd(m, n) = 1 y si d|mn, entonces d = ab con a|m y b|n.
109
como objetos m as generales y estudiar la relacion entre ellas por medio de una
operacion (llamada convolucion).
Una funci on f denida sobre los n umeros naturales, se llama funci on aritmetica.
Por ejemplo,
u(n) = 1 para todo n,
N(n) = n para todo n,
e(n) =
_
1 si n = 1
0 si n > 1
Denici on 7.1 (Convoluci on) Sean f y g funciones aritmeticas. La convolucion de
f y g, se denotada f g, es una funci on aritmetica denida por
f g(n) =

d|n
f (d)g(n/d)
Como los divisores de n ocurren en pares (es decir, si d|n = n = dk y entonces
(n/d)|n), podemos escribir
f g(n) =

c,d
n=cd
f (d)g(c)
EJEMPLO 7.1 Calcule Nu
Solucion: Nu(n) =

c,d
n=cd
N(d)u(c) =

d|n
d 1 = (n)
EJEMPLO 7.2 Calcule uu
Solucion: uu(n) =

c,d
n=cd
u(d)u(c) =

d|n
1 = (n)
Teorema 7.2 Sean f , g y h funciones aritmeticas, entonces
(1) f g = g f
(2)( f g) h = f (gh)
(3) f e = f para cualquier funci on aritmetica f
Prueba: Ejercicio.
La funci on de Mobius se dene as:
(1) = 1,
si n > 1 tiene factorizaci on prima n = p
k
1
1
p
k
2
2
p
k
s
s
, entonces
(n) =
_
0 si k
i
> 1, para alg un i = 1, 2, ..., s
(1)
s
si k
i
= 1, para todo i = 1, 2, ..., s
110 ESTIMACIONES, ESTAD

ISTICAS Y PROMEDIOS
As, por ejemplo (2 3 13) = (1)
3
=1 mientras que (2 3
2
13) = 0.
La funci on de Mobius es importante porque f = g u g = f . Esta es una
formula muy util y se le llama formula de inversion de Mobius.
Lema 7.3 u = e, es decir,

d|n
(d) =
_
1 si n = 1,
0 si n > 1.
Prueba: Si n = 1,
d|1
(d) = (1) = 1. Para probar el caso n > 1, empecemos con
un ejemplo: Si n = 3 5
3
7, los divisores de n que contribuyen con un sumando
no nulo se pueden escribir en pares, 3, 3 7, 3 5, 3 5 7, 5, 5 7. Los divisores se
dividen en dos grupos de igual cardinalidad, los que son divisibles por 7 y los que
no. Si d es divisor del primer grupo, d 7 es divisor del segundo grupo. Observemos
que (d) = (d 7), por tanto la suma cancela:
d|n
(d) = (3) +(3 7) +... =
1+1+111+1 = 0. Formalmente,
Si n > 1 tiene factorizaci on prima n = p
k
1
1
p
k
2
2
p
k
s
s
; los unicos divisores d de n
que contribuyen con un sumando no nulo, son los divisores que son productos de
primos distintos. Estos divisores d se pueden dividir en dos grupos de igual tama no;
D
1
= {d : p
s
|d} y D
2
= {d : p
s
d}, es decir, los productos que no tienen a p
s
y
estos mismos agregando p
s
: d D
1
dp
s
D
2
. Ahora, como (d) = (dp
s
),
entonces hay tantos divisores que contribuyen con 1 a la suma como divisores
que contribuyen con 1, como se quera mostrar.
Teorema 7.3 (F ormula de Inversi on) Sean f y g son funciones aritmeticas, entonces
Si f (n) =

d|n
g(d), entonces g(n) =

d|n
f (d)(n/d)
e inversamente.
Prueba: Usando la notacion de convolucion, hay que probar que f = g u g =
f .
: f = gu = f = (gu) = g(u) = g(u) = ge = g.
: Ejercicio.
EJEMPLO 7.3 Muestre que

d|n
(d)(n/d) = n para toda n Z
+
.
Solucion: Como = N u, entonces por inversion de Mobius, N = , que es lo
que se quera.
111
7.2 A los n umeros primos les gustan los juegos de azar.
La probabilidad de que un n umero natural, tomado al azar,
sea divisible por p es 1/p
Que signica tomar un n umero natural al azar?. Los naturales son un conjunto
innito, as que no tiene sentido decir que vamos a tomar un n umero al azar. Lo
que si podemos es tomar un n umero de manera aleatoria en un conjunto nito
{1, 2, ..., n} y luego (atendiendo al modelo frecuentista de probabilidad) ver que
pasa si n se hace grande (i.e. n ).
Hagamos un peque no experimento: Fijamos un n umero p y seleccionamos de manera
aleatoria un n umero en el conjunto {1, 2, ..., n} y vericamos si es divisible por p.
El experimento lo repetimos m veces y calculamos la frecuencia relativa.
En la tabla que sigue, hacemos este experimento varias veces para n, m y p.
n m p Frecuencia relativa
100000 10000 5 0.1944
0.2083
0.2053
0.1993
10000000 100000 5 0.20093
0.19946
0.1997
0.20089
100000000 1000000 5 0.199574
0.199647
Tabla 7.2
Y efectivamente, parece que la probabilidad de que un n umero tomado al azar
en el conjunto {1, 2, ..., n} sea divisible por p es 1/p
De una manera sintetica: Sea E
p
(n) = los m ultiplos de p en el conjunto {1, 2, ..., n}.
Podemos calcular la la proporci on de estos m ultiplos en este conjunto, es decir,
podemos calcular
E
p
(n)
n
para varios valores de n
112 ESTIMACIONES, ESTAD

ISTICAS Y PROMEDIOS
n M ultiplos de p = 5 Proporcion
100 20 0.2
10230 2046 0.2
100009 20001 0.199992
1000000 199999 0.199999
Tabla 7.3
Parece que en el conjunto {1, 2, ..., n}, la proporci on de los m ultiplos de p = 5 se
aproxima a 1/5, conforme n se hace grande. Signica esto que la probabilidad
de que un n umero natural, tomado al azar, sea divisible por 5 es 1/5?. Por ahora,
lo unico que podemos decir es que este experimento sugiere que la densidad (o la
proporci on) de los m ultiplos de 5 en {1, 2, ..., n} parece ser 1/5 conforme n se hace
grande. Generalizando,
Denici on 7.2 Sea E un conjunto de enteros positivos con alguna propiedad especial
y sea E(N) =E

{1, 2, ..., N}. La densidad (o medida relativa) de E se dene como
D[E] = lim
n
E(n)
n
siempre y cuando este lmite exista.
Es esta densidad una medida de probabilidad en el modelo axiomatico?. No, porque
resulta no ser aditiva, como el modelo exige (ver [10]). Aunque en el esquema
frecuentista se puede ver la densidad como la probabilidad de que un entero
positivo, escogido aleatoriamente, pertenezca a E, aqu identicamos este termino
con densidad o proporci on. Tenemos,
Teorema 7.4 La densidad de los naturales divisibles por p es
1
p
, es decir, si E
p
es
el conjunto de enteros positivos divisibles por p, entonces
D[E
p
] = lim
n
E
p
(n)
n
=
1
p
Prueba: Para calcular el lmite necesitamos una expresi on analtica para E
p
(n).
Como existen p, r tales que n = pk +r con 0 r < p, entonces kp n < (k +1) p,
es decir, hay exactamente k m ultiplos positivos de p que son menores o iguales a n.
Luego E
p
(n) =k =
nr
p
. Por lo tanto, D[E
p
] = lim
n
E
p
(n)
n
= lim
n
(nr)/p
n
= lim
n
1
p

r
pn
=
1
p
113
Un hecho de gran importancia es este: Si p, q son primos, ser divisible por p y
por q son eventos tecnicamente independientes, es decir, D[E
p

E
q
] = D[E
p
]D[E
q
].
Una de sus consecuencias (no tan inmediata) es que los divisores primos de n se
distribuyen de acuerdo a la ley normal (ver [10]).
7.3 Orden de Magnitud
Necesitamos un mecanismo exible para comparar funciones. Esto es necesario,
porque a menudo nos interesa reemplazar funciones complicadas con otras m as
simples. En la parte practica, esto nos permite establecer terminos de error en una
estimacion, de una manera m as exible.
Para comparar dos funciones f y g, es conveniente primero denir la relacion
(se lee dominada por): Decimos que
f (x) g(x) conforme x
si podemos encontrar una constante C y x
0
tal que
f (x) Cg(x) cuando x > x
0
Para establecer esta desigualdad, a menudo es muy util usar el hecho de que si f
es creciente, entonces a b cuando f (a) f (b). A veces se puede usar la derivada
para establecer que f es creciente (o decreciente) en un intervalo.
EJEMPLO 7.4 Muestre que 3x
3
x
2
+1 x
3
conforme x
Solucion: Tenemos que encontrar C y x
0
tal que x
3
x
2
+1 Cx
3
cuando x x
0
.
Como 1 x
2
< 0 si x > 1, entonces 3x
3
x
2
+1 < 3x
3
si x > 1. Por tanto basta
tomar C = 3 y x
0
= 1 para que se cumpla la denicion.
EJEMPLO 7.5 Muestre que exp(
_
log(x)) x conforme x
Solucion: exp(x) y log(x) son funciones crecientes, entonces
exp(
_
log(x)) x (tomando logaritmos)
_
log(x) log(x) (cuadrados a ambos lados)
log(x) log
2
(x), ()
ahora, como u u
2
si u 1, la desigualdad () se cumple si x >e. Por tanto, basta
tomar C = 1, y x
0
= e para que se cumpla la denicion.
114 ESTIMACIONES, ESTAD

ISTICAS Y PROMEDIOS
O grande de Landau
En general, nos interesa una manera de decir que f y g son funciones parecidas en
orden excepto por un termino de error dominado por una funci on h. Decimos que
f (x) = g(x) +O(h(x)) si | f (x) g(x)| h(x)
En particular, si g(x) 0,
f (x) = O(h(x)) si f (x) h(x)
EJEMPLO 7.6 Observe que f (x) =O(1) signica que f es una funci on acotada en un
intervalo ]x
0
, [. Tambien, usando los dos ejemplos anteriores, 3x
3
x
2
+1 =O(3x
3
)
y exp(
_
log(x)) = O(x).
EJEMPLO 7.7 Muestre que
x
x +1
= 1+O
_
1
x
_
.
Solucion: |x/(x +1) 1| = 1/(x+1) < 1/x si x > 0. As, tomando C = 1 y x
0
= 0, el
termino de error es O(1/x).
EJEMPLO 7.8 Sean n, d enteros positivos, Muestre que n/d[ = n/d + O(1).
Solucion: Por el algoritmo de la divisi on, existe k, r Z tal que n =k d +r con 0
r <d o tambien n/d =k+r/d. Luego, n/d[ =k =(nr)/d. Ahora, |n/d[ n/d| =
r/d < 1 para cada n 0. As, tenemos n/d[ = n/d + O(1), tomando C = 1.
o peque na
La denicion de O grande requiere la existencia de una constante C tal que f Cg.
La denicion de la o peque na es similar, solo que esta vez pedimos que 0 f Cg
para toda C > 0. En lo que sigue, solo hacemos referencia un par de veces a este
concepto, as que solo vamos a dar la denicion.
Denici on 7.3 Sea f , g funciones. Decimos que f =o(g) si para toda c R
+
, existe
x
c
tal que 0 f (x) c g(x) si x > x
c
.
7.4 Teorema de los n umeros primos
Ya sabemos que los primos son innitos. De aqu en adelante hay una pregunta muy
natural: cuantos primos hay entre 2 y x?. Por ejemplo, 2, 3, 5, 7 son los primos
Introducci on a la Teora de N umeros.. Walter Mora F.
Derechos Reservados 2009 Revista digital Matem atica, Educacin e Internet (www.cidse.itcr.ac.cr/revistamate/)
115
inferiores a x = 10, as que hay 4 primos entre 2 y 10.
La funci on que se usa para contar los primos por debajo de x se denota con (x) :
Por ejemplo, (2) = 1, (10) = 4 y (

1000) = 11.
Para la funci on (x) no hay una formula sencilla. Algunas formulas actuales son
variaciones un poco m as ecientes que la formula recursiva de Legendre (1808).
F ormula de Legendre.
Esta formula esta basada en el principio de inclusion-exclusi on. Basicamente dice
que el conjunto {1, 2, ..., x[} es la uni on del entero 1, los primos x y los enteros
compuestos x,
x[ = 1+(x) +#{ enteros compuestos x}
Un entero compuesto en A ={1, 2, ..., x[} tiene al menos un divisor primo menor o
igual a

x. Esto nos ayuda a detectar los n umeros compuestos en A : solo tenemos
que contar los elementos de A con un divisor primo

x.
Los n umeros divisibles por p en inferiores a x son los k n umeros p < 2p < ... <
k p x. Como kp x <(k+1)p, entonces k =[x/p_. As, [x/p_ cuenta la cantidad
de enteros x divisibles por p.
Ahora, #{ enteros compuestos x} es igual a al conteo total de los m ultiplos de
cada primo p
i

x? No, pues este conteo incluye a los propios primos p


i
, as que
hay que reponer con (

x) para hacer una correcci on. Pero tambien habra que


restar los compuestos que son divisibles por p
i
y p
j
pues fueron contados dos veces,
pero esto hara que los n umeros divisibles por p
i
, p
j
, p
k
fueran descontados una vez
m as de lo necesario as que hay que agregar una correcci on para estos n umeros, y
as sucesivamente.
EJEMPLO 7.9 Si x = 30, los primos menores que [

30_ = 5 son 2, 3 y 5.
[30/2_ = 15 cuenta {2, 4, 6, 8, 10, 12, 14, 16, 18, 20, 22, 24, 26, 28, 30}
[30/3_ = 10 cuenta {3, 6, 9, 12, 15, 18, 21, 24, 27, 30}
[30/5_ = 6 cuenta {5, 10, 15, 20, 25, 30}
En el conteo [30/2_+[30/3_+[30/5_:
se contaron los primos 2, 3 y 5.
6, 12, 18, 24, 30 fueron contados dos veces como m ultiplos de 2, 3
10, 20, 30 fueron contados dos veces como m ultiplos de 2, 5
15, 30 fueron contados dos veces como m ultiplos de 3, 5
30 fue contado tres veces como m ultiplo de 2, 3 y 5.
116 ESTIMACIONES, ESTAD

ISTICAS Y PROMEDIOS
Finalmente,
#{ enteros compuestos 30} = [30/2_+[30/3_+[30/5_
[30/(2 3)_[30/(2 5)_[30/(3 5)_
+ [30/(2 3+5)_
= 313532+1 = 19
El ultimo sumando se agrega pues el 30 fue contado tres veces pero tambien se resto
tres veces.
Observe ahora que en {1, 2, ..., 30} hay 19 compuestos y el 1, as que quedan 10
primos.
Sea p
i
el iesimo primo. La formula de Legendre es,
1+(x) =
_
x
_
+[x_

p
i

x
_
x
p
i
_
+

p
i
<p
j

x
_
x
p
i
p
j
_


p
i
<p
j
<p
k

x
_
x
p
i
p
j
p
k
_
+
Para efectos de implementaci on es mejor poner = (

x) y entonces la formula
queda
1+(x) =
_
x
_
+[x_

i
_
x
p
i
_
+

i<j
_
x
p
i
p
j
_


i<j<k
_
x
p
i
p
j
p
k
_
+
EJEMPLO 7.10 Calcular (100)
Solucion: Como

100 = 10, solo usamos los primos {2, 3, 5, 7}.


1+(100) = (10) +100
(100/2+100/3+100/5+100/7)
+100/2 3+100/2 5+100/2 7+100/3 5+100/3 7+100/5 7
(100/2 3 5+100/2 3 7+100/2 3 7+100/3 5 7)
+100/3 3 5 7
= 4+100(50+33+20+14) +(16+10+7+6+4+2) (3+2+0+1) +0 = 26
El problema con esta formula es la cantidad de c alculos que se necesita para calcular
las correcciones.
Las cantidad de partes enteras [x/(p
i
1
p
i
2
p
i
k
)_ corresponde a la cantidad de
subconjuntos no vacos {i
1
, i
2
, , i
k
} de {1, 2, ..., }, es decir, hay que calcular
2

1 partes enteras.
117
Si quisieramos calcular (10
33
), entonces, puesto que

10
33
= 10
18
, tendramos
que tener los primos 10
18
y calcular las partes enteras
_
x/(p
k
1
p
k
2
...p
k
j
)

que
corresponden al c alculo de todos los subconjuntos de {1, 2, ..., (10
18
)}. Como (10
18
) =
24739954287740860, tendramos que calcular
2
24739954287740860
1 partes enteras.
que constituye un n umero nada razonable de c alculos.
Estimaci on de (x). Teorema de los n umeros primos.
El c alculo de (x) de manera directa es bastante complicado y requiere mucho
esfuerzo computacional. En general, no podemos responder de manera exacta todo
el tiempo. Curiosamente, hay formulas relativamente simples para responder con
una aproximacion del valor de (x) para valores grandes de x. Legendre y Gauss
iniciaron el estudio de esta estimacion contando primos en intervalos de longitud
adecuada y calculando proporciones, en busca de un ley que gobernara esta distribucion.
La frecuencia relativa (n)/n calcula la proporci on de primos en el conjunto A =
{1, 2, ..., n}. Aunque la distribucion de los primos entre los enteros es muy irregular,
el comportamiento promedio si parece ser agradable. Coo dijimos antes, basandose
en un estudio emprico de tablas de n umeros primos, Legendre y Gauss (en 1792,
a la edad de 15 a nos) conjeturan que la ley que gobierna el cociente (n)/n es
aproximadamente igual a
1
ln(n)
.
En [9] se indica que Gauss y Legendre llegaron a este resultado, de manera independiente,
estudiando la densidad de primos en intervalos que dieren en potencias de diez:
notaron que la proporci on de primos en intervalos centrados en x = 10
n
decrece
lentamente y disminuye aproximadamente a la mitad cada vez que pasamos de x a
x
2
. Este fenomeno es muy bien modelado por 1/ln(x) pues 1/ln(x
2
) = 0.5/ln(x).
n (n) (n)/n 1/ln(n)
10
7
664579 0.0664579 0.0620420
10
11
4118054813 0.0411805 0.0394813
10
12
37607912018 0.0376079 0.0361912
Tabla 7.4
Acerca de este descubrimiento, Gauss escribio a uno de sus ex-alumnos, Johann
Franz Encke, en 1849
Cuando era un muchacho considere el problema de cuantos primos haba
hasta un punto dado. Lo que encontre fue que la densidad de primos alrededor
de x es aproximadamente 1/ln(x).
118 ESTIMACIONES, ESTAD

ISTICAS Y PROMEDIOS
La manera de interpretar esto es que si n es un n umero cercano a x, entonces es
primo con probabilidad 1/ln(x). Claro, un n umero dado es o no es primo, pero
esta manera de ver las cosas ayuda a entender de manera muy intuitiva muchas
cosas acerca de los primos.
Lo que arma Gauss es lo siguiente: Si x es peque no comparado con x (en el
mundillo asintotico esto quiere decir que x/x 0 conforme x ) entonces
(x +x) (x)
x

1
ln(x)
((x+x)(x))/x es la densidad de primos en le intervalo [ x, x+x] y 1/ln(x) es
el promedio estimado en este intervalo. Por esto decimos: 1/ln(x) es la probabilidad
de que un n umero n, en las cercanas de x, sea primo. Para hacer un experimento,
podemos tomar x =

x (que claramente es dominada por x),


x (x +x) (x)
(x +x) (x)
x
1
ln(x)
10 2 0.632 0.434
100 4 0.4 0.217
1000 5 0.158 0.144
10000 11 0.11 0.108
100000000000 12491 0.0395 0.039
1000000000000 36249 0.0362 0.036
Tabla 7.5 Densidad de primos en le intervalo [ x, x +x] con x =

x
Hadamard y de la Vallee Poussin probaron en 1896, usando metodos basados en
analisis complejo, el
Teorema 7.5 (Teorema de los N umeros Primos)
Sea Li(x) =
_
x
2
dt
ln(t)
. Entonces (x) Li(x), es decir lim
x
(x)
Li(x)
= 1
La conjetura de Legendre era (x) x/ln(x). Esta expresi on se usa mucho cuando
se hacen estimaciones gruesas:
Teorema 7.6 Li(x) x/ln(x), es decir lim
x
(x)
x/ln(x)
= 1
7.4.1 La funci on Zeta de Riemann
Este tema esta en el ambito de la teora analtica de n umeros. Aqu solo podemos
hacer una excursi on algo descriptiva con solo algunos c alculos concretos que involucran
119
a la funci on zeta de Riemann. Los resultados que se mencionan aqu fueron
tomados de [9] y [2].
La aproximacion a (x) dada por Gauss y Legendre fue encontrada por metodos
empricos. Riemann fue el primero en deducir de manera sistematica relaciones entre
los n umeros primos y las funciones matematicas conocidas. El punto de partida de
Riemann fue la relacion descubierta por Euler
(s) =

n=1
1
n
s
=

p primo
1
1p
s
(7.1)
donde el producto es tomado sobre todos los primos.
Para entender esta formula debemos aplicar series geometricas,
1
1p
s
= 1+ p
s
+(p
s
)
2
+...
As,

p primo
1
1p
s
= (1+
1
2
s
+
1
2
2s
+...)
(1+
1
3
s
+
1
3
2s
+...)
(1+
1
5
s
+
1
5
2s
+...)

EJEMPLO 7.11 Veamos un ejemplo concreto. Si s = 1 entonces
(1) =

n=1
1
n
=

p primo
1
1p
1
= (1+
1
2
+
1
2
2
+...)
(1+
1
3
+
1
3
2
+...)
(1+
1
5
+
1
5
2
+...)
(1+
1
7
+
1
7
2
+...)

As, el sumando
1
450
se obtiene como
1
2 3
2
5
2
=
1
2

1
3
2

1
5
2
1 1 .
El producto de los dos primeros factores sera,
120 ESTIMACIONES, ESTAD

ISTICAS Y PROMEDIOS
(1+ p
1
1
+ p
2
1
+...)(1+ p
1
2
+ p
2
2
+...) = 1+
1
p
2
+
1
p
2
2
+
1
p
3
2
+
1
p
2
p
1
+
1
p
2
2
p
1
+
1
p
3
2
p
1
+
1
p
1
+
1
p
2
p
2
1
+
1
p
2
1
+
1
p
2
p
3
1
+
1
p
2
2
p
2
1
+
1
p
3
2
p
2
1
+
1
p
2
2
p
3
1
+
1
p
3
2
p
3
1
+
1
p
3
1
Luego, (1) =

1
2

1
3

2
p

n
n
donde la suma cubre todas las combinaciones de
exponentes
i
0 y todos los primos p
i
. El teorema fundamental de la aritmetica
dice que estos productos en los denominadores son todos los enteros positivos:
(1) =

n=1
1
n
.
Riemann toma esta identidad establecida por Euler y pone a trabajar la teora
de funciones analticas (funciones diferenciables de variable compleja). Extiende
la relacion (7.2), la cual esta restringida a s > 1 por razones de convergencia, a
s = +it con > 0 y s = 1 (en este caso |(1)| = ). La nueva funci on luce as
(s) =
1
12
1s

n=1
(1)
n1
n
s
(7.2)
Esta funci on converge para todos los s = 1 con > 0 si s = 1. Para calcular (s)
se usa la formula de Euler-Maclaurin
(s)
N1

n=1
n
s
+
1
s 1
N
1s
+
1
2
N
s
+
1
12
N
s1

s(s +1)(s +2)


720
N
s3
+
s(s +1)(s +2)(s +3)(s +4)
30240
N
s5
,
Por ejemplo, tomando N = 1000,
(2) 1.6449340668482264...
2
/6 = 1.6449340668482262...
(1/2+37.586178 i) =8.91019785731472810
8
2.943779272013280510
7
i
En realidad, 0.5+37.586178158825675... i es el sexto cero no trivial de , es decir,
(1/2+0.5+37.586178158825675... i) = 0. A este respecto, la famosa hip otesis de
Riemann dice que todos los ceros no triviales de (s) son de la forma s =1/2+it. La
importancia de esta hip otesis se debe a que la estimacion del error en varias formulas
relacionadas con la distribucion de los n umeros primos depende del conocimiento
de regiones extensas libres de ceros de la funci on (s). En particular,
(x) = Li(x) +O(

xlnx)
121
Para obtener una formula para (x), Riemann dene la funci on f (x) = (x) +
1/2(x
1/2
) +1/3(x
1/3
) +..., con x > 1 y no entero. Sorprendentemente,
(x) =

n=1
(n)
n
f (x
1/n
) (7.3)
En 1859 Riemann hace la conjetura
f (x) = Li(x)

Li(x

) ln2+
_

x
dt/(t(t
2
1)lnt)
donde la suma corre sobre todos los ceros no triviales de (s), contando multiplicidad.
Esto fue probado por Mangoldt en 1895. Ahora, cambiando f (x
1/n
) por Li(x
1/n
)
en (7.3), Riemann obtiene
Ri(x) =

n=1
(n)
n
Li(x
1/n
) (x)
En esta formula, es conveniente calcular Li(x) =Ei(Log(z)) donde Ei(z) =
_

z
e
t
/t dt;
suponiendo que tenemos una buena implementaci on de esta funci on, por ejemplo
en ([12]).
La funci on Ri(n), n Z
+
, se puede calcular usando la serie de Gram (1893),
Ri(n) = 1 +

k=1
(logn)
k
k! k(k +1),
esta serie exhibe convergencia muy veloz; sin embargo, la aproximacion a (x) es
aceptable si x < 10
9
.
Qu e tan bien se puede aproximar (x)?
El teorema de los n umeros primos indica que (x) Li(x), es decir, el error relativo

Li(x) (x)
Li(x)

0 conforme x 0.
Efectivamente, conforme x es grande, Li(x) se aproxima m as y m as a (x). Si x no
es muy grande, se puede tener un error porcentual peque no y un error real de varios
millones, que a un as, es despreciable respecto a la magnitud de (x). En la tabla
(7.7) se hace una comparaci on entre (x) Li(x). Los valores de (x) se obtuvieron
de tablas especiales mientras que Li(x) se calcul o con Ei(x).
x (x) Li(x) Li(x) (x)
(x) Li(x)
Li(x)
10
13
346065536839 346065645810. 108 971 3.1410
7
10
18
24739954287740860 24739954309690415. 21949555 8.87 10
7
10
22
201467286689315906290 201467286691248261498. 1932355207 9.5910
12
Tabla 7.6 Comparando (x) con Li(x)
122 ESTIMACIONES, ESTAD

ISTICAS Y PROMEDIOS
Una mejora notable se obtiene si cambiamos Li(x) por Ri(x),
x Li(x) (x) Ri(x) (x)
10
13
108971. 5773
10
18
21949555. 3501366
10
22
1932355207. 127132665
Tabla 7.7 Comparando (x) con Ri(x)
7.4.2 Teorema de Mertens.
En este apartado vamos a aplicar algunos c alculos aproximados para establecer un
resultado muy curioso: Tpicamente, los n umeros grandes tienen factores primos
peque nos.
Observemos que en el conjunto {1, 2, 3, ..., 9} solo 3, 6 y 9 son divisibles por 3.
En terminos de proporciones, una tercera parte. La tabla que sigue muestra las
proporciones al variar n. Es esta tabla, d
n
denota la cantidad de enteros positivos
n que son divisibles por 3.
n d
n
/n 1/3
8680 0.33329493087557605
76333 0.33332896650203714
554615 0.333332131298288
Tabla 7.8 Proporci on de n umeros divisibles por 3.
Como 1/p es la proporci on aproximada de n umeros, en el conjunto {1, 2, ..., n},
divisibles por p, 1 1/p sera la proporci on de n umeros en este conjunto que no
son divisibles por p.
Aqu estamos asumiendo demasiado porque esta proporci on no es exactamente
1/p. Este n umero solo es una aproximacion. Si ser divisible por p es un evento
independiente de ser divisible por q,
_
1
1
p
__
1
1
q
_
sera la proporci on de
n umeros en el conjunto {1, 2, ..., n}, que no son divisibles por p ni por q.
En general,

2pG,
p primo,
_
1
1
p
_
sera una estimacion de la proporci on de n umeros en
el conjunto {1, 2, ..., n}, que no son divisibles por ninguno de los primos menores o
iguales a G. Esto si tiene utilidad practica, como veremos m as adelante.
123
EJEMPLO 7.12 Hagamos un experimento. Sea d
n
=#{mn : m es divisible por 2, 3, 5, o 7}.
n d
n
d
n
/n
103790 80066 0.7714230658059543
949971 732835 0.7714288120374201
400044 308605 0.7714276429592745
117131 90359 0.7714354013881893
124679 96181 0.7714290297483939
Tabla 7.9
La proporci on de n umeros naturales n divisibles por 2, 3, 5 es 0.7714. As,
10.7714 =0.2286 es la proporci on de n umeros en {1, 2, ..., n} que no son divisibles
por los primos 2, 3, 5 y 7.
Y efectivamente,
_
1
1
2
__
1
1
3
__
1
1
5
__
1
1
7
_
= 0.228571.
Si intentamos calcular el producto para cantidades cada vez grandes de primos,
rapidamente empezaremos a tener problemas con el computador. En vez de esto,
podemos usar el
Teorema 7.7 (F ormula de Mertens)

2px,
p primo
_
1
1
p
_
=
e

ln(x)
+ O(1/ln(x)
2
)
es la constante de Euler
Para efectos practicos consideramos la expresi on

2px,
p primo
_
1
1
p
_

e

ln(x)

0.5615
ln(x)
si x (7.4)
EJEMPLO 7.13 Veamos la formula en accion,
124 ESTIMACIONES, ESTAD

ISTICAS Y PROMEDIOS
x

primos p

x
(11/p)
2e

ln(x)
100000 0.0965 0.0975
100000000000000 0.034833774529614024 0.03483410793219253
Tabla 7.10
Tambien, multiplicando (7.4) por 2, la formula
G

3 p,
p primo
_
1
1
p
_

2e

ln(G)

1.12292
ln(G)
nos dara la proporci on aproximada de n umeros impares que no tienen un factor
primo G.
EJEMPLO 7.14 En este ejemplo se muestra que los n umeros grandes sin factores
primos peque nos no son el caso tpico.
G
Proporcion approx de impares
sin factores primos G.
100 0.243839
1000 0.162559
10000 0.121919
100000 0.0975355
1000000 0.0812796
10000000 0.0696682
100000000 0.0609597
1000000000 0.0541864
10000000000 0.0487678
Tabla 7.11
Esta tabla nos informa que tpicamente, los n umeros grandes tienen factores
primos peque nos.
7.5 N umeros Armonicos
125
Aunque la serie armonica

k=1
1
k
es divergente, la funci on H
n
=
n

k=1
1
k
es muy util en
teora analtica de n umeros.
Lema 7.4 Existe una n umero real , llamada constante de Euler, tal que
H
n
= ln(n) + +O(1/n).
Prueba: Hay que mostrar que C tal que 0 < H
n
ln(n) <C 1/n para n > n
0
.
Usando integral de Riemann,
n1

k=1
1
k
=
_
n
1
1
x
dx + E
n
i.e. H
n1
= ln(n) +E
n
2 n
1 1
1 2
n
1
1/n
y = 1/x
(a) (b)
rea = 1/(n-1)
rea = 1/2
n-1
Figura 7.1 Comparando el area ln(n) con la suma H
n
.
Geometricamente, H
n1
corresponde a la suma de las areas de los rectangulos desde
1 hasta n y E
n
la suma de las areas de las porciones de los rectangulos sobre la
curva y = 1/x.
En el gr aco (b) de la gura 7.1 vemos que E
n
1 para toda n 1, as que E
n
es
una funci on de n, que se mantiene acotada y es creciente, por lo tanto esta funci on
tiene un lmite, el cual vamos a denotar con . As, lim
n
E
n
=. En particular, para
cada n jo, > E
n
.
Como E
n
corresponde a la suma (innita) de las areas de las regiones sombreadas
en la gura 7.2, se establece la desigualdad
126 ESTIMACIONES, ESTAD

ISTICAS Y PROMEDIOS
E
n
< 1/n
de donde
0 < (H
n1
ln(n)) < 1/n.
Ahora restamos 1/n a ambos lados para hacer que
aparezca H
n
, tenemos
1
n
> H
n
ln(n) > 0
que era lo que queramos demostrar.
1
1
1/n
.
.
.
Figura 7.2 E
n
.
Aunque en la demostracion se establece H
n
ln(n) <1/n, la estimacion del error
O(1/n) corresponde a una funci on dominada por un m ultiplo de 1/n. Veamos ahora
algunos c alculos que pretender evidenciar el signicado de O(1/n).
n H
n
ln(n) |H
n
ln(n) | 1/n
170000 12.62077232 12.62076938 2.9411735810
6
5.8823529410
6
180000 12.67793057 12.67792779 2.7777752010
6
5.5555555510
6
190000 12.73199764 12.73199501 2.6315766310
6
5.2631578910
6
200000 12.78329081 12.78328831 2.4999979110
6
5. 10
6
Observando las dos ultimas columnas se puede establecer una mejor estimacion del
error con
1
2n
y todava mejor con
1
2n

1
12n
2
!
n H
n
ln(n) + +
1
2n

1
12n
2
100000 12.090146129863427 12.090146129863427
150000 12.495609571309556 12.495609571309554
200000 12.783290810429621 12.783290810429623
Tambien, de estas tablas se puede obtener la aproximacion 0.577216
Lema 7.5
n

k=1
(k) = nH(n) +O(n) y
n

k=1
(k) = nln(n) +O(n).
Prueba: Como (k) =

d|k
1,
n
k=1
(k) =
n
k=1

d|k
1
La idea ahora es usar argumentos de divisibilidad para usar la expansion del ejemplo
7.8. Si d|k entonces k =d c n. Esto nos dice que el conjunto de todos los divisores
127
positivos de los n umeros k inferiores o iguales a n, se puede describir como el
conjunto de todos los pares (c, d) con la propiedad cd n (por supuesto, se puede
hacer una demostracion formal probando la doble implicaci on ).
Ahora, cd n d n c n/d. Entonces podemos escribir,
n

k=1
(k) =

c,d
cdn
1 =

dn

cn/d
1
La suma

cn/d
1 corre sobre los enteros positivos menores o iguales que n/d. Esto
nos da n/d[ sumandos, i.e.

cn/d
1 =n/d[. Finalmente, usando el ejemplo 7.8,
n

k=1
(k) =

dn
[n/d]
=

dn
{n/d +O(1)}
=

dn
n/d +

dn
O(1)
= n

dn
1/d +

dn
O(1)
= nH
n
+O(n)
En los ejercicios se pide mostrar, usando la gura 7.1, que H
n
= log(n) +O(1).
Usando este hecho,
n

k=1
(k) = nH
n
+O(n) = n{ln(n) +O(1)}+O(n) = nln(n) +O(n).
(Los peque nos detalles que faltan se completan en los ejercicios)
7.6 Acerca de los factores de un n umero grande
Los siguientes teoremas, los cuales podemos ver en ([8]), nos dan informaci on acerca
de que se podra esperar cuando se intenta factorizar un n umero grande. Aqu hay
que tener cuidado: Las interpretaciones de los teoremas no son del todo rigurosas,
solamente son argumentos heursticos para obtener estimaciones gruesas.
Teorema 7.8 Sea
k
(x) el n umero de enteros x que tienen exactamente k factores
primos diferentes, k 2. Entonces

k
(x)
6

2
x
lnx
(lnlnx)
k1
(k 1)!
=

(x) cuando x
128 ESTIMACIONES, ESTAD

ISTICAS Y PROMEDIOS
La aproximacion

(x) de
k
(x) funciona si k =(1+o(1))lnlnx, es decir, si k esta en
un vecindario de lnlnx. Por ejemplo, si tomamos x = 10
100
y k = 15, la proporci on
de n umeros compuestos (de la totalidad de los compuestos inferiores a x) no da
0.15%. Esto no dice que los n umeros cercanos a x =10
100
, con 15 o m as factores,
no son muy populares.
Otro teorema util es el siguiente,
Teorema 7.9 Normalmente, el n umero de factores primos diferentes de N es
aproximadamente, lnlnN
En este teorema,Normalmente signica que la mayora de los enteros cercanos a
N tienen una cantidad de factores primos entre (1 )lnlnN y (1 +)lnlnN con
> 0.
Ahora, siguiendo un argumento heurstico, podemos concluir que tpicamente, la
cantidad de dgitos del factor primo m as grande de N es aproximadamente un 63%
de la cantidad de dgitos de N. Si P es el factor primo m as grande de N, puesto
que logN es proporcional a la cantidad de dgitos de N, esta estimacion se puede
poner como logP 0.63logN.
La heurstica es muy sencilla, si N tiene s factores, N/P tendra s 1 lnlnN/P =
lnlnN +ln(1 lnP/lnN) = s +ln(1 lnP/lnN). Entonces, tomando logaritmo, 1
lnP/lnN 1/e. Luego, lnP (11/e)lnN = 0.632lnN.
En particular, para el segundo factor primo P
2
de N, tpicamente tendramos
logP
2
0.23logN.
Para terminar, vamos a hablar un poco del teorema de Erd os-Kac. El teorema
del lmite central dice que si una poblaci on (continua o discreta) tiene media y
varianza nita
2
, la media muestral X tendr a una distribucion que se aproxima
a la normal.
Teorema 7.10 (Limite Central) Si tenemos X
1
, X
2
, ..., X
n
variables aleatorias independientes,
identicamente distribuidas, con media y varianza
2
, entonces, si n es sucientemente
grande, la probabilidad de que S
n
= X
1
+X
2
+... +X
n
este entre n +

n y
n+

n es
1

2
_

e
t
2
/2
dt
EJEMPLO 7.15 Si lanzamos una moneda limpia unas 10000 veces, uno esperara que
aproximadamente 5000 veces salga cara. Si denotamos con X
i
= 1 el evento en
EJERCICIOS 129
el lanzamiento i sale cara, como la probabilidad que asumimos para el evento sale
cara es 1/2, entonces n = n 0.5 = 5000 y =

n 0.25 = 5. Luego, para calcular


la probabilidad de que el n umero de caras este entre 4850 y 5150, debemos calcular
los lmites y . Por razones de ajuste del caso discreto al caso continuo, se usa un
factor de correcci on de 1/2. Resolviendo, 5000+()

50 =48500.5 ==3.01
5000+()

50 = 5150+0.5 = = 3.01
1

2
_
3.01
3.01
e
t
2
/2
dt = 0.997388
As, la probabilidad de que el n umero de caras este entre 4850 y 5150 es de 0.997388
Si (n) denota la cantidad de factores primos de n, esta funci on se puede denotar
como una suma de funciones
p
(n), estadsticamente independientes, denidas por

p
(n) =
_
1 si p|n
0 si p n
Esto sugiere que la distribucion de los valores de (n) puede ser dada por la ley
normal (con media lnlnn y desviaci on estandar

lnlnn).
Mark Kac y Paul Erd os probaron que la densidad del conjunto de enteros n para el
cual el n umero de divisores primos (n) esta comprendido entre lnlnn+

lnlnn
y lnlnn+

lnlnn, es
1

2
_

e
t
2
/2
dt
es decir, el n umero de divisores primos esta distribuido de acuerdo a la ley normal.
Teorema 7.11 Denotamos con N(x, a, b) la cantidad de enteros n en {3, 4, ..., x} para
los cuales

(n) lnlnn

lnlnn

Entonces, conforme x ,
N(x, a, b) = (x +o(x))
1

2
_

e
t
2
/2
dt
EJERCICIOS
7.1 Muestre que 7x
3
12x +9 x
3
conforme x . Ayuda: Use la derivada.
7.2 Si f , g : N R
+
, muestre que Max{f (n), g(n)} = O( f (n) +g(n)).
7.3 Muestre que 2
n+1
= O(2
n
) pero 2
2n
= O(2
n
)
130 ESTIMACIONES, ESTAD

ISTICAS Y PROMEDIOS
7.4 Muestre que f O(g) no implica necesariamente que g O( f )
7.5 Si f (n) 1 y lg[g(n)] 1 entonces muestre que si f O(g) = lg[ f ]
O(lg[g])
Ayuda: Use la hip otesis para concluir que lg f (n) lg c+lg g(n) (lg c+1)lg g(n).
7.6 Usando la gura 7.1, muestre que H
n
= log(n) +O(1).
7.7 Muestre que

d|n
(d)(n/d) = 1
Bibliografa
[1] R. Carmichael. The Theory of Numbers. 1er Ed. John Wiley and Sons,
1914.
[2] P. Ribenboim. The Little Book of Bigger Primes. Springer, 2004.
[3] T. Koshy. Elementary Number Theory with Applications. 2da. Ed. Aca-
demic Press, 2007.
[4] N. Koblitz A course in number theory and cryptography. 2da ed.,
Springer,1994.
[5] H. Cohen Number theory. Volume I: Tools and Diophantine Equations.
Springer, 2007.
[6] H. Cohen A course in computational algebraic number theory. Springer,
1996.
[7] Lindsay N. Childs. A Concrete Introduction to Higher Algebra. Springer-
Verlag New York, 1995.
[8] G.H. Hardy, J.E. Littlewood. An Introduction to Theory of Numbers.
Oxford Univ. Press. 1938.
[9] Hans Riesel. Prime Numbers and Computer Methods for Factorization.
Springer; 2 edition. 1994.
[10] Mark Kac. Statistical Independence in Probability Analysis and Number
Theory. John Wiley and Sons, Inc. 1964.
[11] Harold Stark, An introduction to number theory. The MIT Press, 1987.
[12] William H. Press et all,NUMERICAL RECIPES. The Art of Scientic
Computing. Third Edition. Cambrifge University Press.
[13] S. Y. Yan. Number Theory for Computing. 2nd edition. Springer. 2001.
[14] Eric Weisstein, Polygonal Number. MathWorldA Wolfram Web Re-
source. http://mathworld.wolfram.com/PolygonalNumber.html
[15] Jim Delany, Geometric Proof of the Tetrahedral Number Formula.
The Wolfram Demonstrations Project.
http://demonstrations.wolfram.com/GeometricProofOfTheTetrahedralNumberFormula/
Introducci on a la Teora de N umeros.. Walter Mora F.
Derechos Reservados 2009 Revista digital Matem atica, Educacin e Internet (www.cidse.itcr.ac.cr/revistamate/)

You might also like